Zur-Natur-der-Zeit





Warum die Zeit wahrscheinlich netzartige Struktur hat

   


Zur Natur der Raumzeit

   


Zeitreisen in die Vergangenheit

   


Ist die Zeit tatsächlich, was sie zu sein scheint?

   


Alte und neue Vorstellungen zum Wesen der Zeit

   


Zur Relativität von Zeit und Raum

   





D i s k u s s i o n


 Beitrag 0-421
1944: Das erste Betriebssystem eines elektronischen Rechners bestand aus 6 Frauen, die man » Computer « nannte

 
 

 
Das Betriebssystem des ersten elektronischen Allzweckrechners

 
 
bestand aus 6 Frauen, die man » Computer « nannte. Sie waren die ersten Programmierer im heutigen Sinne und gleichzeitig auch Compiler bzw. Interpreter ihrer Programme.
 
Lies mehr dazu auf Seite » Als Computer noch weiblich waren: Die Pioniere des Programmierens waren sechs Frauen — als Bediener des ersten elektronischen Universalrechners der Welt. Sein Nutzer war (1945) das US-amerikanische Militär. «
 

 
 
 
Die ersten Programmierer waren Frauen, die man » Computer « nannte
 
 
Jean Jennings: Mathematiker, Programmierer und Bediener des ersten kommerziellen elektronischen Allzweckrechners (1945)
 

 
 
 
Diesen Computer — den Röhrenrechner ENIAC — zu programmieren, bedeutete, ständig seine Verkabelung abzuändern:
 
 
 
Den Röhrencomputer ENIAC zu programmieren, bedeutete, ihn fortlaufend neu zu verkabeln
 
Quelle: NZZ


 

  Beitrag 2097-38
Erzeugt erst Licht die Zeit?

 
 
Harti in 2097-1:
Hallo zusammen,

einerseits bewegt sich das Licht mit einer endlichen Geschwindigkeit von rund 300 000 km/sec; d.h. es bewegt sich in Raum und Zeit
andererseits
vergeht für das Licht bei seiner Bewegung keine Zeit (es bewegt sich nur im Raum).

Wie ist dieser Widerspruch aufzuklären ?

MfG
Harti


Hallo Harti,

Raum, Zeit und Geschwindigkeit scheinen Begriffe zu sein, die man mit einem Gerüst vergleichen kann, das wir um ein Haus herum gebaut haben, welches es zu studieren gilt als etwas, das ein uns unbekannter Architekt in grauer Vorzeit geschaffen hat.

Licht (als Phänomen) wäre das Haus, von dem ich hier spreche, und es genau zu studieren erscheint notwendig, da nicht klar ist, ob die Physik das Licht seinem Wesen und Zweck nach wirklich schon voll verstanden hat.

Was mir beim Versuch, mehr Verständnis zu erreichen, so durch den Kopf geht, ordnet sich am ehesten dem Stichwort » Licht im räumlichen Sinne « zu. Du kannst ja dort mal nachlesen.


Gruß, grtgrt
 

  Beitrag 2078-1
Zur so überaus engen Verwandtschaft von Licht und Zeit

 
 

Mehrere Fragen zu Licht und Zeit


Schwarze Löcher — so denkt man — müssten schwarz sein, da ja sogar von ihnen eingefangenes Licht nicht mehr entkommen kann.

In John Barrow: Einmal Unendlichkeit und zurück (2004) aber liest man:

Zitat von Barrow, S. 113:
 
Ein großes Schwarzes Loch sieht eher rot als schwarz aus, da das Licht, das einen außen stehenden Beobachter von der Schicht knapp über dem Ereignishorizont erreicht, beim Versuch, dem Schwerefeld zu entkommen, viel Energie verloren hat, wodurch sein Farbspektrum in Richtung rot verschoben ist.

Er fährt fort und sagt:

Zitat:
 
Ganz gegen einen weit verbreiteten Glauben sind Schwarze Löcher nicht unbedingt kompakte Objekte. Große Exemplare, wie man sie im Zentrum von Galaxien vermutet, sind milliardenfach schwerer als unsere Sonne, aber trotzdem ist ihre Dichte geringer als die von Luft! Wir könnten ihren Ereignishorizont glatt durchqueren und würden nichts Seltsames bemerken. Nur ein Versuch umzukehren würde scheitern.


Damit ist klar: Schwarze Löcher sind nicht einfach nur Ansammlungen von Materie, sondern sind Stellen der Raumzeit, an denen sich Energie in wohl jeder denkbaren Form sammelt und zusammenballt, ganz so wie sich in Vertiefungen der Erdoberfläche Wasser zu Seen und Meeren sammelt.

Bisher dachte ich immer, jedes Photon sei eine Welle, die als Energieportion unteilbar sei, sich also nur ganz oder gar nicht mit einem anderen Elementarteilchen — einem Elektron etwa — vereinigen könne. Wie aber kann diese Vorstellung verträglich sein mit der Tatsache, dass Licht, welches nahe an einem Schwarzen Loch vorbeizieht, seine Wellenlänge vergrößert bekommt?

Kann mir das jemand erklären?


Und noch ein Gedanke geht mir durch den Kopf: Das Licht wird ja nicht, wie man denken könnte, schneller oder langsamer, wenn das Loch an ihm zerrt (die Licht­geschwindigkeit ist stets konstant). Nun ist aber Geschwindigkeit Quotient aus örtlichem Abstand und entsprechendem zeitlichen Abstand, und so bleibt nur der Schluss, dass durch die Gegenwart großer Energie-Ansammlung räumliche und zeitliche Abstände im selben Ausmaß, genauer: um jeweils denselben Faktor, verkleinert werden.

Damit scheint die Zeit eine Dimension des Universums zu sein, die sich ihrer Qualität nach in rein gar nichts von den rein räumlichen Dimensionen unterscheidet.
  • Müsste sie dann aber nicht infolge der ständig stattfindenden Expansion des Universums ebenso gedehnt werden wie der Raum?
  • Wenn ja, könnte das die Erklärung dafür sein, dass man Atomuhren wirklich als etwas sehen kann, das einen im gesamten Universum einheitlich anwendbaren Zeitmaßstab darstellt?
  • Warum aber kann man Zeit — anders als andere Dimensionen — in nur  e i n e r  Richtung durchqueren?
    Licht könnte gut einziger Träger der Zeit sein. Schließlich und endlich bettet es ja die Vergangenheit uns ferner Sterne in unsere Gegenwart ein (!).

 

  Beitrag 2085-3
Wie die Zeit entsteht und der Kosmos sich wandelt

 
 
In Ergänzung von Beitrag 2085-1 würde man heute vielleicht allgemeiner sagen:


Wie die Zeit entsteht



Zeit entsteht, wo sich  V e r ä n d e r u n g  ergibt.

Veränderung ergibt sich überall dort, wo der Kosmos — als Summe von Potentialwellen gesehen — seinen Wert ändert.

Mit andere Worten:

Zeit entsteht  ü b e r a l l ,

aber genau besehen nicht überall in exakt gleichem Ausmaß oder exakt gleicher Ausprägung.


Der Kosmos in Vergangenheit und Gegenwart ist die Summe dessen, was alle Wirkungsquanten, die ihre Wirkung schon entfalten konnten, bewirkt haben

( Wirkung, so eine Axiom der Quantenphysik, ist grundsätzlich gequantelt )



Gebhard Greiter (2013)

siehe auch:  [ Gequantelte Zeit ]  und  [ Die Raumzeit der Quanten ]

 

  Beitrag 2085-14
Doch wie definiert sich die Zeit aus Sicht der Natur (aus Sicht von Einsteins Feldgleichungen)?

 
 
Harti in 2085-9:
 
Unsere Vorstellung von Zeit kommt deshalb letztlich dadurch zustande, dass wir Vorgänge (Geschehensabläufe) zueinander in Beziehung setzen, miteinander vergleichen.
Dieser Vergleich hat sich verselbständigt und zu einem allgemeinen Gefühl entwickelt, dass Vorgänge nicht instantan erfolgen, sondern dauern. Wir meinen sogar, dass Zeit im Sinn von Dauer eine objektive, außerhalb unserer Vorstellung existierende Naturerscheinung ist. Dies sehe ich aus den dargelegten Gründen anders. Es gibt nicht nur dort, wo es keine Objekte/Veränderung gibt keine Zeit, sondern auch dort, wo es keine Menschen gibt.


Hallo Harti,

wenn es um den » Zeitbegriff im Sinne der Menschen « geht, sehe ich das ganz genau so.

Da nun aber Einsteins Gleichungen der ART ebenfalls einen Zeitbegriff kennen und der ganz offensichtlich  n i c h t  in der von dir beschriebenen Weise zustande gekommen sein kann, muss es über den Zeitbegriff der Menschen hinaus wohl doch auch einen der Natur selbst geben. Wie sonst könnte es sein, dass Einsteins Gleichungen die Geometrie des Raumes ebenso wie die Geschwindigkeit des Alterns unterschiedlich beschleunigter Objekte derart verlässlich und genau beschreiben?


Bernhard Kletzenbauer in 2085-11:
Hans-m in 2085-10:
 
Zeit ist die Distanz in der Raumzeit zwischen zwei Ereignissen.(würd´ ich mal so definieren)
 
Das habe ich schon etliche Male hier geschrieben. Zeit ist der Abstand zwischen Zuständen und Ereignissen.


Ja, Bernhard & Hans-m,

auch ich bin dieser Meinung (und habe das sogar hier im Forum und anderswo ja auch schon präzisiert in dem Sinne, dass die Ereignisse, um die es hier geht, die nicht mehr teilbaren sind (die sog. Elementarereignisse im Sinne der Quantenphysik, siehe 2001-6 und 1986-1).


Bauhof in 2085-12:
 
Die Länge der Weltlinie, die eine Person beim Flug von Ereignis zu Ereignis zurücklegt, entspricht der in der Raumzeit ’zurückgelegten’ Eigenzeit der Person.
Und dabei spielt es keine Rolle, ob diese Weltlinie gekrümmt ist oder nicht.

Ungekrümmte Weltlinie: Flug ohne Beschleunigung.
Gekrümmte Weltlinie: Flug mit Beschleunigung.


Das Problem mit diesem Definitionsversuch scheint mir zu sein, dass die Begriffe » Weltlinie « und » Eigenzeit « den Begriff der Zeit ja schon voraussetzen, d.h. sie bauen auf ihm auf und können deswegen schon rein technisch  n i c h t  Teil seiner Definition sein.


Gruß, grtgrt
 

  Beitrag 2085-42
Das nie aufhörende Ticken der Zeit: Ursache, Wirkung, Ursache, Wirkung, Ursache, Wirkung ...

 
 
Quante in 2085-40:
 
Im Kausalen gilt, Ursache-Wirkung. Ursache-Wirkung ...

und die Wirkung wird zugleich ja auch, im Moment ihres Eintretens, zu einer neuen Ursache. Das geht dann immer so weiter, und so weiter... usw.


Richtig, Quante,

und genau der Gedanke hat mich zu meinem Modell RZQ netzartig gestrickter Zeit geführt (siehe hier und anderswo und alles dazu). In ihm ist
  • Ursache = Quantenfluktuation oder die Kollision von Elementarteilchen
  • Daraus sich ergebende Wirkung = das (durch ein Elementarereignis bewirkte) Entstehen eines Paares virtueller Teilchen bzw. das Ersetzen miteinander kollidierender Elementarteilchen durch eine Menge neuer, mit Impuls versehener Elementarteilchen.

Gruß, grtgrt
 

  Beitrag 2085-58
Warum die Zeit uns Menschen modifiziert

 
 
Stueps in 2085-56:
 
Das Zwillingsparadoxon: Unterschiedliches Vergehen von Zeit ist nicht eine subjektive Empfindung, sondern kann sich objektiv manifestieren.


Hi Stueps,

im Sinne meines Modelles RZQ netzartig angelegter Zeit (auf das ich in Beitrag 2085-42 hinwies) wäre die Zeit, die einem an zwei Ereignissen E1 und E2 beteiligten Objekt X zwischen diesen beiden Ereignissen vergeht — und seine Alterung bewirkt — die Menge aller Elementarereignisse, die wenigstes ein Elementarteilchen erzeugt oder zerstört haben, welches nach E1 und vor E2 mit ein Teil von X war.

Die Modifikation, die X zwischen den beiden Ereignissen E1 und E2 erfährt (sein Wachsen, sein Schrumpfen, kurz: sein  A l t e r n ) wäre so wohl recht gut erklärt.

Wie gefällt Dir diese Idee?


Gruß,
grtgrt
 

  Beitrag 2085-98
Sind Zustandsänderung und Zeitenstehung ein und dasselbe?

 
 
Bernhard Kletzenbauer in 2085-96:
Stueps in 2085-93:
 
Sind Zustandsänderung und Zeitentstehung synonym?
Ja


Ja, Stueps,

auch nach meinem Modell RZQ netzartig gestrickter Zeit — siehe die Links in Beitrag 2085-42 — sind Zustandsänderung und Zeitentstehung ein und dasselbe.

Gruß, grtgrt
 

  Beitrag 2085-105
Was kennzeichnet den als Gegenwart zu bezeichnenden Zustand des Universums?

 
 
C... in 2085-95:
 
Folgende Fragen nagen m.E. weiter an unseren bisherigen Definitionsversuchen:

— Was genau ist das "Jetzt"? (Die Gegenwart allein kann es nicht sein, da diese nur ein Zeitpunkt wäre, den man nicht wahrnehmen könnte).
— Warum ist das "jetzt" ausgerechnet "jetzt" und nicht z.B. gestern?
 


Hallo C...,

mein Modell RZQ netzartig gestrickter Zeit — siehe die 3 Links in Beitrag 2085-42 — beantwortet deine Frage so:


Das "Jetzt" ist der Zustand unseres Universums, der aus genau den Elementarteilchen besteht, für die gilt:
  • Sie kennen die Ursache ihres Entstehens (sprich: das Elementarereignis, das sie hervorgebracht hat),
  • wissen aber nicht, mit welch anderen Elementarteilchen sie verschmelzen werden, ihre Existenz zu beenden.

Plakativer ausgedrückt könnte man sagen:


Das » Jetzt « ist ein Zustand des Universums, der als Gewissheit existiert,

ohne dass auch nur einer seiner möglichen Nachfolgezustände ebenfalls Gewissheit ist.



Note: Hier wird vorausgesetzt, dass jeder Zustand des Universums jedem Elementarteilchen eine Wahrscheinlichkeit seiner Existenz zuordnet. Genau dann, wenn diese Wahrscheinlichkeit den Wert 1 hat, spricht man von Gewissheit.

Es würde mich interessieren, was man — und Du, C..., speziell — an dieser Definition des "Jetzt" unbefriedigend finden könnte.


Beste Grüße,
grtgrt

PS: Bitte siehe auch Hans-Peter Dürrs Meinung zu diesem Thema (zitiert in Beitrag 1213-21) sowie 1992-4 und 1992-1.

 

  Beitrag 2085-114
-

 
 
C... in 2085-108:
 
Anmerkung: Die Relativität der Gleichzeitigkeit deutet dabei darauf hin, dass jedes "Jetzt" ein subjektives Jetzt sein muss.


Hallo C...,

deine Argumentation

» ... deutet dabei darauf hin, dass jedes "Jetzt" ein subjektives Jetzt sein muss. «


kann ich als zwingend nur anerkennen unter der Voraussetzung, dass die Zeit linear voranschreitet.
Mein Modell RZQ aber geht davon aus, dass die Zeit sich netzartig ausbreitet (wie ein Kristall, der an jeder Stelle seiner Oberfläche wachsen kann).

Nach RZQ sind Vergangenheit und Gegenwart der Teil des Netzes, der schon geknüpft ist, wohingegen die Zukunft die Menge aller möglichen Varianten für den Teil des Netzes "Zeit" darstellt, über den noch nicht entschieden ist.

In meiner Theorie wird jedes Elementarereignis das Netz weiterknüpfen und so die Menge aller Möglichkeiten, Zukunft zu gestalten, reduzieren. Die Menge möglicher Varianten von Zukunft bleibt dennoch stets unendlich groß (aber natürlich nur dann, wenn man annimmt, dass tatsächlich kein Elementarereignis alles zu Nichts machen kann — und genau das scheint uns der Energie-Erhaltungssatz ja tatsächlich zu garantieren).


Gruß, grtgrt
 

  Beitrag 2085-123
Wie aus dem Netz der Möglichkeiten (für Zukunftsgestaltung) das ausgedünnte Netz vorhandener Tatsachen wird

 
 
C... in 2085-120:
 
wenn die Zeit objektiv ein "Netz der Möglichkeiten" darstellen soll, muss gelten: Wenn es das Netz "vorher" gegeben hat, muss es doch auch dann bestehen bleiben, wenn die zukünftige Zeit subjektiv zur vergangenen geworden ist. Die Möglichkeiten müssen, weil gleichwertig, irgendwo verblieben sein.

Oder es muss einen "Mechanismus" dafür gegeben haben, der die anderen Möglichkeiten beim Übergang in die Vergangenheit zerstört.


Nicht die Zeit ist ein Netz von  M ö g l i c h k e i t e n , sondern nur die Zukunft ist es (der Teil der Zeit also, der noch nicht "gestrickt" wurde).

Natürlich hat auch die Zeit im Bereich Gegenwart und Vergangenheit netzartige Strukrur: Diese Version des Netzes aber ist eine ausgedünnte, die dadurch zustande kam, dass der Kollaps der Wellenfunktion des Universums, zu dem es lokal mit jedem Elementarereignis E kommt, aus den vielen Möglichkeiten für output(E) genau eine ausgewählt hat. Alle anderen sind ersatzlos gestrichen. Eben dies ist der Mechanismus, den Du vermisst.

 

 Beitrag 0-112
Imaginäre Zeit — Ist Hawkings Euklidische Quantengravitation eine Sackgasse?

 
 

 
Euklidische Quantengravitation

 
 
Unter der Theorie Euklidischer Quantengravitation versteht man die Idee von Hartle und Hawking, die Zeit der Raumzeit statt durch reele durch imaginäre Zahlen zu messen.
 
Der Vorteil dieses Ansatzes besteht darin, dass dann
  • die Minkowski-Metrik der Raumzeit zur euklischen Metrik wird
     
  • und die Zeit dann einfach nur als eine weitere Raumdimension gesehen werden kann.

Mit anderen Worten: Hawking und Hartle wollen die Sonderrolle die Zeit beseitigen.
 
Dies gelang ihnen zunächst durch einen mathematischen Trick, dem sie dann aber auch physikalische Bedeutung zumessen. Hawking erklärt das wörtlich so:


Hawking (Zitat):
 
Die Erkenntnis, dass sich die Zeit wie eine weitere Raumdimension verhalten kann, bedeutet, dass wir uns der Frage, ob die Zeit einen Anfang hat, auf ähnliche Weise entledigen können wie derjenigen, ob die Welt einen Rand hat ...
 
Wenn wir die Allgemeine Relativitätstheorie mit der Quantentheorie kombinieren, wird die Frage, was vor dem Anfang des Universums geschah, zu einer sinnlosen Frage.
 


Hartle und Hawking nannten dies die kein-Rand-Bedingung.
 
 
Raum und Zeit als von gleichem Wesen zu sehen, führt dann aber leider auch zu recht unangenehmen Konsequenzen: Die Theorie bekommt Lösungen, welche das ansonsten in der Physik stets unterstellte Postulat der Kausalität verletzen (Wurmlöcher und geschlossene Zeitkurven).
 
Hawking und Hartle denken, dass sich die mikroskopischen Quanteneffekte — also z.B. auch mikroskopische Wurmlöcher — in makroskopischer Sicht wieder herausmitteln und somit in der klassischen Physik bedeutungslos seien.
 
Andere Physiker geben zwar zu, dass die Theorie der Euklidschen Quantengravitation von ihrem mathematischen Ansatz her aussichtsreich erscheint, meinen dann aber doch, dass sie eher eine Sackgasse sei, da nicht klar ist, wie man von der imaginären Sicht wieder zurück in die reelle findet (mehr dazu in Euclidean Gravity: A lost cause).
 
Dieter Lüst betont, dass der Ansatz Euklidischer Quantengravitation weder in Konkurrenz zur kanonischen Schleifenggravitation noch zur Stringtheorie steht: Er lasse sich problemlos kombiniert mit beiden anwenden.
 
 
 
Quelle: Dieter Lüst: Quantenfische, DTV 2014, S. 245-246
 
 
Nebenbei: Die Idee, die Zeit mit der imaginären Einheit zu multiplizieren, um so die Raumzeit mit der euklidischen Metrik auszustatten, mag künstlich und unmotiviert erscheinen. Entscheidend aber ist, dass das dieses Rezept — es geht zurück auf Richard Feynman — die richtigen Antworten für die Wahrscheinlichkeiten von Teilchenreaktionen liefert. Das lässt sich sogar exakt beweisen, wie zwei mathematische Physiker, der Schweizer Konrad Osterwalder und der Deutsche Robert Schrader zeigen konnten: Die Eigenschaften einer herkömmlichen Quantentheorie, die auf der Raumzeit der Speziellen Relativitätstheorie definiert ist, lassen sich exakt aus dem Feynman-Rezept für eine korrespondierende, imaginärzeitige Raumzeit rekonstruieren.


 

 Beitrag 0-498
Wie steif ist die Raumzeit unseres Universums?

 
 

 
Wie steif ist die Raumzeit?



Günter Spanner (2016):
 
Gravitationswellen nachzuweisen ist extrem schwierig, da die Raumzeit extrem steif ist:
 
Unter dem Elastizitätsmodul eines Stoffes versteht man eine Zahl, welche charakterisiert, wie stark sich aus diesem Stoff bestehende Objekte einer Verformung widersetzen.
 
Gewöhnlicher Gummi z.B. kann leicht gedehnt oder zusammengedrückt werden. Hartgummi zu verformen ist schon weit schwieriger. Holz erreicht bereits den 100-fachen Wert von gewöhnlichem Gummi und ist somit kaum mehr komprimierbar. Bei Stahl ist es das 2000-fache, bei Diamant sogar das 12000-fache.
 
Dennoch ist die Härte von Diamant nichts im Vergleich zur » Härte « der Raumzeit:
 
 
Der Elastizitätsmodul der Raumzeit entspricht dem 1022-fachen der Härte von Diamant.

 


 
 
Die Arme des Gravitationswellen-Detektors LIGO sind 4 km lang, und man kann noch registrieren, wenn sich diese Länge auch nur um 1 Tausendstel des Protonen­durchmessers ändert. Das ist so, als würde man die Entfernung zwischen Alpha Centauri und unserer Sonne (4,3 Lichtjahre) millimetergenau bestimmen können.
 
 
Quelle: Günter Spanner: Das Geheimnis der Gravitationswellen, Kosmos-Verlag (2016), S. 73


 

  Beitrag 1209-25
Ist die Natur der Raumzeit wirklich schon voll verstanden?

 
 
Bernhard Kletzenbauer in 1209-22:
 
Raum ist für mich das imaginäre, leere (unendlich große, nicht von Wänden begrenzte) Gefäß.

RaumZeit ist für mich der Inhalt, der das Gefäß vermutlich völlig ausfüllt (zusätzlich zu Materie und Strahlung). Dieser Inhalt ist auf direktem Weg bisher mit keinem Mittel meßbar.

Genauer habe ich es hier beschrieben:
Beitrag 52-42

Der Vergleich hinkt gewaltig, aber ich sehe Raum als das leere Trinkglas - und RaumZeit als das Bier.


Hi Bernhard,

mir scheint, dein » Inhalt, der das Gefäß ausfüllt « könnte  Kraftpotential  sein: das Potential des Vakuums, Potentialwellen zu erzeugen — das also, was Quanten­fluktuation hervorruft.

Die Raumzeit scheint vergleichbar mit der Oberfläche eines Sees: Sie/er kann Wellen bilden, hat dieses Potential aber nur deswegen, weil es unter der Oberfläche Tiefe gibt.

Was die entsprechende » Tiefe « ist, über der die Raumzeit — als das, was verformbare Oberfläche ist — sitzt, wäre sicher überlegenswert.

Die Summe aller in unserem Universum vorhandenen Energie könnte dem Volumen des Sees unter seiner Oberfläche entsprechen.

Gruß, grtgrt

 

  Beitrag 2110-27
Zeit und Raum aus Sicht der Lebewesen (Hartis Argumentation)

 
Hallo Grtgrt,

um der Frage, was das Wesen der Zeit ausmacht, näher zu kommen, könnte man auch von der Entwicklung des menschlichen Geistes ausgehen.

Was die Menschen von der Natur zunächst wahrgenommen haben, waren Veränderungen. Bäume wachsen, die Sonne wandert über den Himmel, es wird hell und dunkel. Im Laufe ihrer geistigen Entwicklung haben sie gelernt, diese Veränderungen mit zwei Kategorien ( Vorstellungen) zu erfassen und zu beschreiben.
Sie haben die Vorstellung entwickelt, dass Veränderungen dauern. Diese Vorstellung entsteht dadurch, dass während eines Vorgangs zahlreiche andere Vorgänge stattfinden. Daraus ist eine allgemeines Gefühl für Dauer (Zeitgefühl) entstanden.
Sie haben aufgrund ihrer dreidimensionalen Sehfähigkeit eine Vorstellung für Distanzen entwickelt und dies als Raum bezeichnet.
Die Vorstellungen von Raum und Zeit sind zunächst vollkommen unabhängig voneinander entwickelt worden und werden auch heutzutage bei Veränderungen meist unabhängig von einander betrachtet.
Beispiel: Veränderung eines Neugeborenen zu einem Erwachsenen. Zeitliche Veränderung ca. 20 Jahre, räumliche Veränderung ca. 140 cm.
Es ist nicht üblich, die Beziehung zwischen räumlicher und zeitlicher Veränderung (Wachstumsgeschwindigkeit) in diesem Beispiel zu Gegenstand von
von Überlegungen zu machen.
Es ist allerdings so, dass jede Veränderung, genau genommen, die Betrachtung mit beiden Kategorien erfordert; denn nichts passiert instantan und absolute räumliche Ruhe ist eine Fiktion (mathematische Idealvorstellung). Die getrennt Betrachtung der Kategorien von Raum und Zeit erfolgt also lediglich aus Zweckmäßigkeitsgründen, weil die eine oder andere Kategorie in dem jeweiligen Zusammenhang nicht interessiert.
Dies ist bei Bewegungen grundsätzlich anders, weil diese nur mit ihrem räumlichen und zeitlichen Anteil vollständig beschrieben werden können.

Aus diese Überlegungen folgere ich, dass die Aussage, Raum und Zeit seien mit dem Urknall entstanden nicht zutreffend ist. Mit dem Urknall hat das Universum begonnen sich zu verändern. Die Vorstellungen von Raum und Zeit sind erst im Laufe der geistigen Entwicklung der Menschen zur Beschreibung der Veränderungen in der Natur entwickelt worden.
MfG
Harti
 

  Beitrag 2094-1
Raum, Zeit und Raumzeit existieren nur auf gedanklicher Ebene — konkret sind nur Ereignisse (Raumzeitpunkte)

 
 
Hans-Peter Dürr sagt uns klar, dass Materie nur eine sich makroskopisch ergebende Illusion ist. Ähnliches gilt auch für Raum und Zeit sowie ihre Summe, die Raumzeit.

Die Tatsache nämlich, dass zueinander unterschiedlich schnell bewegte Beobachter stets die gleiche Lichtgeschwindigkeit messen, kann der Relativitätstheorie nach nur so gedeutet werden, dass man feststellt:


Den als » Raumzeit « bezeichneten Hintergrund, auf dem sich alles physikalische Geschehen abspielt,

gibt es nur in Form beobachterspezifischer Illusionen R(B).



Hierbei bezeichnet B den Beobachter, welcher R(B) als eine (n+1)-dimensionale Umgebung seiner selbst sieht, deren Struktur durch Geodäten und deren Abstandsbegriff durch die Minkowski-Metrik gegeben ist.

    Die Geodäten verhalten sich wie Schienen durch den Raum. Jedes sich bewegende Objekt bewegt sich — in R(B) — grundsätzlich mit Lichtgeschwindigkeit1 und stets nur entlang dieser Schienen. Das Objekt zu beschleunigen bedeutet, die Schienen zu verbiegen (eine wichtige Erkenntnis Albert Einsteins).
    In der SRT, die ja grundsätzlich nur unbeschleunigte Bewegungen betrachtet, ist jede dieser Schienen eine Gerade im üblichen (euklidischen) Sinne, und genau des­wegen hat die Raumzeit der SRT auch keine Singularitäten: Ihre Struktur ist die eines euklidischen Vektorraumes.
    Beobachter B, so kann man sich vorstellen, ruht hinsichtlich eines geeignet gewählten Koordinatensystems K für R(B) im Ursprung von K, und K macht R(B) zu einem Bezugssystem R(B,K), in das sich alle durch B grundsätzlich beobachtbaren Ereignisse (Synonym: Raumzeitpunkte) einordnen.
    Eine der n+1 Dimensionen von R(B,K) modelliert, was B als seine » Uhrzeit « sieht. Über die in Beitrag 2089-45 beschriebene Regel lässt die sich verallgemeinern zu einem wohldefinierten Uhrzeitbegriff auf der Menge  a l l e r  dem B grundsätzlich bekannt zu machender Ereignisse (Raumzeitpunkte).

SRT und ART sagen uns, welche Eigenschaften die durch diese Bezugssysteme R(B,K) erzeugten Sichten haben und wie sich raumzeitliche Entfernungen beim Wechsel von einem ins andere transformieren in Abhängigkeit davon, wie schnell sich die ihnen zugeordneten Beobachter relativ zueinander bewegen und welcher Beschleunigung jeder einzelne unterliegt.

FAZIT also:


Raum, Zeit und Raumzeit existieren nur als rein gedankliche Konstruktion.

Von konkreterer Existenz sind lediglich Ereignisse (auch Raumzeitpunkte genannt),
die sich über beobachterspezifisch gedachte, mathematisch präzisierbare Netze verbiegbarer Geodäten relativ zueinander angeordnet sehen.

Soweit diese Ereignisse unteilbare Wirkung haben, erzeugen oder vernichten sie Elementarteilchen.

Jedes Elementarteilchen ist einfach nur Potentialwelle.




1 Siehe Seite 50 in Brian Greenes Buch The Elegant Universe, wo man liest:

Zitat:
 
When an object moves throgh space relative to us, its clock runs slow compared to ours. That is, the speed of its motion through time slows down. Here’s the leap: Einstein proclaimed that all objects in the universe are always travelling through spacetime at one fixed speed — the speed of light.

This is a strange idea; we are used to the notion that objects travel at speeds considerably less than that of light. We have repeatedly emphasized this as the reason relativistic effects are so unfamiliar in the everyday world. All of this is true.

We are presently talking about an object’s combined speed through all four dimensions — three space and one time — and it is the object's speed in this generalized sense that is equal of the speed of light.
 


Gebhard Greiter, Okt..2013
 

  Beitrag 2094-13
Die 3 Grundformen aller Existenz

 
 
Haronimo in 2094-12:
 
Deine, und Stueps seine, durch einige Beispiele nachvollzogene Gedanken, werden nur unter den Hinweis auf experimentell durchgeführte Wissenschaft, Akzeptanz erlangen. Wir müssen vielleicht, die Assoziationen zwischen manche Theorien intensiver betrachten.


Vor allem sollten wir klar unterscheiden
  • Dinge und Phänomene, die allein nur logischer Verstand konstruiert (z.B. Raum, Zeit, Raumzeit),
  • Dinge und Phänomene, deren Existenz wir unseren Sinnen verdanken (z.B. anfassbare Objekte, Lärm, Wärmegefühl, Gefühle) und
  • Dinge und Phänomene, die keines von beidem sind, aber doch in der Natur vorkommen (Energie, Wellen, atomar eintretende Wirkung).


Dinge, die nur unser Verstand konstruiert, werden auch Experimente uns  n i e  zeigen können.


 

  Beitrag 2094-17
Fiktion, Illusion — nicht wirklich passende Worte

 
 
Horst in 2094-16:
 
So ein Schalk aber auch der Schöpfer, gaukelt uns lauter Illusionen vor .......wenn er nicht selbst auch nur ne Illusion ist!:smiley9:


Hallo Horst,

nur für den — wie mir scheint recht unwahrscheinlichen — Fall, dass Du doch mal den Wunsch haben solltest, wirklich zu verstehen, sei gesagt:

Worte wie » Illusion « (oder ersatzweise » Fiktion «) gebrauche ich, wenn ich über Physik spreche, nur widerwillig — einfach nur deswegen, weil mir kein besserer Begriff einfällt.

Was ich sagen möchte, wenn ich davon spreche, dass Materie, Raum oder Zeit nur Fiktion (Illusion) seien, ist einfach nur:

Unsere Sinne täuschen uns eine wesentlich konkretere Existenz dieser Dinge vor als tatsächlich vorhanden.



Unsere Sinne glauben zu verstehen, und doch verstehen sie die wahre Natur solcher Dinge einfach nur falsch.

So lange es um so einfache Dinge wie Pakete von Potentialwellen geht, bemerken Physiker das irgend wann. Was aber, wenn es um ein so komplexes Wesen geht, wie unser Schöpfer es ist?

Gruß, grtgrt
 

  Beitrag 2094-22
-

 
 
Horst in 2094-21:
Grtgrt im Beitrag 2094-20

Zitat:
Wenn jemand etwas betrachtet und später — nachdem er es genauer betrachtet hat — feststellt, dass es seiner Natur nach was ganz anderes ist als das, was er zunächst dachte, dann bezeichne ich diese zweite Natur des Betrachteten als die, die der Wahrheit näher kommt. (Ob das dann schon wirklich die  l e t z t e Wahrheit ist, bleibt natürlich stets offen).

Hallo Grtgrt

Thema verfehlt,
denn ich habe dich im Verdacht, dass du diese Erkenntnisse bei der genauen Betrachtung deiner, oder der Frauen im Allgemeinen gewonnen hast? :smiley32:

Gruß Horst


Hi Horst,

die Tatsache, dass aus dir ein solch wahrhaft philosophischer Vergleich kommt, scheint mir das erste überzeugende Argument dafür zu sein, dass Du wohl doch mehr sein musst als nur ein kompliziertes Paket von Potentialwellen ...

Aber wie ich schon sagte: Der wirklich  l e t z t e n  Wahrheit können wir uns nie sicher sein.

Beste Grüße,
Gebhard
 

  Beitrag 2080-40
Blockuniversum und Eternalismus

 
Grtgrt in 2080-35:
 
Henry in 2080-33:
 
... er [Einstein] war später der Ansicht, seine Raumzeit sei real, was ihn sogar soweit brachte, ein Blockuniversum als wahrscheinlich anzunehmen.


Hallo Henry,

es würde mich sehr wundern, wenn es eine Literaturstelle gäbe, in der Einstein selbst den Begriff "Blockuniversum" in den Mund nimmt.




Zeitpunkte im Sinne der ART machen stets nur  l o k a l  Sinn,

und so sind Zeitscheiben im Sinne des Blockuniversums gar keine wohldefinierten Konzepte.


 

Hallo, Gebhard!

"Später hatte sich Einstein den – damals noch nicht so genannten – Eternalismus ebenfalls zu eigen gemacht. 1952 betonte er im 5. Anhang zur 15. Auflage seines Buchs "Relativity: The Special and General Theory", dass es natürlicher erscheint, die physikalische Realität als eine vierdimensionale Existenz zu denken statt wie bisher als Entwicklung einer dreidimensionalen Existenz." ist ein Zitat über Einstein aus Focus online, ich kannte den Zusammenhang aus anderer Quelle, mir ist aber nicht mehr klar, woher.

"Für uns gläubige Physiker hat die Scheidung zwischen Vergangenheit, Gegenwart und Zukunft nur die Bedeutung einer wenn auch hartnäckigen Illusion." ist ein Zitat von Albert Einstein aus dem Jahre 1955 und bezieht sich auf den Eternalismus, der als philosophiisches Konzept eine Konsequenz der SRT ist.

Beide Zitate beziehen sich auf das "Blockuniversum".

Was stets übersehen wird, ist, dass die Raumzeit als GESAMTHEIT real ist, und nur in diesem Zusammenhang macht auch die Auffassung des Universums als Blockuniversum tatsächlich Sinn, denn nur, wenn Vergangenheit, Gegenwart und Zukunft gleichberechtigt real sind, macht es Sinn, von einem "Blockuniversum" zu sprechen. Das hat nichts mit der "Lokalität" zu tun, sondern damit, dass unterschiedliche Beobachter sich nicht über die Gleichzeitigkeit von Ereignissen einigen können (was natürlich nicht wörtlich zu nehmen ist, denn wenn sie über die Hintergründe wissen, können sie sich auch einigen, Lorenz-Transformation) bzw. wegen der verschiedenen Weltlinien der Beobachter, die sich aus der Bewegung der Beobachter zueinander ergeben.
 

 Beitrag 0-464
Wie unser subjektives Zeitgefühl entsteht

 
 

 
Unser subjektives Zeitgefühl

ergibt sich, da Nervenzellen ermüden und uns das bewusst wird

 
 


Görnitz & Görnitz (2015):
 
Hin und wieder begeben Personen sich freiwillig über längere Zeit hinweg in abgeschlossene Räume oder Höhlen, um herauszubekommen, wie sich ihr natürliches Zeitempfinden ohne Kontakt nach außen gestaltet:
     
  • Einer der ersten und bekanntesten war der damals erst 23-jährige Geologe Michael Siffre, der 1962 ununterbrochen 61 Tage unter der Erde in einer Höhle verbrachte (natürlich mit Essen und Trinken), und dem dann 25 Tage in seiner Zeitwahrnehmung fehlten.
     
  • Ein Jahrzehnt später wiederholte er diesen Versuch über 205 Tage hinweg und musste dann feststellen, dass es seiner Aufzeichnung nach ganze 2 Monate weniger waren.

 
Wir sehen: Ohne Außenreize verläuft die wahrgenommene Zeit ganz offensichtlich langsamer. Dem Bewusstsein fehlen — der Isolation wegen — Ereignisse, durch die für uns die Zeit unabhängig vom eigenen Verhalten gegliedert und als fortschreitend erkannt wird. Kurz:
 
 
Wahrgenommene Zeit ist wahrgenommene Veränderung.

 
 
Nebenbei: Man kann annehmen, dass auch eine unter freiwilligen Bedingungen beschlossene Isolation bei manchen Mensch nicht ohne psychische Folgen bleibt. So ist zumindestens nicht auszuschließen, dass das bei einer Schweizerin, die 111 Tag lang isoliert gelebt hatte, nach 1 Jahr zum Selbstmord führte.
 
 
Wegen des komplexen Verhältnisses der Psyche zur Zeit wird verstehbar, warum Einzelhaft der Folterung von Gefangenen dienen kann: Wo sie sich über lange Zeit hinweg in kalten, dunklen Räumen fast ohne Anhaltspunkt für objektiv verstrichene Zeit aufhalten müssen, kann das bei Überlebenden psychische und somatische Leiden hervorrufen, die oft schwer in Worte zu fassen sind.
 
Andererseits berichtet Karlfried Graf Dürkheim (1896-1988) in einem Interview über seine als glückhaft empfundene, mehr als 1-jährige Einzelhaft in Japan. Er war Diplomat, sei aber als vermeintlicher Spion von dem Amerikanern nach dem Kriege festgesetzt worden. Seine gute Erinnerung an jene Zeit führt er darauf zurück, dass er dort ungestört arbeiten und meditieren konnte (und froh darüber war).
 
Später trug er wesentlich zur Verbreitung des Zen und der Zen-Meditation in der Bundesrepublik bei.
 
 
 
Zellermüdung: Unsere biologische Uhr

 
 
In unserer Kultur haben wir gelernt, so manche 2-dimensionale Zeichnungen als Bild eines 3-dimensionalen Gegenstandes zu interpretieren. Wenn man etwa gegen einander versetzte Quadrate gleicher Größe und Ausrichtung an ihren 4 Ecken mit jeweils einer Linie verbindet, erscheint uns dies als Darstellung eines Würfels (sog. Necker-Würfel).
 
Wenn wir nun bewusst versuchen, das eine der Quadrate als "das vordere" zu sehen, so wird uns dennoch schon nach wenigen Sekunden das andere als das vordere erscheinen.
 
Der Grund hierfür: Das Festhalten der Gegenwart — ohne Gliederung durch zwischenzeitlich eintretende, bewusst wahrgenommene Ereignisse — kann nur andauern, solange nichts unsere Aufmerksamkeit ablenkt. Sobald Nervenzellen aber ermüden, wird uns das bewusst und so dieser "zeitfreie" Zustand beendet und die Informationsverarbeitung (= Interpretation dessen, was wir sehen) dann durch andere, frische Zellverbände durchgeführt.
 
Eine solch "andauernde Gegenwart" — die erst mit dem Erkennen eines neuen Faktums endet — kann sich beim Menschen durch meditative Übung ergeben oder in Situationen, in denen wir zu fokussiert sind, um anderes Geschehen um uns herum noch mitzubekommen.
 
Wir können also sagen, dass eine "Gegenwart" immer wieder durch einen neue "Gegenwart" abgelöst wird. Allerdings ist uns das meist nicht so deutlich bewusst wie hier beschrieben. Präsent ist uns i.A. nur, dass wir die Vergangenheit ebenso wie die Zukunft als wesentlich größere Zeiträume empfinden als die jeweilige Gegenwart.
 
Ziemlich gut isoliert von der Umwelt ist unser Bewusstsein. Wenn wir darüber nichts erzählen, kann es von außen nur mit großem technischen Aufwand und nur sehr ungefähr erkannt werden. Nur wegen dieser seiner Isolierung können wir gelegentlich eine andere Zeitwarnehmung haben, als die, welche eine Normaluhr anzeigt. Solch "ausgedehnte Gegenwart" wird für unser Bewusstsein aber nicht beliebig lange dauern können: unter normalen Umständen nur bis etwa 3 sec (wie Experimentalpsychologen beobachtet haben).
 


 
Quelle: Thomas und Brigitte Görnitz: Von der Quantenphysik zum Bewusstsein, Springer 2015, S. 444-446
 
Klein S.: Zeit — Der Stoff, aus dem das Leben ist. Eine Gebrauchsanleitung, Fischer 2006, ab Seite 20


 

  Beitrag 2073-4
Uhren als Beobachter der Zeit — sie beobachten Taktgeber

 
 
Horst in 2073-3:
 
Nehmen wir an, der physikalische Beobachter sei eine Uhr, falls sie ein Messgerät für "Zeit" sein sollte.

Diese Uhr, das physikalische Objekt B, beobachtet "die Zeit" mit der es also kollidieren muß, um durch die entstandene Spur ein Messergebnis zu erkennen.

Beschreibe mir doch bitte mal den physikalischen Vorgang, wie durch Kollision des "Beobachters Uhr"mit der "Zeit" das Uhrwerk dahingehend beeinflusst wird, dass ein Messergebnis und vor allem welches zu erkennen ist.


Hallo Horst,

die Uhr kollidiert nicht mit der Zeit. Die nämlich ist, was die Uhr angeht, nur gedankliche Abstraktion: eine Abstraktion, die uns die Uhr liefert als Übersetzung und Präsentation ihrer Beobachtungsergebnisse.

Was die Uhr tatsächlich beobachtet, ist ein geeigneter Taktgeber:


Zitat von Wikipedia:
 
Uhren können die Zeit umso genauer angeben, je konstanter die Schwingung ihres Taktgebers ist.
  • Bei Räderuhren dient als Taktgeber das Pendel oder die Unruh,
  • bei der Quarzuhr ist es ein Oszillator, dessen Frequenz mit Hilfe des Schwingquarzes konstant gehalten wird.
  • In Atomuhren macht man sich die Eigenschaft von Atomen zu Nutze, beim Übergang zwischen zwei Energiezuständen elektromagnetische Wellen einer bestimmten Frequenz abzustrahlen oder zu absorbieren.


PS: Bitte beachte auch: Beobachter im Sinne des quantenphysikalischen Messprozesses beobachten Quanten (allgemeiner: Quantensysteme). Taktgeber sind Quantensysteme, die Zeit aber ist sicher keines (!).

 

  Beitrag 2073-8
Die Zeit existiert nur als Abstraktum

 
 
Horst in 2073-7:
Hallo Grtgrt

Gehe ich recht in der Annahme, dass du mit dem Taktgeber ein mechanisches oder elektronisches Gebilde meinst und mit dem Zähler die Uhrzeiger oder die Digitalanzeige?


Ja, Horst, so meine ich das ( mehr Beispiele im Wikipedia-Zitat aus 2073-4 ).

Horst in 2073-7:
 
Dann ist es natürlich klar:

Zitat:
Weder Taktgeber noch Zählwerk kennen den Begriff "Zeit".
Damit ist von einer Uhr abgelesene "Zeit" etwas Abstraktes, der Uhr völlig Unbekanntes.

Die Frage ist daher, ist denn der Begriff "Zeit" für den Bewerter, den subjektiven Beobachter, etwas völlig Bekanntes, wenn Du doch sagst:

Zitat:
Er wird durch  u n s  gemacht und ist reine Abstraktion dessen, was die Uhr uns (über welche Art von Bild auch immer) als zum Zählerstand äquivalent
zeigt.

Im Klartext, ohne  u n s  gäbe es den Begriff "Zeit" nicht, weder als Abstraktum noch als reales Objekt!
Dann wäre ja auch die sogenannte (!) "Raumzeit" nur ein Produkt menschlichen abstrakten Denkens!

Oder ist die mit dem Taktgeber Uhr nachzuweisen?


Für den subjektiven Beobachter — wenn Du dir darunter den Nutzer der Uhr vorstellst — ist "Zeit" etwas Nichtmaterielles,
  • das sich verbraucht
  • und das er quantifiziert sieht durch den Zähler der Uhr als Vielfaches eines Taktschrittes im Sinne des Taktgebers (oder einer darauf aufbauenden Definition der Begriffe "Sekunde", "Minute", "Stunde" usw.).

Mit der "Raumzeit" oder der "Zeit" im Sinne der Raumzeit hat das zunächst gar nichts bzw. nur wenig zu tun. Beide sind rein mathematisch erdachte Konstrukte, mit Hilfe derer wir versuchen, durch uns beobachtetes Verhalten der Natur zu modellieren und soweit zu verstehen, dass Vorhersagen möglich werden: Sie  e n t s p r e c h e n  etwas, dessen wahre Natur auch die Physiker bisher NICHT verstehen ( siehe [1 und [2 ), das sie aber noch am ehesten mit fortlaufender Veränderung identifizieren.

Was ich selbst über "Zeit" im Sinne der Natur zu sagen weiß, findet sich in [3 und ist streng genommen nur im Rahmen beobachter- und aspektspezifischer Sichten quantifizierbar.

Auf jeden Fall ist Zeit nichts, mit dem irgend ein Beobachter interagieren könnte. Zeit ist, in welchem Kontext auch immer, ein Abstraktum.

Gruß, grtgrt

 

 Beitrag 0-56
Was uns Martin Bojowald (als Physiker) zur Natur der Zeit sagt

 
 

 
Was Bojowald über das Wesen der Zeit sagt


Martin Bojowald ( 2012, S. 262-267 seines Buches "Zurück vor den Urknall" ):
 

Das mathematische Bild der Zeit ist hilfreich, aber  i d e a l i s i e r t .
 
Eine solche Zeit nämlich wird nicht beobachtet; was wir stattdessen sehen, sind Änderungen von Materiekonfigurationen wie des Standes der Sonne relativ zur Erde oder des Zeigers der Uhr relativ zum Ziffernblatt.
 
Uhren jeder Art sind aus Materie aufgebaut und unterliegen dem Einfluss durch andere Materie.
 
Man spricht von einer guten Uhr, wenn der Einfluss anderer Materie darauf gering ist. Doch schwachen Einfluss gibt es immer, womit das Bild einer von der Materie vollkommen unabhängigen Zeit idealisiert ist.
 
Es wird immer nur Materie beobachtet, nie aber Zeit direkt. Dies ist vor allem in der Kosmologie bedeutsam, denn während sich unter den heutigen Bedingungen auf der Erde leicht brauchbare Uhren herstellen lassen, kann es in den frühen Phasen des Universums Situationen geben, in denen wegen extrem hoher Dichte unausweichlich ein starker Einfluss zwischen allen Materiebestandteilen vorherrscht. In solchen Fällen kann man als Maß für die Zeit höchstens kosmologische Größen nutzen wie etwa das sich ändernde Volumen des Universums.
 



 

 Beitrag 0-63
Durch die Natur konstant gehaltene Schwingungen des Energiefeldes liefern uns die Zeit

 
 

 
Quantenmechanik — unsere bisher genaueste Theorie

 
 
Seltsamerweise ist die Quantentheorie — obgleich sie für alles von ihr Vorhergesagte doch nur Wahrscheinlichkeiten nennt — das erfolgreichste und genaueste je von Menschen ersonnene physikalische Modell. Seine Vorhersagen sind oftmals bis auf den zehnmilliardsten Teil akkurat.
 
Erst diese Genauigkeit macht die technischen Wunder der heutigen Zeit möglich: Laser, Computer, Mobilfunk, Mikrowellenherd und vieles mehr.
 
Und nicht zuletzt auch ganz unglaublich genau gehende Uhren ( jetzt und demnächst ).
 
 
Da solche Uhren den Fluß der Zeit anhand der Schwingungen von Quanten zeigen, wird klar:
 
 
Durch die Natur konstant gehaltene Schwingungen des Energiefeldes liefern uns die Zeit.
 
Diese scheint der Grund dafür zu sein, dass für zueinander nicht beschleunigte Beobachter die Zeit gleich schnell voranschreitet.


 

 Beitrag 0-Zeit
Die Zeit: In welchem Sinne existiert sie wirklich?

 
 

 
Zeit, Realität und Wirklichkeit

 
Gebhard Greiter, Jan 2016

 
 
Unsere Umgangssprache kennt keinen Unterschied zwischen den Begriffen Realität und Wirklichkeit. Im Englischen z.B. gibt es gar nur ein einziges Wort dafür.
 
Philosophen, Neurologen und Quantenphysiker aber berichten uns übereinstimmend , dass — wer genau darüber nachdenkt — einen ganz gravierenden Unterschied zwischen Realität einerseits und Wirklichkeit andererseits entdeckt:

     
  • Niels Bohr wurde nicht müde, seine Studenten immer wieder darauf hinzuweisen, dass die Physik uns nicht sagen kann, wie die Natur funktioniert, sondern nur darüber nachdenkt, wie sie auf uns wirkt, d.h. wie wir sie wahrnehmen.
     
  • Schon vor ihm hatte auch Kant das so gesehen: Unter "Ding an sich" versteht Kant die Wirklichkeit, wie sie unabhängig von aller Erfahrungsmöglichkeit, für sich selbst besteht, die absolute Realität. Jedenfalls will Kant sagen, daß die Art und Weise, wie sich das Wirkliche sinnlich-kategorial uns darstellt, das Wesen desselben nicht erschöpft. (zitiert aus Rudolf Eislers Nachschlagewerk zu Kant). Kants eigener Wortlaut: Es gibt die Dinge der Erscheinungen und die Dinge an sich. Wir kennen die Dinge nur so, wie sie auf uns wirken..
     
  • Viel früher noch kam Parmenides (geboren um 530 v.Chr.) zu genau derselben Meinung. Er schrieb (Zitat): Die Welt, in der wir zu leben glauben, ist die vermeintliche Welt der Sinneswahrnehmungen; die Welt ist nur Meinung ....

Wichtige Erkenntnis also:

 
Realität ist stets nur Bild der Wirklichkeit,
 
erzeugt von unserem Verstand und daher subjektiv: i.W. nur Meinung.
 
Wo wir uns Modelle für die Wirklichkeit schaffen — in der Physik oder einfach nur in unseren Köpfen —
haben wir damit noch lange nicht entdeckt, wie die Natur tatsächlich funktioniert.

 
 
Wie sich mit Hilfe dieser Einsicht weitere Erkenntnis ergeben kann, sei nun gezeigt am Beispiel der Zeit:

     
     
    Wichtiger Teil der Natur scheint die Zeit zu sein — aber ist sie es wirklich? Könnte sie nicht einfach auch nur Teil unserer Realität sein?
     
    Und in der Tat: Man kann das sogar beweisen. Die genaue Argumentation lässt sich nachlesen an Stelle » Wirklich ist nur Alter — aber nicht die Zeit «. Grundidee der Beweisführung ist: Nach Einsteins Spezieller Relativitätstheorie lässt Zeit sich stets nur beobachterspezifisch (genauer: in Abhängigkeit vom gewählten Bezugssystem) quantifizieren, so dass der erhaltene Wert subjektive Meinung ist. Objektiv quantifizierbar ist nur Eigenzeit — die aber entspricht dem physikalischen Alter des Beobachters im Sinne seiner eigenen Realität.
     
    Statt der Zeit (die subjektiv ist) die Eigenzeit zu betrachten (die eindeutig, also objektiv quantifizierbar ist), erscheint zunächst nur wie ein terminologischer Trick — sie ist ja schließlich eine der vielen möglichen subjektiven Quantifizierungen der Zeit. Oder ist sie doch mehr? Müssen wir Eigenzeit — das Alter eines Objekts im physikalischen Sinne — der Wirklichkeit zurechnen?
     
    Es gibt tatsächlich eine Beobachtung, die dafür spricht, dass Eigenzeit nicht einfach nur Realität, sondern wohl doch auch der Wirklichkeit zuzurechnen ist: Ich meine die Tatsache, dass jedes Elementarteilchen eine genau benennbare, nur von seinem Typ abhängige, immer und überall gleich beobachtete mittlere Zerfallszeit hat. Und damit nicht genug: Auch jedes biologische Lebewesen (wir Menschen sind keine Ausnahme) hat eine für seine Art typische mittlere Lebenserwartung.
     
    Unser Gehirn, welches uns unsere Realität produziert — unser Bild der Wirklichkeit — kann die mittlere Lebenserwartung der Exemplare einer bestimmten Art nun aber nicht in eigener Regie beliebig festsetzen. Damit, so denke ich, muss Eigenzeit sich der Wirklichkeit zuordnen.
     
    Hätte ich damit recht, könnte das bedeuten, dass auch die Physik uns nie wird erklären können, wie Eigenzeit funktioniert (und wie Objektarten zu ihrer für sie typischen mittleren Lebenserwartung kommen, speziell Elementarteilchen zum konkreten Wert ihrer mittleren Zerfallszeit).
     
    Die Wirklichkeit der Zeit scheint sich darin zu erschöpfen, dass jedes sich nicht mit Lichtgeschwindigkeit bewegende Objekt einem ganz bestimmten Zwang zu altern unterliegt. Warum der mit zunehmender Beschleunigung des Objekts geringer wird, ist wohl die eigentlich interessante Frage.
     
    Bisherige Forschung, das Wesen der Zeit betreffend, scheint diesen Aspekt noch nicht gezielt zu adressieren.

 
 
Note: Was Physiker und Philosophen bisher über die Zeit zu sagen wissen, wurde erst kürzlich — und wie es scheint gezielt vollständig — dokumentiert von Lee Smolin (aus Sicht der Theoretischen Physik) und Roberto Mangabeira Unger (aus Sicht der Philosophie) im ihrem Buch:
 
Roberto Mangabeira Unger & Lee Smolin: The Singular Universe and the Reality of Time: A Proposal in Natural Philosophy (2014)


 

 Beitrag 0-194
Zeit im Sinne von Einsteins Theorie ist nur Konzept — nicht Wirklichkeit

 
 

 
Zeit im Sinne der Relativitätstheorie ist nur Konzept

 
 
Gegeben den Unterschied zwischen Realität und Wirklichkeit, liegt es nahe zu fragen:
 
 
Wie wirklich ist die Zeit?

 
 
Wie die Diskussion auf Seite » Relativitätstheorie beschreibt beobachterspezifische Realität « ganz klar zeigt, kann mindestens die Zeit im Sinne der Speziellen Relativitätstheorie nicht wirklich sein.
 
Wenigstens zwei Argumente belegen das:
 


Kip S. Thorne stellt fest:
 
Wenn Sie und ich uns relativ zueinander bewegen, muss das, was ich als Raum bezeichne, eine Mischung aus Ihrem Raum und Ihrer Zeit sein, und das, was Sie Raum nennen, eine Mischung aus meinem Raum und meiner Zeit.
 



Palle Yourgrau nennt auch Gödels Argument:
 
Gödel argues forcefully that in the Special Theorie of Relativity (STR) the relativity of the 'now' (of intuitive time) to an inertial frame implies the relativity of existence: "... the idea of an objective lapse of time (whose essence is that only the present really exists) ..." [Gödel 1949, p. 202, Note 4].
 
But the latter is impossible: "The concept of existence cannot be relativized without destroying its meaning completly." [Gödel 1949, p. 203, Note 5].
 
Hence, if the STR is true, time disappears.
 
 
Sources:
  • Gödel 1949: A Remark about the Relationship between Relativity Theory and Idealistic Philosophy in: Feferman & Solomon, eds.: Kurt Gödel: Collected Works, Vol 2, 1995
  • Yourgrau: Gödel meets Einstein, Preface to the new expanded edition 1999.

 


 
 
Dass auch die Zeit im Sinne von Einsteins Allgemeiner Relativitätstheorie nicht wirklich sein kann, zeigte Gödel nach Pale Yourgrau wie folgt (ausgehend von den durch Gödel entdeckten Lösungen von Einsteins Feldgleichung, den sog. Gödel-Universen (GUs)):
 


Yourgrau (1999):
 
In certain non-expanding GUs, there exist closed timelike wordlines, permitting time travel. But if you can revisit the past, it never 'passed' from existence in the first place. So, once again, time disappears.
 
But the GU is a mere possible world. What about the actual world?
 
Gödel reasons as follows: Our world and the GU differ only in the global distribution of matter and motion. The two universes are described by the same fundamental laws of nature and provide for observers the same experiences (as of) time. So, if time is an illusion in the one world, therefore (as it provably is), it must also be an illusion in the other. In Gödel's own words:
 
"A lapse of time ... would have to be found, one should think, in the laws of nature, i.e. it could hardly be maintained that whether or not an objective laps of time exists depends on the special manner in which matter and its impulse are distributed in the world."
 
 
Sources:
  • Gödel 1946/49: A, B2, C1: Some Observations about the Relationship between Theory of Relativity and Kantian Philosophy in: Feferman & Solomon, eds.: Kurt Gödel: Collected Works, 1995
  • Yourgrau: Gödel meets Einstein, Preface to the new expanded edition 1999.

 



 

 Beitrag 0-178
Die Natur kennt keine Zeit — sie kennt nur objektspezifische Eigenzeit (= Alter)

 
 

 
Wirklich ist nur Alter — aber nicht die Zeit

 
 
Immer noch kann die Theoretische Physik nicht beantworten, was denn nun eigentlich das wahre Wesen der Zeit ist. Der gegenwärtige Stand der Diskussion findet sich — gezielt vollständig und sehr genau — aufgeschrieben in den Büchern
 

Meiner Ansicht nach aber haben Einstein und die Quantenphysik das Rätsel um die Zeit längst gelöst, denn:

 
 
Die Natur kennt keine Zeit — sie kennt nur Alter
 
im Sinne von » Eigenzeit «

 
Beweis hierfür ist folgende Argumentation:
 
Einstein und Bohr haben uns gelehrt, dass es zwischen Wirklichkeit und Realität einen ganz gravierenden Unterschied gibt:
     
  • Wirklichkeit ist das objektiv Vorhandene (dessen Funktionsweise wir aber nicht kennen),
     
  • Realität aber ist der Menschen subjektive Sicht auf die Wirklichkeit (es kann dies die Sicht eines Individums oder auch die Sicht einer ganzen Gruppe von Individuen, beispielsweise einer ganzen Gesellschaft sein, oder gar sämtlicher Menschen überhaupt).
     
    Anders ausgedrückt: Realität ist stets etwas nur durch unseren Verstand Konstruiertes — etwas nicht Wirkliches (siehe Wie real ist die Welt um uns herum?).

Nun weiß man aber:
     
  • Einsteins Spezielle Relativitätstheorie zeigt: Jede Quantifizierung von Zeit ist subjektiv (genauer: abhängig vom gewählten Bezugssystem).
     
  • Seine Allgemeine Relativitätstheorie aber — und insbesondere das sog. Zwillingsparadoxon sowie seine den Physikern ja bekannte Lösung — zeigen:
     
    Da sich das Alter eines Objekts aufgrund seiner Eigenzeit ergibt, ist Alter etwas Objektives, d.h. etwas, das man als wirklich (statt als nur real) anzusehen hat. Experimente im Labor mit schnell im Kreis bewegten, und damit beschleunigten Myonen bestätigen das:
     
    In ihrem Experiment verglichen die Wissenschaftler zehntausend ruhende Teilchen im Labor mit zehntausend schnell bewegten Teilchen im Speicherring.
    Die Messung zeigte, dass nach zwanzig Mikrosekunden kein einziges ruhendes Myon mehr vorhanden war, aber doch noch 8600 bewegte Myonen. Das Zwillingsparadoxon ist hier in spektakulärer Weise realisiert: Die meisten im Kreis bewegten Myonen existieren noch, wenn fast alle ruhenden schon zerfallen sind.
     
    Quelle: http://www.weltderphysik.de/gebiete/theorie/albert-einstein-und-die-relativitaetstheorie/spezielle-relativitaetstheorie/
     
    Siehe auch: Myonen und Spezielle Relativitätstheorie.
     
    Auch die zentralen Ergebnisse der beiden Relativitätstheorien sollte man sich nochmals vor Augen geführt haben.

 
 
Nebenbei: Auf die Frage, » Was ist Zeit? « soll Einstein einmal geantwortet haben » Zeit ist, was man von der Uhr abliest «. Wie meine Argumentation oben zeigt, wissen wir jetzt, dass Einstein damit recht hatte — und das wirklich im buchstäblichen Sinne. Was er sagte, war ganz offenbar mehr als nur eine flapsige Bemerkung (wie klar auch immer ihm selbst das gewesen sein mag). Sein Gefühl dafür, was physikalisch richtig sein mag, war einzigartig.
 
 
Da wir jetzt also wissen, dass Zeit stets nur das subjektiv gefärbte Ergebnis einer Beobachtung von Eigenzeit ist, stellt sich die Frage, warum wir sie in unserem Alltagsleben dennoch dermaßen erfolgreich nutzen. Das aber liegt daran,
  • dass in unserem Alltagsleben die Unterschiede zwischen Zeit und objektspezifischer Eigenzeit gering genug sind, dass man sie vernachlässigen kann
  • bzw. dass wir gelernt haben, sie zu berücksichtigen in jedem Kontext, in dem sie nicht mehr ignoriert werden dürfen (beim GPS-System etwa).
Tatsache also ist: Nicht Zeit, sondern Eigenzeit, ist das, was uns hilft. Eigenzeit aber ist normiert quantifizierte Veränderung des Objekts, dem sie sich zuordnet. Solche Normierung gelingt nur im Fall hinreichend einfach gebauter Objekte (Atomuhren). Wir nutzen sie als Stellvertreter komplizierterer Objekte — wählen sie aber für jedes Objekt so, dass seine Eigenzeit und die der es vertretenden Uhr nur unwesentlich unterschiedlich sind.
 
 
Nebenbei noch: Was für das Paar Zeit und Eigenzeit richtig ist, gilt natürlich auch für das Paar Länge und Abstand: Länge ist das beobachterspezifisch subjektiv gefärbte Ergebnis einer Beobachtung von Abstand.
 
 
Mit der hier angesprochenen beobachterspezifischen Subjektivität ist natürlich stets das vom Beobachter gewählte Bezugssystem gemeint.
 
Natürlich lässt sich der eben beschriebene Sachverhalt aber auch über die Physik hinaus noch als richtig erkennen: Je spannender die Tätigkeit ist, die ein Mensch gerade ausübt, desto schneller wird ihm — seinem Gefühl nach — die Zeit vergehen. Ursache so entstandener Subjektivität ist dann (z.B.), wie oft jene Person sich gedrängt fühlt, auf eine Uhr zu sehen.
 
 
Nebenbei noch: Wie wir jetzt wissen, muss Einsteins Ausspruch
 
 
Zeit ist, was man von der Uhr abliest.

verstanden werden als
 
Zeit ist stets nur, was man von der eigenen Uhr abliest.

 
 
Letztlich ist jedes Objekt seine eigene Uhr.
 
Und tatsächlich schrieb Einstein (1905 in seinem Aufsatz Zur Elektrodynamik bewegter Körper): "Es könnte scheinen, dass alle die Definition der Zeit betreffenden Schwierigkeiten dadurch überwunden werden können, dass ich an Stelle der Zeit die Stellung des kleinen Zeigers meiner Uhr setze."
 
 
Da beschleunigte Objekte im Vergleich zu nicht beschleunigten aus der Sicht Dritter weniger schnell altern, sind die Entfernungen, die ein Mensch während seines Lebens im Prinzip überwinden kann, nur deswegen begrenzt, weil sein Organismus nur begrenzt hohe Beschleunigung auszuhalten im Stande ist.
 
Für Objekte, die sich mit Lichtgeschwindigkeit bewegen, gibt es keine Zeit.

 

 Beitrag 0-180
Welche Eigenschaften hat Eigenzeit? Ist sie wirklich?

 
 

 
Eigenschaften der Eigenzeit

 
 
Auch Einsteins Relativitätstheorie ist nur eine Theorie. Wir können deswegen nicht sicher sein, dass sie in allem recht hat, was sie uns sagt. Legen wir sie aber zugrunde, so haben wir davon auszugehen, dass Eigenzeit folgende Eigenschaften hat:
     
  • Sie ist — mit letzter Genauigkeit wenigstens — nur durch das Objekt selbst beobachtbar.
     
  • Wie viel Eigenzeit ein Objekt verbraucht, um von einem Ereignis E1 zu einem anderen Ereignis E2 zu kommen, ist davon abhängig, welchen Weg durch die Raumzeit es nimmt (Beweis: das Zwillingsparadoxon und seine Lösung durch die ART).
     
  • Der Fluß der Eigenzeit entspricht einer Alterung des Objekts.
     
  • Wir kennen nur ganz wenige Objekte, von denen wir nicht mit Sicherheit sagen können, ob sie unendlich alt werden können. Es sind dies
     
    • der Kosmos (bzw. Universen)
    • sowie alle Elementarteilchen, von denen wir bisher nicht wissen, ob sie zerfallen (Protonen und Neutrinos).

     
  • Nach Einsteins Theorie lässt sich nicht ausschließen, dass Eigenzeit der Wirklichkeit zuzurechnen ist. In dem Fall könnt es gut sein, dass wir nie mehr über sie erfahren, als wir jetzt schon wissen.

 
Dass wir über Eigenzeit tatsächlich schon alles wissen, was die Natur uns offenbart, ist keineswegs klar (selbst dann nicht, wenn sie sich tatsächlich der Wirklichkeit zurechnen sollte).
 
Spekulationen, die es noch zu klären gibt, sind wenigstens folgende:
     
  • Hat tatsächlich  j e d e  Art von Objekten eine durch die Natur gegebene Halbwertszeit?
     
    Diese Vermutung liegt nahe, da wir für alle uns bekannten Elementarteilchen — Protonen und Neutrinos ausgenommen — eine Halbwertszeit beobachtet haben, die sogar noch ihrem Wert nach Naturgesetz zu sein scheint, obgleich dieser Wert spezifisch zur Art der Teilchen ist.
     
    Nur für Protonen und Neutrinos scheint er — wenn sie überhaupt zerfallen — jenseits dessen zu liegen, was für menschliche Experimentalphysik bisher nachweisbar war.
     
    Auch alle uns bekannten Arten biologischer Lebenwesen haben eine für die jeweilige Art typische mittlere Lebenserwartung.
     
    Dies gilt selbst für große unbelebte Objekte, für Sterne etwa, ja sogar für Schwarze Löcher (auch wenn für sie die "Art" durch ihre Masse gegeben erscheint, Masse aber — wie wir wissen — relativ ist).
     
  • Man könnte auf die Idee kommen, daraus zu folgern, dass der Fluß der Eigenzeit — den wir ja auch unter dem Begriff "Alterung" kennen — Objektverschleiß bewirkt.
     
    Auch scheint die Existenz von Eigenzeit immer an die Existenz des Objekts gebunden zu sein, dem sie sich zuordnet. Dies ist der Grund dafür, dass ich in Notiz Wirklich ist nur Alter — aber nicht die Zeit für die Eigenzeit auch den Begriff » Alter « verwende.
     
    Solch physikalisches Objektalter darf natürlich nicht mit dem im Sinne von Biologie oder Medizin verwechselt werden, da letzteres ja auch mit Zerfall von Form zu tun hat, die per Emergenz entstand.

 
 
Ein Spezialfall von Eigenzeit ist

die Eigenzeit nicht beschleunigter Objekte (sog. proper Time): Sie ist nicht-relativ.


Lee Smolin in seinem Buch » Time Reborn « (S. 58-59):
 
If you remove everything corresponding to the observations of particular observers from the description of nature given by Special Relativity, there remains the casaul structure. Since this is all that's observer-independent, it must — if the theory is true — correspond to physical reality [Wirklichkeit]. Hence, to the extent that Special Relativity is based on true principles, the universe is timeless. It is timeless in two senses:
  • There is nothing corresponding to the experience of the present moment,
  • and the deepest description is of the whole history of causal relations at once.
Relationships are the only reality that corresponds to time — relationships of a causal sort.
 
Besides the causal structure, there is another piece of information all observers agree about:
 
Consider a physical clock, which ticks off seconds, floating freely in space. It strikes noon, then a minute later ist strikes a minute past noon. The first event can be considered a cause of the second. In between the clock ticks 60 times. The number of times it ticked between the two events is something else all observers, regardless of their relative motion, can agree about. This is called the proper time.
 
[Smolin ergänzt in Fußnoten 6 und 9, S. 276-277]:
 
This does not mean that all clocks will tick the same number of times between two events. Consider two movings clocks that pass each other when they both read noon, they seperate. One of them accelerates and reverses direction, passing the other clock again when that clock reads 12:01. The accelerating clock will display a different time. But the point is that all observers will agree about how many times one particular clock ticks between two events.
 
The clock that ticks the most times between two events is the one that is free-falling — and
 
because the time a free-falling clock measures is distinguished in this way, we call it the proper time.

 
The geodesics of spacetime, as opposed to space, are the paths that take the most proper time rather than the shortest distance:
 
A free-falling clock ticks faster and thus more often than any other clock traveling between two events. This leads to a good piece of advice: If you want to stay young, accelerate.
 


 
Was Smolin hier sagt, bedeutet: Der zeitliche Abstand zweier Ereignisse, die nicht gleichzeitig stattfinden, ist nicht eindeutig, denn
     
  • sein Wert ist zwar nach oben hin begrenzt
     
  • kann aber jeden noch so kleinen Wert haben (für entsprechend stark beschleunigte Objekte).

 
In Summe lässt sich feststellen:
 
Die Wirklichkeit der Zeit scheint sich darin zu erschöpfen, dass jedes sich nicht mit Lichtgeschwindigkeit bewegende Objekt einem Zwang zu altern unterliegt. Warum der mit zunehmender Beschleunigung des Objekts geringer wird, ist wohl die eigentlich interessante Frage.

 

 Beitrag 0-293
Kann Zeit auch rückwärts fließen?

 
 

 
Kann Zeit auch rückwärts fließen?

 
 
Wer diese Frage beantworten möchte, muss sich zunächst mal auf eine ganz konkrete Definition von Zeit festlegen.
 
Da Relativitätstheorie keinerlei Quantenfluktuation berücksichtigt — sie aber allgegenwärtig ist, und das sogar noch auf unbegrenzt kleiner Zeit- und Ortsskala — kann die uns tatsächlich verstreichende Zeit nicht wirklich die der Relativititätstheorie sein.
 
Unter Physikern scheint Einigkeit darüber zu bestehen, dass man die Zeit am ehesten noch als den thermodynamischen Zeitpfeil sehen sollte.
 
 
Dann aber ergibt sich aus der wirklich präzisen Formulierung des 2. Hauptsatzes der Thermodynamik ganz klar:
     
  • Fast alle — aber nicht wirklich alle — Zeitschritte führen in die Zukunft.
     
  • Je größer ein Zeitschritt in die Vergangenheit ist, desto unwahrscheinlicher ist sein Auftreten (und die Wahrscheinlichkeit seines Auftretens fällt drastisch, je weiter zurück in die Vergangenheit er führt).

Diese Tatsachen führen auf Hawkings Vermutung zum Schutz der Zeitordnung ( Choronology Protection Conjecture, 1992 ). Sie besagt:
 
    In ihrem Zusammenwirken verhindern die Naturgesetze, dass makroskopische Körper Information in die Vergangenheit tragen können.
     
    Die Quantentheorie erlaubt zwar, dass einzelne Quanten sich auf mikroskopischer Skala auch rückwärts in der Zeit bewegen können, aber die Wahrscheinlichkeit daraus entstehender winziger Zeitschleifen ist extrem gering — um viele Größenordnungen geringer, als dass sie in unserem Alltag beobachtbare Konsequenzen haben könnte.

 
Kip S. Thorne — einer der intimsten Kenner der Allgemeinen Relativitätstheorie und zugleich einer der wenigen Physiker, die selbst ernsthaft über die Möglichkeit von Zeitreisen geforscht haben — will Hawking da nicht widersprechen.
 
 
Lies auch: Time and the Multiverse (2011).

 
 
HINWEIS: Die Zeit als den thermodynamischen Zeitpfeil, d.h. als Zunahme von Entropie zu definieren, ist nicht ganz unproblematisch. Entropie nämlich — so schreibt Hohnerkamp — quantifiziert uns fehlendes Wissen über den Mikrozustand des betrachteten Systems. Doch aus Sicht welchen Betrachters?
 
Will man sie eindeutig machen, müsste man das tun über die Formel
 
Entropie = Wärme dividiert durch Temperatur.

Zeit in solchem Sinne ist dann auf jeden Fall relativ.

 

 Beitrag 0-307
Warum unteilbare Materieteilchen keine feste Richtung der Zeit kennen.

 
 

 
Je komplexer ein Objekt,

desto unmöglicher wird ihm Umkehr seines Zeitpfeils

 
 
Jedes aus Materie bestehende Objekt ist — ständig gegebener Quantenfluktuation wegen — ständigem Zustandswechsel unterworfen. Mit der Komplexität eines Objekts steigt drastisch
     
  • die Zahl der ihm möglichen Zustände,
     
  • und damit einhergehend auch die Wahrscheinlichkeit, dass jeder Zustand, in den es gerät, höhere Entropie hat als sämtliche Zustände, in denen es sich schon einmal befunden hat.

Wir sehen also: Die Tatsache, dass in der Welt unserer Alltagserfahrung der Fluss der Zeit nur eine Richtung kennt, ist darauf zurückzuführen, dass sämtliche materiellen Gegenstände, mit denen wir in unserem Alltag zu tun haben, quantenmechanisch gesehen eine überaus große Zahl möglicher Zustande haben.
 
Ganz anders gilt für die einfachsten aller existierenden Objekte:
 
Ein einzelnes Elektron etwa altert nicht, denn die Zahl seiner möglichen Zustände ist sehr klein, so dass die Wahrscheinlichkeit dafür, dass sie sich ständig wiederholen, sehr groß ist.
 
Zeitumkehr auf der Ebene elementarer Teilchen ist daher etwas ganz Natürliches, recht Häufiges — erst für zunehmend komplexere Objekte wird es zunehmend unwahrscheinlicher, dass sie in einen einmal aufgegebenen Zustand später rein zufällig nochmals annehmen.

 
 
Quelle: Josef Hohnerkamp: Wissenschaft und Weltbilder, Springer Spektrum 2014, S. 197.


 

 Beitrag 0-199
Zeitmaschinen kann es nicht geben

 
 

 
Was Physik über Zeitreisen und Zeitmaschinen sagt

 
 
Einer der ganz wenigen Physiker, die sich jemals ernsthaft die Frage gestellt haben, ob Zeitmaschinen möglich sein könnten, ist Kip S. Thorne.
Andere – wie etwa sein Schüler Richard H. Price – begannen deswegen an seinem Verstand zu zweifeln (so berichtet Thorne selbst).
 
 
Ergebnis seiner Überlegungen:

     
  • Wurmlöcher und Zeitreisen werden heute von den meisten Physikern abgelehnt, obgleich die Gesetze von Einsteins Allgemeiner Relativitätstheorie ihre Existenz nicht ausschließen können oder — wie nur manche glauben — sogar nahelegen.
     
    Es gibt jedoch Erkenntnisse, die vermuten lassen, dass quantenphysikalische Gesetze Wurmlöcher und Zeitreisen unmöglich machen.
     
    Thorne schreibt wörtlich: "Erst wenn es gelingt, diese Gesetze besser zu verstehen, werden wir  v i e l l e i c h t  erfahren, wie die physikalischen Gesetze das Universum vor Wurmlöchern und Zeitmaschinen — mindestens aber vor Zeitmaschinen — bewahren."
     
     
  • Hawking glaubt, dass die Gesetze der Physik keine Zeitmaschinen zulassen: Quantenfluktuation würde dafür sorgen, dass die Zeitrichtung gewahrt bleibt.
     
    Thorne schreibt: "Es macht mir Spaß, mit Hawking zu wetten — aber nur dann, wenn ich eine reelle Chance sehe zu gewinnen." Er sieht sie nicht, denn Berechnungen, die er zusammen mit zwei seiner Studenten angestellt hat, lassen ihn vermuten, dass Hawking wahr­scheinlich recht hat. Endgültig wird man
    es erst wissen, wenn eine voll ausgearbeitete Theorie der Quantengravitation die Frage entscheidet.

 
Quelle: Kip S. Thorne: Gekrümmter Raum und verbogene Zeit, 1994, S. 592 und 596

 
 
Als blanken Unsinn empfinde ich, was Heinrich Päs in einem Interview geantwortet haben soll, als er danach gefragt wurde, ob denn nicht schon das sog. Großvater­paradoxon die Möglichkeit von Zeitreisen widerlege. Er soll gesagt haben:
    "Nein, denn es gibt Ideen, wie man das lösen könnte. ... Eine ... Möglichkeit existiert im Rahmen der Viele-Welten-Interpretation der Quantenphysik. In der Quantenphysik ist es ja häufig so, dass verschiedene Dinge gleichzeitig existieren können. Ein Teilchen kann sich an verschiedenen Orten gleichzeitig befinden. In dieser Viele-Welten-Interpretation gilt das nicht nur für Teilchen, sondern auch im Großen. Dann spalten sich diese zwei Realitäten in parallele Universen. Die Lösung der Zeitreise wäre dann, dass der Zeitreisende seinen Großvater in einem Parallel-Universum ermordet. Aber in dem Universum, wo er aufgebrochen ist, der Großvater nicht ermordet wird und insofern kein direkter Widerspruch entsteht."

Dass nicht nur Heinrich Päs, sondern auch andere Physiker, Michio Kaku etwa, heute immer noch so tun, als wäre Hugh Everetts Viele-Welten-Theorie ernst zu nehmen, ist eine Kuriosität für sich, zeigt aber wohl, dass selbst Hochschullehrer für Theoretische Physik i.A. wirklich nur ihr eigenes Spezialgebiet gut kennen (oder der Versuchung nicht widerstehen können, Laien mit möglichst spektakulären Aussagen zu beeindrucken).
 
Kurios auch: David Deutsch geht in Kap. 2 seines Buches The Fabric of Reality (1997) sogar so weit, die Interferenzmuster hinterm Doppelspalt über Hugh Everetts Viele-Welten-Theorie zu erklären. Auf Seite 51 behauptet er dann: "As I have just said, we do not need deep theories to tell us that parallel universes exist — single-particle interference phenomena tell us that." Er übersieht dabei, dass sein auf Seite 44-45 gegebener Erklärungsversuch der Ergebnisse eines erweiterten Doppelspalt­experiments die Existenz der Parallelwelten ja schon voraussetzt, sie also  n i c h t  beweisen kann.

 

  Beitrag 2112-3
Zur — durchaus begrenzten — Möglichkeit von Zeitreisen

 
 

Zur — durchaus begrenzten — Möglichkeit von Zeitreisen



Kakashi in 2112-1:
Hallo.

Ich habe mir paar Sendungen über Astrophysik angeschaut und es wurde dort beschrieben das Zeitreisen theoretisch möglich wären.
In die Vergangenheit zu reisen wäre eher schwieriger bzw. paradox.


Hi Kakashi,

es gibt bis heute nur zwei Wissenschaftler, denen es nachweislich gelang, unser Wissen über die Zeit — über das, was sie erlaubt bzw. nicht erlaubt — zu erweitern: Albert Einstein und Kurt Gödel.

Beide waren, vor allem gegen Ende ihres Lebens, enge Freunde, und Gödel hat zu Einsteins 70. Geburtstag eine ganz erstaunliche Eigenschaft der Zeit entdeckt: Der Allgemeinen Relativitätstheorie entsprechend — sie gilt heute als unser mit Abstand genauestes Modell der Raumzeit — kennt tatsächlich Raumstruktur existieren, die, wo sie vorliegt, Zeitreisen in die Vergangenheit zulässt.

Auch könnte solche Struktur, wo sie heute nicht vorliegt, sich im Laufe der Zeit bilden.

Auf Seite 137 seines Buches » Gödel, Einstein und die Folgen - Vermächtnis einer ungewöhnlichen Freundschaft, Verlag C.H. Beck 2005 « fasst Palle Yourgrau Gödels Erkenntnis zusammen wie folgt:

Zitat:
 
Eine Reise entlang der geschlossenen ... zeitähnlichen Weltlinien, die Gödel in dem, was gemeinhin unter dem Namen Gödel-Universum bekannt ist, entdeckt hat, lässt sich nur als Zeitreise beschreiben. Gödel hat auf verblüffende Weise demonstriert, dass Zeitreisen im striktesten Sinne mit der Relativitätstheorie vereinbar sind.

Die Fans des Zeitreisens waren von dieser Entdeckung natürlich begeistert, übersahen bei alledem jedoch, dass das Hauptergebnis eine überaus gewichtige Aussage war: Dann, wenn Zeitreisen möglich sind, ist Zeit selbst [im Sinne dessen, wie alle Menschen sie erleben und begreifen] unmöglich.

Wenn diese [Gödels] Überlegung und ihre Deutung näherer Überprüfung standhalten, dann hätte Gödel das Kunstück fertiggebracht, mit mathematischen Methoden eine Erkenntnis über die Realität (bzw. die Nicht-Realität) von Zeit zu gewinnen, die sämtlichen idealistischen Philosophen von Platon bis Kant Jahrhunderte hindurch entgagngen war ...

Vor Einsteins Augen hatte eine Metamorphose stattgefunden. Die Theorie, die er entworfen hatte, um Zeit zu erfassen, mathematisch festzuschreiben und dem menschlichen Verstehen zugänglich zu machen, war unter Gödels Händen einem Taschenspielertrick zum Opfer gefallen.


Bisher hat niemand — auch Einstein nicht — in Gödels Argumentation Fehler finden können.

Zeitreisen in die Vergangenheit im Sinne der durch Gödel entdeckten Konsequenzen von Einsteins Gravitationstheorie (allgemeiner Relativitätstheorie) führen gleichzeitig auch in die Zukunft und sind nur in Raumregionen möglich, in denen der Raum in ganz bestimmter Weise extrem gekrümmt ist. Solche Krümmung aber liegt — nach allem, was bis bis heute wissen —, im durch Menschen beobachtbaren Teil des Universums (einer kugelförmigen Umgebung um uns herum, deren Radius grob 42 Milliarden Lichjahre beträgt) NICHT vor.

Daraus folgt:
  • Man muss auch heute noch davon ausgehen, dass Zeitreisen in die Vergangenheit dem Menschen NICHT möglich sind.
  • Auch in unsere eigene Zukunft kann uns nur unsere eigene, ganz gewöhnliche Alterung führen.
  • Prinzipiell möglich aber sind jedem Menschen Reisen in selbst noch die fernste Zukunft von Objekten, die NICHT Teil seiner selbst sind (genauer: von Objekten, die völlig anderer Beschleunigung unterliegen können als er selbst). Physiker nennen dies das Zwillingsparadoxon und konnten es durch Experimente mit unterschiedlich beschleunigten Atomuhren oder auch Myonen schon mehrfach gut nachvollziehbar bestätigen.

Nebenbei: Nichts, was sich durch den Raum bewegt (das Licht ausgenommen) kann sich ebenso schnell oder gar noch schneller als das Licht bewegen.


Gruß, grtgrt
 

  Beitrag 2108-3
Wo Zukunft — wie manche glauben — auch Vergangenheit sein kann

 
 
C... in 2108-2:
 
Mit einer Zeitmaschine kannst du immer nur in die Zukunft, nie jedoch in die Vergangenheit reisen.


In Gödel-Universen (den R-Universen) gilt das, wie viele glauben, nicht:

Dort nämlich gibt es in sich geschlossene Wege durch die Raumzeit, und auf denen — so könnte man denken — Vergangenheit stets auch Zukunft ist (und umgekehrt).



Vorsicht aber:
    Gödel selbst war der Ansicht, dass die Zeit im Sinne der Relativitätstheorie auf keinen Fall die durch uns gefühlte sein könne.

    In seinem Buch Gödel meets Einstein — Time Travel in the Gödel Universe argumentiert Palle Yourgrau — der in Gödels Nachlass auch ein philosophisch orientiertes Papier zu diesem Thema fand —, dass es in jeder geschlossenen, zeitartigen Kurve einer durch die Relativitätstheorie beschriebenen Raumzeit weder Vergangenheit noch Zukunft gibt, sondern nur Gegenwart (die Gegenwart dort also nicht vergehen kann und sich über die ganze Kurve erstreckt):

    Sich auf einer solchen Kurve zu "bewegen" bedeute eben nicht, sich zeitlich zu bewegen, sondern entspreche eher einer Betrachtung der auf ihr liegenden einzelnen Ereignisse wie sie sich dort hintereinander aufgereiht finden, d.h. bedingen und — da die Kurve geschlossen ist — sogar gegenseitig bedingen.

    Auch nur eines dieser Ereignisse mehrfach zu   e r l e b e n   sei   n i c h t   möglich.


    Damit gilt dann aber auch:


    Zeitreisen im dem Sinne, dass man gewisse Zeitabschnitte mehrfach durchlebt, kann es auch in Gödel-Universen   n i c h t   geben.


 

  Beitrag 2112-10
Auch wer die Vergangenheit aufsuchen könnte, würde sie NICHT abändern können

 
 


In die Vergangenheit zu reisen  ( wo das überhaupt möglich sein sollte )

bedeutet  N I C H T , sie abändern zu können



Die meisten Menschen — auch Physiker, wie etwa Harald Lesch — argumentieren so, als würde die Möglichkeit, eine Zeitreise in die Vergangenheit zu unternehmen (wenn das denn möglich wäre) uns die Möglichkeit eröffnen, jene Vergangenheit abzuändern.

Nun hat Kurt Gödel zwar bewiesen, dass die Allgemeine Relativitätstheorie durchaus Situationen kennt, in denen, eine Reise in die Zukunft auch eine Reise in die Vergangenheit ist. Abänderbar ist jene Vergangenheit aber selbst dann NICHT.

Ich weiß nicht, ob Lesch das nicht bewusst war oder ob er in seiner Sendung zu Zeitreisen nur deswegen nicht darauf zu sprechen kam, weil er sie einfach und verständlich halten wollte. Auf jeden Fall scheint mir die Begründung für die Unmöglichkeit von Zeitreisen, die er gegen Ende seiner Sendung gibt, völlig unlogisch (er gibt, genau genommen, sogar zwei Begründungen — aber keine ist logisch unangreifbar).


Nebenbei noch:

In unserem Universum kann man die Vergangenheit sicher nicht aufsuchen (so argumentiert Steven Hawking).

In sog. Gödel-Universen allerdings ist das anders: nur deswegen allerdings, weil dort Zukunft stets auch Vergangenheit ist (und umgekehrt).
 

  Beitrag 2112-14
Ereignisse können sukzessive konstruiert oder als Gesamtheit einzig mögliche Lösung einer Randbedingung sein

 
 

Ereignisse können sukzessive konstruiert sein

oder als ( in ihrer Gesamtheit ) einzig mögliche Lösung gewisser Randbedingungen



Bauhof in 2112-13:
 
Hallo Grtgrt,

Gödels Ergebnis ist unphysikalisch, aber es zeigt auf, dass die Kausalität nicht aus der ART herleitbar ist. Hendrik van Hees begründet das hier wie folgt:

Zitat:
Das Gödel-Universum hat keine große physikalische Bedeutung, da es aufgrund der Kausalitätsverletzungen unserem Empfinden der Realität entgegenläuft. Das Gödel-Universum lässt jedoch folgendes erkennen: Die Kausalität wird nicht bereits durch die Einstein'schen Feldgleichungen bedingt, da ja die Gödel-Metrik eine Lösung dieser Gleichungen ist.

Das einzige Fazit, das aus Gödels Resultat gezogen werden kann, ist folgendes: Man muss nach den Gründen der Kausalität weiter suchen, weil die ART darüber nichts hergibt. Alle anderen Aussagen über Gödels Resultat sind Science Fiction.

M.f.G. Eugen Bauhof


Hallo Eugen,

ich weiß nicht, ob Du da recht hast. Es gibt nämlich Kausalität unsymmetrischen Typs und Kausalität symmetrischen Typs:
  • Unsymmetrisch: Ereignis A bestimmt die Randbedingung unter der Ereignis B passiert (oder auch nur bestimmte Ausprägung annimmt). B kann A nicht prägen.
  • Symmetrisch: Ereignis A bestimmt Randbedingungen für B, und auch umgekehrt. Sie prägen einander in gleichberechtigter Weise. Die Verallgemeinerung auf mehr als nur 2 Ereignisse, die die jeweils anderen mit prägen, ist offensichtlich.

Du argumentierst so, als gäbe es nur den unsymmetrischen Fall.
Gödels Beispiel aber repräsentiert den symmetrischen.

Nebenbei: In endlichen grenzenlosen Welten gibt es den unsymmetrischen Fall überhaupt nicht.


Gruß, grtgrt

PS: Ich würde Gödels Beispiel auch nicht als unphysikalisch bezeichnen — aber sehr wohl als  r e a l i t ä t s f e r n  (da der durch uns beobachtbare Teil des Universums bisher ja nirgendwo derart ausgefallene Krümmung zeigt).

U n p h y s i k a l i s c h  kann es schon allein deswegen nicht sein, da die Physik die Wirklichkeit zu modellieren versucht und die ART dasjenige ihrer Raumzeit-Modelle ist, das besser passt als alle andern uns bekannten. Gödels Beispiel wiederum ist Teil dieses Modells.
 

  Beitrag 2112-24
Noch eine Klarstellung zu Zeitreisen

 
 
Bauhof in 2112-22:
Hans-m in 2112-21:
 
Unter echter Zeitreise verstehe ich eine Reise, bei der ich bewusst ein Ziel bestimmen und auch erreichen kann, und nicht nur durch eine verzögerte Alterung irgend wann dort ankomme. Das wäre in etwa das gleiche, als würde ich mich 1000 Jahre einfrieren lassen, um dann in ferner Zukunft, wieder zu erwachen.

Hallo Hans-m,

tut mir leid, aber da liegst du vollkommen falsch.
Der reisende Zwilling unterliegt keiner "verzögerten Alterung", sondern in seinem Inertialsystem läuft die Zeit langsamer ab relativ zum Zeitablauf des daheim gebliebenen Bruders. Der reisende Zwilling sieht sich selbst während seiner Reise genau so schnell altern wie sonst auch.

Wenn der reisende Zwilling wieder bei seinem Bruder eintrifft, dann hat er eine Zeitreise in die Zukunft seines Bruders gemacht. Warum? Weil sein Bruder dann älter ist als er. Da kann auch bewusst ein Ziel bestimmt werden, indem der reisende Zwilling eine entsprechende Relativgeschwindigkeit wählt. Je höher die Relativgeschwindigkeit, um weiter reist er in die Zukunft seines Bruders.

M.f.G. Eugen Bauhof


Hallo Eugen,

so wie ich das sehe, habt ihr beide recht, denn:

Der Widerspruch, den Du zu sehen scheinst, wird nur suggeriert, weil Hans-m sich nicht genau genug ausgedrückt hat. Statt zu sagen


Hans-m in 2112-21:
 
Zeitdilatationen verstehe ich nicht als Zeitreisen. ( Siehe Zwillingsparadoxum )

Nur weil Zwilling 2 die Möglichkeit hat, 1000 Jahre älter zu werden als Zwilling 1,weil er entsprechend langsamer altert, so sehe ich darin keine Möglichkeit zur Zeitreise.


hätte er genauer sagen sollen:

Zitat von Grtgrt:
 
Zeitdilatationen verstehe ich nicht als Zeitreisen. ( Siehe Zwillingsparadoxum )

Nur weil Zwilling 2 aus Sicht seines Bruders die Möglichkeit hat, 1000 Jahre älter zu werden, (da 2 aus der Sicht seines Bruders langsamer altert), sehe ich darin keine Möglichkeit zur Zeitreise.


Natürlich gilt dieses Argument ( ebenso wie deines ) nur im Rahmen der SRT — bei gleichförmiger Bewegung also.

Vermutlich aber hatte Hans-m den allgemeinen Fall im Sinn — für den aber macht seine Aussage durchaus Sinn, denn der reisende Bruder wird nach seiner Rückkehr zur Erde, ja wirklich älter sein als der daheim gebliebene. Beide werden das übereinstimmend so feststellen.

Kurz: Man sollte nie vergessen, dass Zeitdilation im Kontext von Beschleunigung ein ganz anderer Effekt ist als Zeitdilatation im Sinne der SRT (siehe Beitrag 0-144).

Gruß, grtgrt
 

  Beitrag 2107-14
Was Wege allein durch den Raum von allgemeineren Wegen durch die Raumzeit unterscheidet

 
 
Hallo C...,

Hier will ich jetzt mal meine Vermutung verteidigen, dass Wege, die nicht nur durch den Raum sondern auch durch die Zeit führen, von grundsätzlich anderer Qualität sind als solche, die  n u r  durch Raum führen:
  • Zunächst fällt auf, dass es massebehafteten Objekten gar nicht möglich ist, sich allein durch den Raum (aber nicht auch gleichzeitig durch die Zeit) zu bewegen: Sie müssten sich ja sonst mit Lichtgeschwindigkeit durch den Raum bewegen, was sie nicht können.
    Der Grund hierfür: Wie die Minkowskimetrik zeigt, ist  j e d e  Bewegung durch die Raumzeit eine Bewegung mit Lichtgeschwindigkeit. Das ist uns, die wir 3-dimensional denken, nur nicht ständig bewusst: Was wir als Bewegung mit kleinerer Geschwindigkeit durch den Raum wahrnehmen, ist schließlich nur die Projektion dieser Gesamtgeschwindigkeit aus der Raumzeit heraus auf den Raum.
  • Desweiteren: Jeder Weg, der nicht nur durch den Raum führt, ist eine Einbahnstraße — kann also nur in einer Richtung gegangen werden (und das scheint selbst noch in R-Universen so zu sein). Ganz anders Wege durch den Raum: Jeder von ihnen ist Projektion mindestens eines Weges (meist aber sogar vieler Wege) durch die 4-dimensionale Raumzeit auf den 3-dimensionalen Raum — und nur deswegen, kann er in beiden Richtungen durchquert werden; aber genau deswegen handelt es sich auch um einen nur virtuellen Weg: Er existiert nur gedanklich und ist eine Art Schatten wirklicher Wege.

Wir sehen: Insbesondere das zweite Argument zeigt uns, dass
  • Wege durch die Raumzeit, die nicht nur durch den Raum führen, real existieren und Einbahnstraßen sind,
  • wohingegen es Wege allein durch den Raum ausschließlich für das Licht gibt (und sie keine Einbahnstraßen sind).

Projeziert auf den 3-dimensionalen Raum (durch vernachlässigen der Zeitkomponente sämtlicher Ereignisse) ist unsere Welt vergleichbar einem zusammengefalteten Fächer. Was sich dort als in beiden Richtungen befahrbarer Weg zwischen zwei Orten darstellt, wird — wenn man den Fächer entfaltet — erkennbar als Bild zahlreicher Einbahnstraßen unter jener Projektion.

Bitte auch beachten: In dieser Argumentation kann die Dimension t ihre Rolle mit keiner der Dimensionen x, y, z tauschen, denn nur x, y, z spannt den 3-dimensionalen Teilraum der Raumzeit auf, entlang dem sich das Licht bewegt.

Die zeitliche Dimension hat somit auch in der ART eine wesentlich andere Qualität als die der rein räumlichen Dimensionen.


Gruß, grtgrt
 

  Beitrag 2108-8
Wer sich nicht von sich selbst entfernen kann, wird immer altern ...

 
 
Wrentzsch in 2108-7:
 
Wo könnte man außerhalb des R-Universums auf die Wiederkehr des Ziel-Zustandes des Universums warten ohne selbst zu altern?


Auf diese Frage, lieber Wrentzsch, kenne ich keine Antwort.

Vielleicht aber interessiert dich: Wenn Du dich extrem nahe an den Ereignishorizont eines Schwarzen Lochs begeben könntest, würdest Du von dort aus in aller Ruhe den Untergang unseres Sonnensystems — und vielleicht auch das Entstehen eines neuen Sonnensystems und einer neuen Erde — beobachten können.

Aus Sicht der Erdbewohner — so lange es solche denn noch gibt — würdest Du praktisch gar nicht altern (wenigstens nicht so viel, dass es durch sie feststellbar wäre).

Aus deiner eigenen Sicht heraus aber würdest auch dort ebenso schnell alt werden wie hier auf unsere schönen Erde ...


Was für ein Pech also, dass ein Enfernen von uns selbst nur per Schizophrenie möglich ist ... aber immerhin!


 

  Beitrag 2110-1
Das Rätsel der Zeit — wer kann es lösen?

 
 
Wie Einsteins allgemeine Relativitätstheorie vorhersagt — und wie selbst noch im Labor sehr einfach durchführbare Experimente mit unterschiedlich stark beschleunigten Myonen klar bestätigen —, korrespondiert der zunächst nur als  R a u m d i m e n s i o n  zu verstehende Zeitbegriff der ART mit biologischer  A l t e r u n g .

Das Rätsel der Zeit besteht darin, dass wir nicht wissen, warum das so ist.

Wer kann sich auch nur  e i n e n  plausiblen Grund dafür vorstellen?

 

  Beitrag 2110-3
Die Welt um uns herum ist nur eine Welt gedanklicher Modelle

 
 
Horst in 2110-2:
 
Grtgrt im Beitrag 2094-1
Zitat:
Raum, Zeit und Raumzeit existieren nur als rein gedankliche Konstruktion!


Hallo Horst,

wie sich meinem Beitrag 2094-15 entnehmen lässt, haben Physiker (Niels Bohr etwa), aber auch Philosophen wie Kant — und lange vor ihm schon Parmenides — immer wieder versucht uns klar zu machen, dass die Welt um uns herum nur die Welt unserer Sinneswahrnehmungen ist — eine Welt gedanklicher Modelle also, die wir uns von bestimmten Teilen der Natur machen.

Raum, Zeit und Raumzeit sind da keine Ausnahme. Gleiches gilt für das, was Einsteins Feldgleichungen der ART uns als räumliche Zeit einerseits und als biologische Zeit (Alterung) andererseits definieren und präsentieren.

Rätselhafte Beziehungen kann es halt auch zwischen rein nur gedanklichen Konstruktionen geben!

Gruß, grtgrt
 

  Beitrag 2110-9
-

 
 
Horst in 2110-7:
 
Henry in 2110-5:
 
Eine Reaktion hat auch eine "Dauer", nämlich eine bestimmbare Zeitspanne,

Hallo Henry

Das heißt für mich, die "Dauer" einer Reaktion (Bewegung, Veränderung) hat objektiv mit der herkömmlichen Vorstellung von Zeit nichts zu tun. Das bestimmen einer Zeitspanne ist ein subjektiver Vorgang, der willkürlich die Dauer einer bestimmten Bewegung zur Norm erhebt und mit Maßeinheiten versieht.

Eine bestimmte "Zeitspanne" ergibt sich doch erst durch Messung und ist mit Maßeinheiten versehen, damit also ein reines, subjektives Messergebnis und weiter nichts.


Hallo Horst,

wer unterschiedlich stark beschleunigte Myonen betrachtet, aus dessen Sicht haben die stärker beschleunigten längere Lebensdauer (genauer: längere Halbwertszeit, d.h. längere durchschnittliche Lebensdauer).

Und das ist keineswegs nur ein subjektiver Eindruck, sondern ein wirklich  o b j e k t i v e r , mit Hilfe der ART  n a c h r e c h e n b a r e r .

Zeit ist also für jedes Objekt tatsächlich am ehesten noch das, was es von seiner Uhr abliest: Zeit im biologischen Sinne.


Subjektiv werden Zeitspannen erst dadurch, dass die Uhren verschiedener Beobachter verschieden schnell gehen.

Mit anderen Worten: Nicht die Zeit (bzw. Zeitspanne) ist subjektiv, sondern subjektiv sind die in den Beobachtern entstehenden Bilder davon.

Diese Tatsache ist Spezialfall des allgemeinen Gesetzes:


Jeder von uns kennt jeden Teil der Natur nur als das Modell, das er sich davon macht.


Gruß, grtgrt
 

  Beitrag 2110-10
Kausalität hat wohl nur scheinbar etwas mit Zeit zu tun

 
 
Henry in 2110-5:
 
... der " naturwissenschaftliche Zeitbegriff " beinhaltet eindeutig die Kausalität.


Gödels Entdeckung von Zeitschleifen in rotierenden Universen scheint dem zu widersprechen.


 

  Beitrag 2110-19
Wo Wissenschaft sich entwickelt, wird auch die Begriffswelt sich entwickeln müssen

 
 
Henry in 2110-15:
 
Der im wissenschaftlichen Zusammenhang immer wieder erwähnte "Beobachter" ist kein Individuum, sondern soll nur dafür stehen, dass bestimmte physikalische Größen vom Bewegungszustand eines Systems abhängen, also z. B. die Dauer von Prozessen. Das hat gar nichts damit zu tun, dass sie tatsächlich gemessen werden, ein Myon "altert" auch dann langsamer, wenn man diesen Alterungsprozess nicht misst.Der Beobachter steht dafür, dass man etwas messen KANN, nicht dafür, dass die Ergebnisse der Messungen vom Beobachter abhängen.


Ja, Henry,

das ist mir völlig klar, aber du sagst ja selbst (sogar noch in diesem Zitat), dass so ein "Beobachter" im wissenschaftlichen Zusammenhang immer wieder "erwähnt" wird — warum also sollte nicht auch ich das so halten?

Kurz: Für jemand, der über Relativitätstheorie diskutiert, hat das Wort dann halt eine ganz spezifische Bedeutung — eine, die sich vom Alltagsgebrauch des Wortes unterscheidet.

Hier also geht deine Kritik völlig ins Leere.


Was allerdings meinen Begriff biologischer Zeit betrifft, kann ich dich eher verstehen. Auch mir gefällt er nicht wirklich, es war aber der beste, der mir einfiel, um mich stets daran zu erinnern, dass die Zeit, die man von Uhren abliest, nicht nur gefühlsmäßig, sondern wirklich durch Experimente nachweisbar ein Altern des beobachteten Objekts quantifiziert (was sich bei Myonen in deren mittlerer Zerfallszeit widerspiegelt).

Berücksichtigt man, dass diese Quantifizierung nur statistisch richtige Aussagen macht, kann man sich sehr gut vorstellen, dass man für Objekte, die aus Milliarden mal Milliarden Elementarteilchen bestehen — wie etwa Lebewesen — dann halt nur noch zu Aussagen kommt wie

» ein Mensch wird typischerweise maximal 100 Jahre alt « .


Und so reicht die Zeit der Uhren eben halt doch schon wirklich auch in die Biologie hinein.


Henry in 2110-15:
 
Weshalb kannst du einen Begriff nicht schlicht und einfach so verwenden, wie er üblicher Weise und somit direkt verständlich verwendet wird?


Aus demselben Grund, aus dem auch Physiker den Begriff des "Beobachters" selbst dort noch verwenden, wo gar kein Mensch damit gemeint ist.

Es ist halt einfach so, dass man in der Wissenschaft präzise Begriffe braucht, hierfür aber nicht ständig neue Worte erfinden kann — und auch gar nicht immer erfinden will, da es ja häufig darum geht, schon durch die Wortwahl auf existierende Analogien hinzuweisen. Man nimmt dann also ein Wort, das die richtige Analogie suggeriert und ergänzt: Wir verwenden es jetzt im folgenden präzisierten Sinne ...


Nebenbei: Dass die Semantik gebrauchter Worte kontextfrei sein kann, ist ein weit verbreiteter Irrtum.

Gruß, grtgrt
 

 Beitrag 0-546
2022: Die bisher genaueste Messung gravitativer Zeitdilatation

 
 

 
Gravitative Zeitdilatation

und die 2022 präziseste Zeitmessung menschlicher Geschichte

 
 
Auch die Stärke von Gravitationspotential hat Einfluss auf den Verlauf der Zeit: In unmittelbarer Nähe eines massereichen Körpers gehen Uhren langsamer als in gewissem Abstand davon. Physiker konnten diese Vorhersage der allgemeinen Relativitätstheorie in den vergangenen Jahrzehnten mehrfach experimentell bestätigen, indem sie Präzisionsuhren mit Flugzeugen oder Raketen auf große Geschwindigkeiten oder Höhen brachten oder ihre Experimente auf Berge oder Türme verlegten.
 
 
Nun haben Forscher vom National Institute of Standards and Technology in Boulder (Colorado) den Effekt solch gravitativen Zeitdehnung (Zeitdilatation) dem bislang präzisesten Test unterzogen. Sie haben den Gangunterschied zweier horizontal über einander angeordneter Uhren auf der Millimeterskala verglichen.
 
 
 

 
Quelle: Einstein hat recht, sogar millimetergenau, FAZ vom 28.2.2022

 
 
Diese so extrem präzise Messung vorzunehmen verwendeten die Physiker in Boulder für ihre Messungen die derzeit beste optische Atomuhr. Sie geht noch einmal um einen Faktor fünfzig genauer als eine Cäsiumuhr. Rund hunderttausend tiefgekühlte Strontiumatome werden hier mit sich kreuzenden Laserstrahlen in einer optischen Gitterstruktur in der Schwebe gehalten und mit einem extrem stabilen roten Laserstrahl angeregt. Schwingen die Atome und das externe Laserfeld im Takt, wird die Frequenz des von den Atomen ausgesandten Lichts und damit die Taktrate der Strontium-Atomuhr gemessen. Da man viele tausend synchron schlagende Taktgeber vorliegen hat, erreicht man eine deutlich größere Messgenauigkeit, als wenn man mit einem einzelnen Ion arbeiten würde.
 
Mögliche Anwendungen für derart genaue Messung sehen die Forscher z.B. in der Geodäsie. Die präzisen Chronometer könnten dazu genutzt werden, genaue Höhenprofile von Bergen zu erstellen oder die Tiefe der Ozeane präzise auszuloten. Sogar für die Erdbebenvorhersage ließen sich die genauen Strontium-Atomuhren einsetzen.
 
 
|
 
 
Diese neue Messung verbessert deutlich den bisherigen Rekord aus 2010, auf den in (2010) hingewiesen wird wie folgt:
 
"According to general relativity, atomic clocks at different elevations in a gravitational field tick at different rates. The frequency of the atoms' radiation is reduced—shifted toward the red end of the electromagnetic spectrum—when observed in stronger gravity, closer to Earth. That is, a clock ticks more slowly at lower elevations. This effect has been demonstrated repeatedly; for example, NIST physicists measured it in 2010 by comparing two independent atomic clocks, one positioned 33 centimeters (about 1 foot) above the other".

 

 Beitrag 0-490
Was Physiker über die Natur der Zeit zu sagen wissen

 
 

 
Ursprung, Wesen und Struktur der Zeit

nach Carlo Rovelli

 
 
Der theoretische Physiker Carlo Rovelli sagt über die Zeit Folgendes:


Rovelli (2018):
 
Wir sind ausgegangen vom vertrauten Bild der Zeit, nach dem Gegenwart — ein Jetzt — existiert, in dem die Zeit gleichmäßig und in nur eine Richtung zu fließen scheint. Vergangenes ist fix, die Zukunft offen und unbestimmt.
 
Dies, so dachten wir, sei die Grundstruktur der Welt.
 
Bei genauerem Hinsehen zerbröckelt dieses einfache Bild: Die Realität — insbesondere, wenn man sie nicht nur lokal betrachtet — erweist sich als deutlich komplexer:
     
  • Eine dem gesamten Universum gemeinsame Gegenwart gibt es nicht, denn Gleichzeitigkeit ist relativ und Kausalität eine nur teilweise Ordnung (keine lineare) auf der Menge aller die Raumzeit ausmachenden Ereignisse.
     
  • Die elementaren Gleichungen, alles Geschehen physikalisch zu beschreiben, kennen keinen Unterschied zwischen Vergangenheit und Zukunft. Er entspringt allein nur der Tatsache, dass unserem Eindruck nach, Vergangenes andere Qualität hat als Zukünftiges.
     
  • Schon in unserer nahen Umgebung vergeht die Zeit keineswegs überall gleich schnell:
       
    • Im Dachboden jeden Hauses vergeht sie schneller als im Keller
       
    • und relativ zu einander bewegte Objekte beobachten ganz grundsätzlich, dass die Uhr des jeweils anderen nicht synchron zur eigenen tickt.

     
  • Wenn man Quanteneffekte vernachlässigt, erscheinen uns Raum und Zeit als Aspekte einer gewaltigen, beweglichen Gelatine (der Raumzeit), in die wir eingebacken sind.
     
  • Schleifen-Quantengravitation zeigt, dass das nur eine näherungsweise Sicht zu sein scheint: Raum und Zeit sind gequantelt. Genauer:
     
  • In der elementaren Grammatik der Welt gibt es weder Raum noch Zeit, sondern nur prozesshaftes Geschehen, welches die Werte physikalischer Größen der sich ständig neue Form gebenden Verteilung von Energie anpasst.
     
    Wie Lee Smolin erklärt, sind hier zwei Regeln am Werk: Stetige Veränderung beschrieben durch Schrödingers Gleichung und unstetige, realisiert durch Quantenfluktuation (Kollaps — d.h. spontane Neudefinition — der Wellenfunktion).
     
  • Auf der wirklich grundlegenden Ebene also — soweit wir sie heute zu erahnen beginnen — gibt es demnach wenig, was unserer Erfahrungswelt ähnlich ist: Es gibt weder eine spezielle Variable » Zeit «, noch einen physikalishen Unterschied zwischen Vergangenheit und Zukunft, und sogar die Raumzeit zerbröckelt. So wie Nebel beim genauen Hinsehen als Wassertröpfen besteht, besteht Raumzeit aus Ereignissen:
     
     
     
    Es gibt keine » statische « Welt, die unveränderliches » Blockuniversum « wäre.
     
    Das Gegenteil trifft zu:

     
    Die Welt ist Geschehen — keine Menge von Dingen.

 
Es gibt keine Zeit, welche alle Geschehnisse linear anordnet — schon gar nicht weiträumig gesehen.

 


 
Soweit Rovellis eigene Zusammenfassung der Kapitel 1 bis 7 seines Buches. In Kapitel 8 bis 12 erklärt er dann, wie es zu unserem Zeitempfinden aus der makroskopischen Sicht unseres Alltags kommt:


Rovelli (2018):
 
Die Überraschung — so Rovelli — bestehe darin, dass wir selbst uns dieses grobe, allzu einfache Bild der Zeit schaffen:
 
Aus unserer Perspektive sehen wir ein in der Zeit ablaufendes Weltgeschehen, welches wir aber nur unscharf wahrnehmen, da unsere Interaktion mit anderen Teilen der Welt nur allzu partiell ist und sein kann.
 
Die Unbestimmtheit der Quanten setzt der Schärfe unseres Bildes sogar ganz prinzipiell eine Grenze.
 
Die sich hieraus ergebende Unkenntnis führt zum Begriff der » thermodynamischen Zeit « sowie der » Entropie «. Letztere quantifiziert unser fehlendes Wissen über Mikrozustände.
 
Die Entropie der Welt — bezogen auf uns — nimmt ständig nur zu, und nur deswegen kennt unsere Zeit [ als die thermodynamische ] nur eine Richtung.
 
Somit können wir am Ende anstatt von vielen möglichen Zeiten nur von einer einzigen reden: von derjenigen unserer Erfahrung. Nur sie ist universell, gleichförmig und von nur einer Richtung. Sie ist Näherung einer Näherung einer Näherung einer korrekten Beschreibung der Welt, geschaffen aus der besonderen Perspektive von uns Kreaturen, die wir uns am Wachstum der Entropie nähren, in dem wir das Fortschreiten dieser Zeit verankert sehen.
 
Und so gilt für uns, wie es im Kobelet heißt, » eine Zeit zum Gebären und eine Zeit zum Sterben «.
 
 
Das also ist die Zeit für uns:
    ein geschichtetes, komplexes Konzept mit vielfältigen unterschiedlichen Eigenschaften, die sich aus unterschiedlich genauer Betrachtung — und somit aus unterschiedlicher Näherung — ergeben.

 
 
So also verstehe ich — Carlo Rovelli — die physikalische Struktur der Zeit, nachdem ich mich ein Leben lang mit ihr befasst habe.
 
Viele Aspekte meiner Sichtweise sind solide, andere plausibel und wieder andere gewagte Versuche, zu einem Verständnis des Gegenstandes zu kommen:
     
  • Durch zahllose Experimente abgesichert ist, wie Gravitation den Lauf der Zeit bremst, ihre Relativität, dass die Grundgleichungen der Physik keine Zeit benötigen, die Beziehung zwischen Entropie und der Richtung des Fließens der Zeit, und nicht zuletzt, dass zwischen Entropie und [quantenphysikalischer] Unbestimmtheit eine Beziehung besteht.
     
  • Dass das Gravitationsfeld Quanteneigenschaften aufweist, ist eine allgemein geteilte Überzeugung in der Physik — auch wenn bisher nur theoretische Argumente sie stützen.
     
  • Plausibel ist das Fehlen einer Zeit-Variablen in den Grundgleichungen [wie im zweiten Teil des Buches erörtert] — auch wenn um die Form der Gleichungen noch gerungen wird.
     
  • Der Ursprung der Zeit in der Nicht-Kommutativität der Quanten, die thermodynamische Zeit und dass die beobachtbare Zunahme der Entropie von unserer Interaktion mit dem Universum abhängt, sind Ideen, die faszinieren, aber alles andere als gesichert sind.
     
  • Tatsache jedenfalls ist, dass die Zeitstruktur unserer Welt vom naiven Bild, das wir uns von ihr machen, abweicht.

 
Viele Diskussionen über den Zeitbegriff sind nur deshalb konfus, da seine Vielschichtigkeit, sein wirklich komplizierter, erst durch Einstein aufgedeckter Aspekt, unberücksichtigt bleibt. Der Fehler besteht darin, zu übersehen, dass seine verschiedenen Schichten von einander uabhängig sind.
     
  • Das Geheimnis der Zeit beunruhigte Philosophen schonn immer:
       
    • Parmenides (etwa 500 v. Chr.) wollte der Zeit die Realität absprachen,
       
    • Platon (etwa 350 v. Chr.) ersann ein Reich der Ideen außerhalb der Zeit, und
       
    • Hegel (1770-1831) spricht vom Augenblick, in dem der Geist die Zeitlichkeit überwindet.
       
    • Mit Hans Reichenbach: The Direction of Time (1956) ist eines der scharfsinnigsten Werke über das Wesen der Zeit entstanden.
       
    • Der Menschen zutiefst emotionale Haltung der Zeit gegenüber hat eher zum Errichten philosophischer Kathedralen als zu einer Auseinandersetzung auf Basis von Logik und Vernunft beigetragen.
       
    • Die Physik aber hilft uns, Schicht um Schicht in das Geheimnis vorzudringen. Sie tut das, indem sie uns klar macht, wie weit sich die Zeitstruktur der Welt von der unserer intuitiven Vorstellung unterscheidet.

     
    Vielleicht ist ja unsere emotionale Haltung der Zeit gegenüber genau das, was für uns die Zeit ausmacht.
     
    Ich glaube nicht, dass es noch sehr viel mehr zu verstehen gibt. Genauer:
     
    Man kann sich weitere Fragen stellen, sollte aber darauf achten, dass sie sich gut formulieren lassen. Wo ein Problem nicht präzise formuliert werden kann, handelt es sich meist nur um ein Scheinproblem. Die Zeit haben wir gefunden, sobald wir auf alle ihre sagbaren Eigenschaften gestoßen sind.
     
     
     
     
    Carlo Rovelli zum Wesen der Zeit

     
     
    Quelle: Carlo Rovelli: Die Ordnung der Zeit, Rohwohlt 2018, S. 159-166


     

 Beitrag 0-39
Über das Jetzt und den Fluß der Zeit

 
 

 
Über das Jetzt und den Fluß der Zeit

 
 
Zeit, so könnte man es sehen, vergeht überall dort, wo die Wellenfunktion des Universums (im quantenmechanischen Sinne) sich neu konfiguriert, d.h. wo Möglich­keiten verworfen und oder zu Fakten werden.
 
Etwas als Faktum bzw. als lediglich eine von mehreren Möglichkeiten zu bewerten, bedarf es aber stets Informationsverarbeitung. Da nun aber Information nur mit endlicher Geschwindigkeit transportierbar ist — mit maximal Lichtgeschwindigkeit —, folgt daraus zwangsläufig, dass der Fluß der Zeit beobachterspezifisch sein muss.
 
Da Gegenwart die Menge aller Raumzeitpunkte ist, in denen sich die Wellenfunktion des Universums umkonfiguriert, lässt sich feststellen:
 
 
Gegenwart ist überall,
 
ist aber nicht dasselbe, was ein spezifischer Beobachter B als Gegenwart empfindet:

 
Seine Gegenwart G(B) und sein Jetzt J(B) sind mehr oder weniger verschieden von der jedes beliebigen anderen Beobachters.

 
 
Dies bestätigt H. Dieter Zeh, der sagt:

Zeh auf Seite 11-12 in The physical Basis of the Direction of Time (2001), 4th ed. Springer
 
Der Begriff des Jetzt scheint ebenso wenig mit dem Zeitbegriff selbst zu tun zu haben wie die Farbe mit dem Licht. ...
 
Sowohl Jetzt als auch Farbe sind bloß Aspekte dessen, wie wir Zeit oder Licht wahrnehmen. ...
 
Jedoch kann weder [der Farben] subjektive Erscheinung (wie "blau") noch die subjektive Erscheinung des Jetzt aus physikalischen oder physiologischen Ansätzen hergeleitet werden.
 


 
All das betrachtet, würde ich sagen:

Gebhard Greiter (2014)
 

Zeit ist das, was der Fluß der Zeit generiert: Veränderung und eintretende Gewissheit

 
Und natürlich ist Zeit in diesem Sinne ein rein beobachterspezifischer Begriff.
 
Dennoch ist Zeit Z(B) in diesem Sinne nicht einfach ein im Beobachter B entstehender Eindruck. Nein, Z(B) ist des Beobachters spezifische Sicht auf etwas,
  • das die Natur produziert,
  • jedem von uns aber nur aus sehr spezifischer Sicht heraus beobachtbar macht.

 
 
Note: Wenn ich hier von Gewissheit spreche, so ist damit nicht subjektive Gewissheit gemeint, sondern beobachterspezifisch gegebene objekte Gewissheit, d.h. die Summe aller Fakten, die der Beobachter kennen würde, wenn er in der Lage wäre, sämtliche Information, die die Natur ihm zustellt, auch wirklich zur Kenntnis zu nehmen (und korrekt zu interpretieren).
 
Man sieht hier, dass unsere Alltagserfahrung völlig zu Recht einen objektiven (wenn auch stets beobachterspezifischen) Zeitbegriff kennt, daneben aber auch nur gefühlte, völlig subjektive Zeitbegriffe, die sich ergeben, wenn man nur einen besonders kleinen Teil der objektiven Gewissheit zur Kenntnis nimmt.
 



 

 Beitrag 0-87
Was Physiker bislang über die Möglichkeit von Zeitreisen zu sagen wissen

 
 

 
Was Physiker heute (2011) insgesamt über die Möglichkeit von Zeitreisen zu sagen wissen

 
 
Anlässlich von Einsteins 70-ten Geburtstag machte Kurt Gödel (berühmter Logiker, enger Freund Einsteins) eine Entdeckung, die eigentlich wie eine Bombe hätte einschlagen müssen:
 
Gödel fand Lösungen von Einsteins Feldgleichungen, die zeigen, dass die Allgemeine Relativitätstheorie auch Welten zulässt, in denen die Zeitdimension zu einer geschlossenen Kurve gekrümmt ist, so dass, wenn Bewohner solcher Welten hinreichend lange leben, sie schließlich wieder den Zeitpunkt ihrer Geburt erreichen.
 
Das hypothetische Universum, welches durch Gödels Gleichungen beschrieben wurde, ist
  • weder statisch (wie Einsteins instabile Lösung mit Hilfe seiner kosmologischen Konstante),
     
  • noch expandierend (wie unser Universum)
Es ist stattdessen ein  r o t i e r e n d e s  Universum:
 
Rotation eines Universums führt dazu, dass sich Raum und Zeit vermischen: Je weiter man sich in einem solchen Universum vom Zentrum seiner Rotation entfernt, desto mehr ist der Doppelkegel gedreht, der dem Beobachter prinzipiell einsehbare Vergangenheit und Zukunft von den für ihn prinzipiell unerreichbaren Regionen der Raumzeit trennt. Ab einer bestimmten Entfernung von Zentrum der Rotation kann der Beobachter dann auf einer in zeitlicher Hinsicht geschlossener Kurve durch die Raumzeit reisen und so in seiner eigenen Vergangenheit ankommen.
 
Ob diese Lösungen von Einsteins Feldgleichungen aber tatsächlich existierende Welten beschreiben, ist eine noch völlig offene Frage.
 
Noch offen ist bisher insbesondere, ob es physikalische Gesetze gibt, die ein Reisen in die eigene Vergangenheit unmöglich machen.
 
Gödels Ergebnis könnte bedeuten, dass es die Zeit in Wirklichkeit gar nicht gibt.
 
 
Auf jeden Fall haben die Physiker Gödels Ergebnis zunächst überhaupt nicht beachtet, und tatsächlich spricht bislang nichts dafür, dass unsere Universum ein rotierendes sei.
 
Es dauerte fast 25 Jahre, bis das Thema Zeitreisen wieder Gegenstand wissenschaftliches Interesses wurde: Diesmal durch eine Arbeit, die Frank J. Tipler als Doktorand an der University of Maryland verfasste: Er studierte die Raumzeit um rotierende Zylinder herum, die erstmals 1936 von Jacob von Stockum berechnet worden war.
 
Tiplers Arbeit endete mit dem Satz
 
Kurz gesagt: Die Allgemeine Relativitätstheorie legt nahe,
 
dass man durch die Konstruktion eines hinreichend großen rotierenden Zylinders eine Zeitmaschine erschafft.

 
 
Wieder hätte man erwarten können, dass so ein Ergebnis die Gemeinschaft der Physiker in hellen Aufruhr versetzt. Dies aber war nicht der Fall. Bis 1985 ist Tiplers Arbeit nur ganze 3 Mal zitiert worden. Dann aber kam ein erster Durchbruch:
 
Kip Thorne — ein Professor für Physik am Caltech — war von Carl Sagan — einem Astronomen und Science-Fiction-Autor — gebeten worden, eine von Sagan geschriebene phantasievolle Geschichte, bei der es um Reisen durch den sog. Hyperraum ging, dahingehend zu prüfen, ob Sagon darin irgend etwas sage, das bekannten Gesetzen der Allgemeinen Relativitätstheorie widerspreche.
 
Bei dieser Gelegenheit erkannte Thorne, dass sog. Wurmlöcher — deren Existenz Einsteins Theorie voraussagt — abkürzende Wege durch die Raumzeit beschreiben mit dem Effekt, dass wer ein Wurmloch durchquert, von seinem Eingang zu seinem Ausgang weit schneller gelangt als Licht, das den üblichen Weg — einer Geodäte entlang — durch die Raumzeit nimmt. Der Grund hierfür: Im Wurmloch selbst, das man sich als eine Art Schlauch vorzustellen hat, ist der Raum so stark gestaucht, dass Anfang und Ende dieses "Schlauches" nur einen Katzensprung weit voneinander entfernt erscheinen.
 
Wie Kip Thorne und zwei seiner Studenten (Mike Morris und Ulvi Yurtsever) dann errechnet — und in der Fachzeitschrift Physical Reviews Letters auch publiziert — haben, ergibt sich daraus ein enfaches Rezept für eine Zeitreise: Der Reisende würde am Ziel ankommen, noch bevor er — aus wessen Sicht? — abgereist ist.
 
 
Vier Jahre später veröffentlichte auch Steven Hawking Überlegungen zu Zeitreisen mit Hilfe von Wurmlöchern. In einem Aufsatz mit dem Titel » Die Chronologie-Schutz-Vermutung « (veröffentlicht 1992 in der Fachzeitschrift Physical Review D « erklärte er: Es scheint, als ob es eine Chronologie-Schutz-Kommission gibt, die die Existenz geschlossener zeitartiger Kurven verbietet und so das Universum zu einem sicheren Platz für Historiker macht. [Es war dies eine Anspielung auf die sog. Timekeepers der Comics-Serie Marvel, die dort ebenfalls über die Zeit wachen und um sicherzustellen, dass das Universum gedeiht.]
 
Hawking also denkt, es müsse da etwas sein, das Reisen in die eigene Vergangenheit unmöglich macht.

 
Und es gibt tatsächlich wenigsten zwei Gründe für diese Vermutung:
  • Berechnet man nämlich, was für Materieverteilungen notwendig sind, die Geometrie eines Wurmloches zu erzeugen, so erkennt man: Jede solche Materievertelung muss negative Energie haben. Solche Raumzeitzustände aber sind instabil, wie 2005 Stepen Hsu und Roman Buniy zeigen konnten.
     
  • Ein zweites Argument hat Hawking selbst ausgearbeitet: Selbst wenn das Problem mit der negativen Energie nicht existieren würde, könnten Teilchen mit einer solchen Zeitmaschine in der Zeit wiederholt zurückreisen, so dass sich ihre Zahl explosionartig bis ins Unendliche hinein steigen würde, was dann zu einem Kollaps der Raumzeit führen würde (so jedenfalls zeigen Rechnungen für klassische Teilchen; ob genauere quantenphysikalische Rechnung dieses Ergebnis bestätigen, ist bislang nicht bekannt: Sie durchzuführen ist wohl allzu schwierig).

Wie dem auch sei:
 
Es sieht schlecht aus für Reisen in die Vergangenheit.
 
Viel spricht dafür, dass die Zeit nicht Teil der Wirklichkeit, sondern nur ein von den Möglichkeiten des Beobachtens geprägter Eindruck des Beobachters ist.
 
Diese Möglichkeiten sind unvollkommen ( da die Lichtgeschwindigkeit endlich ist ).

 
 
 
Quelle der Fakten: Heinrich Päs: Die perfekte Welle — Mit Neutrinos an die Grenzen von Raum und Zeit, Piper 2011, Kapitel 14: Wie baut man eine Zeitmaschine?
 
Päs — Professor für Theoretische Physik an der TU Dortmund — forscht über Neutrinos und Theorien jenseits des Standardmodells.


 

 Beitrag 0-158
Warum in unserer Welt die Zeit nur eine Richtung kennt

 
 

 
Wie Hawking sich

die Unumkehrbarkeit der Zeit erklärt



Steven Hawking ( 1996, Zitat ):
 
Ich werde diese Vorlesung mit einem Thema abschließen, zu dem Roger Penrose und ich unterschiedliche Meinung haben — dem Zeitpfeil:
 
In unserem Bereich des Universums gibt es eine klare Unterscheidung zwischen der Vorwärts- und der Rückwärtsrichtung der Zeit. Man muss sich nur einen rückwärts laufenden Film ansehen, um den Unterschied zu erkennen. Man sieht dann Tassen, die nicht mehr vom Tisch fallen und zerbrechen, sondern Scherben, die auf den Tisch springen und sich dort zu einer Tasse zusammenfügen.
 
Die lokalen Gesetze, denen die physikalischen Felder genügen, sind zeitsymmetrisch, genauer gesagt CPT-invariant.
 
Der beobachtete Unterschied zwischen Vergangenheit und Zukunft muss also von Randbedingungen des Universums herrühren.
 
Nehmen wir an, das Universum sei räumlich geschlossen, expandiere zu einer maximalen Größe und kollabiere dann wieder. Wie Penrose betont, sieht das Universum an beiden Enden dieser Geschichte gravierend unterschiedlich aus: Was wir als seinen Anfang bezeichnen, scheint ein sehr glatter, regulärer Zustand gewesen zu sein. Wenn es jedoch wieder kollabiert, erwarten wir, dass es sehr ungeordnet und irregulär wird.
 
Da es weitaus mehr ungeordnete als geordnete Konfigurationen gibt, bedeutet dies, dass die Anfangsbedingungen unglaublich genau hätten ausgewählt werden müssen.
 
Es scheint sich daher so zu verhalten, dass an den beiden Enden der Zeit unterschiedliche Randbedingungen vorliegen müssen.
 
Und tatsächlich: Rogers Vorschlag sieht vor, dass der Weyl-Tensor an einem Ende verschwindet, jedoch nicht am anderen. Der Weyl-Tensor ist der Teil der raumzeitlichen Krümmung, der nicht über die Einstein-Gleichungen lokal durch die Materie bestimmt ist. Er sollte also in den glatten, geordneten Phasen des frühen Universums klein, im kollabierenden Universum aber groß sein.
 
 
Dieser Vorschlag würde die beiden Enden der Zeit voneinander unterscheiden
 
und könnte so den Zeitpfeil erklären.

 
 
Note: Ich schrieb eine Arbeit, in der ich behauptete, der Zeitpfeil kehre sich um, wenn das Universum kollabiere. Danach aber haben mich Diskussionen mit Don Page und Raymond Laflamme davon überzeugt, daß ich damit meinen größten Fehler — mindestens meinen größten Fehler in der Physik — begangen hatte: Das Universum würde während des Kollaps nämlich keineswegs in einen glatten Zustand zurückkehren, der Zeitpfeil also würde sich nicht umkehren: Er würde weiter in die gleiche Richtung deuten wie während der Expansion.
 


 
Quelle: Hawking und Penrose: Raum und Zeit, Rowohlt 1998, Seite 135-137
 
Titel der Originalausgabe: The Nature of Space and Time, Princeton University Press (1996)

 
Note: In general relativity, the Weyl curvature is the only part of the curvature that exists in free space — a solution of the vacuum Einstein equation. It governs the propagation of gravitational radiation through regions of space devoid of matter.

 

 Beitrag 0-327
Über Zeit und die Theorie der Quantengravitation

 
 

 
Carlo Rovelli: » Zeit ist uns fehlende Information «

 


Carlo Rovelli (2014):
 
Unser Zeitbegriff — der thermodynamische Zeitpfeil — entspringt der Tatsache, dass wir stets nur mit makroskopischen Variablen als Mittelwert sehr vieler mikroskopischer Variablen interagieren.
 
Sobald wir den Mikrozustand eines Systems betrachten, existiert das System — als eben dieser Zustand — zeitlos.
 
Doch kaum beschreiben wir das System anhand von Mittelwerten zahlreicher Variablen, verhalten sich diese Wert so, als existiere Zeit im Sinne unserer Alltagserfahrung. [ Aber warum? Weil unvollständige Kenntnis Raum für's Dazulernen lässt und wir ständig Neues erfahren? ]
 
 
Die Zeit ist folglich kein Grundbestandteil der Welt, aber gleichwohl allgegenwärtig, da die Welt riesig ist und wir sie stets nur aus makroskopischer Sicht kennen.
 
 
 
Wörtlich schreibt Rovelli:
 
 
» Die Zeit ist eine Auswirkung der Tatsache, dass wir die physikalischen Mikrozustände der Dinge außer Acht lassen.
 
Die Zeit ist uns fehlende Information — sie ist unser Unwissen.

 
 
In Kapitel 7 habe ich gezeigt, dass sich der Begriff der Zeit für eine physikalische Beschreibung [ im Modell der Schleifen-Quanten-Gravitation ] erübrigt und es letztlich sogar besser ist, ganz auf ihn zu verzichten. Hat man dies erst mal erkannt, wird es einfacher, die Gleichungen der Quantengravitation aufzustellen. «
 
 
 
In Kapitel 7 schreibt er:
 
 
Die Forschung der Quantengravitation drehte sich lange nur um Fragen des Raumes, ehe sie den Mut aufbrachte, sich der Zeit zuzuwenden. Erst ab etwa 2000 hat sich das Verständnis der Zeit etwas geklärt.
 
Der Raum als ein amorpher Behälter für die Dinge verschwindet mit der Quantengravitation aus der Physik. Die Dinge (Quanten) liegen nicht im Raum, sondern im Umfeld der jeweils anderen Quanten. Der Raum ist das Gewebe ihrer nachbarschaftlichen Beziehungen.
 
Die Gravitationsquanten entwickeln sich nicht in der Zeit, es entsteht vielmehr die Zeit als Folge ihrer Wechselwirkungen.
 
Wie sich in der Wheeler-DeWitt-Gleichung zeigt, ist die Zeit aus den Gleichungen verschwunden.
 
 
 
Wie der Raum sich als ständig ausgetauschtes Spin-Netzwerk darstellt (S. 191-195):
 
 
Die Graphen, die die Quantenzustände des Raumes, d.h. des Gravitationsfeldes, beschreiben, nennt man Spin-Netzwerke. Sie sind durch ein Volumen V für jeden Knoten und ein ganzzahliges Vielfaches von 1/2 für jede Verbindungslinie gekennzeichnet (Spin).
 
Der Raum [ als Summe dieser Volumina V ] ist diskret. Diese Einsicht bildet den Kern der Theorie der Quantengravitation.
 
Zwischen den Photonen, d.h. den Quanten des elektromagnetischen Feldes, und den Raumquamten V besteht ein entscheidender Unterschied: Photonen existieren im Raum, während Raumquanten den Raum selbst ausmachen.
 
Photonen sind durch ihre Position, ihr » Wo sie sich befinden, d.h. welche Raumquanten sie verbinden « charakterisiert, Raumquanten aber haben keinen Aufenthaltsort. Die charakterisieren sich durch die Information, neben welchen anderen Raumquanten sie liegen.
 
Ich kann mir vorstellen, mich von einem Raumkörnchen an einem Link entlang zu einem anderen zu begeben. Wenn ich so Körnchen um Körnchen weiterschreite, bis ich zum Ausgangskörnchen zurückgekehrt, d.h. einen Rundweg gegangen bin, habe ich eine Schleife — einen » Loop « — abgeschritten. Dies sind die ursprünglichen Loops der Theorie.
 
In Kapitel 4 habe ich gezeigt, dass sich die Krümmung des raumes messen lässt, indem man bestimmt, ob ein Pfeil, den man über den ganzen Weg mitgeführt hat, in seine ursprüngliche Zeigerichtung oder gedreht an den Ausgangspunkt des Weges zurückkommt. Die Mathematik der Theorie bestimmt die Krümmung für jeden geschlossenen Weg im Spin-Netzwerk. Die ermöglicht eine Einschätzung der Krümmung des Raumes und damit der Stärke des Gravitationsfeldes.
 
Es sei noch daran erinnert, dass
     
  • die Art, wie sich ein Spin-Netzwerk entwickelt, zufallsgetrieben ist (wir können nur Wahrscheinlichkeiten berechnen).
     
  • Auch müssen wir und den Austausch der Spin-Netzwerke als Auswirkung des Raumes auf die Dinge denken: So wie sich ein Elektron etwa an keinem bestimmten Ort, sondern als Wahrscheinlichkeitswolke an allen Orten befindet, so ist auch der Raum kein spezifisches Spin-Netzwerk, sondern eine Wahrscheinlichkeitswolke über sämtliche möglichen Spin-Netzwerke.

Damit stellt der Raum sich — unterhalb der Planckskala — dar als ein waberndes Gewimmel aus Gravitationsquanten, die
     
  • wechselseitig aufeinander einwirken,
     
  • alle gemeinsam auf die Dinge einwirken
     
  • und sich in dieser Wechselwirkung als ständig durch Quantenfluktuation umgebautes Spin-Netzwerk manifestieren.

 
 
Quelle: Carlo Rovelli: Die Wirklichkeit, die nicht so ist, wie sie scheint (2016), S. 276-281 und ab S. 196 bzw. 191

 


 
 
Rovellis Argumentation, die Zeit sei uns fehlende Information, wird wohl nicht jeden überzeugen. Vielleicht sollte man es besser so sagen:
 
 
 
Was wir als Zeit wahrnehmen, ist die nicht aufhaltbare Modifikation unserer Welt durch Quantenfluktuation.
 
Zeit wird uns bewusst, da jedes Entstehen oder Vergehen von Quanten unsere Realität ein klein wenig verändert.

 

 

 Beitrag 0-480
Wie wirklich ist die Zeit?

 
 

 
Was ist Zeit?

 
 
Es gibt dicke Bücher, in denen theoretische Physiker ebenso wie Philosophen darüber nachdenken, was Zeit denn eigentlich sei und ob sie wirklich existiere (genauer: als von welcher Qualität wir sie uns vorzustellen haben):
 
Die mir am ehesten einleuchtende Antwort auf solche Fragen gibt Carlo Rovelli. Er argumentiert wie folgt:


Carlo Rovelli (2014):
 
Der Vorschlag von der Nichtexistenz der Zeit, der im Zentrum der Bemühungen steht, eine Quantengravitationstheorie zu formulieren sarf nicht mit der naiven Auffassung einer eingefrorenen Welt verwechselt werden.
 
Die Nichtexistenz der Zeit auf grundsätzlicher Ebene zu postulieren kommt aus der Erkenntnis, dass sich keineswegs alle zeitlichen Aspekte unserer Realität mit Hilfe einer absoluten, universellen Zeit, die » vergeht « beschreiben lassen, d.h. mit einem nur eindimensionalen Zeitkonzept.

     
    Es gibt die Zeit — aber halt nur so, wie es ein Oben und ein Unten gibt.
     
    Einsteins Theorie kennt kein Oben und kein Unten, sondern einfach nur das Gravitationsfeld.
     
    In unserer Alltagserfahrung ist unten dort, wohin etwas fällt.
     
    Ganz analog dazu scheint es auch kein Früher und kein Später zu geben: Später ist einfach nur, wohin Zustände sich entwickeln.

 
Man erkennt:
 
Zeit und Raum sich Begriffe ähnlich wie Farbe, Geschmack oder Temperatur.
 
Um die Fundamente unserer Welt zu erkennen, muss man sich solcher Vorstellungen entledigen, muss sich also auch von der Variablen t in physikalischen Formeln verabschieden.
 
 
Unsere übliche Vorstellung von Zeit ist ebenso vielgestaltig wie vielschichtig und enthält eine Fülle impliziter Hypothesen und Vorannahmen. Es handelt sich um ein ganzes Bündel von Vorstellungen, die sich in unserem kopf mischen, das das Auflösungsvermögen unserer Sinnesorgane begrenzt ist.
 
Das Universum ist riesig und komplex. Es gibt Abermilliarden von Teilchen, die Einfluss auf einander ausüben und noch mehr Variable, die entsprechenden Felder zu beschreiben. Wir erkennen, dass dieses ganze System in seinem dynamischen Verhalten von Gleichungen bestimmt wird, in denen die Zeit auf fundamentalem Niveau gar nicht auftaucht:
 
 
Schleifenquantengravitation beschreibt den Raum als Spin-Netzwerk ohne von Zeit sprechen zu müssen.
 
 
Wir messen immer nur einen winzigen Teil aller Variablen, welche das System bestimmen.
 
Wenn wir beispielsweise ein Stück Metall einer bestimmten Temperatur untersuchen, können wir seine Temperatur messen, seine Länge, seinen Ort, aber nicht die mikroskopischen Bewegungen jedes seiner Atome — obgleich doch gerade sie seine Temperatur verursachen.
 
In solchen Fällen benutzen wir nicht nur die Gleichungen der Dynamik, um die Physik des Systems zu beschreiben, sondern auch die der statistischen Mechanik und Thermodynamik.
 
 
Ausgehend von einer Theorie, welche die Existenz der Zeit grundsätzlich leugnet, lässt sich im Kontext der statistischen Physik aber dennoch eine makroskopische Zeit wiederfinden: Sie tritt auf als emergentes Phänomen, erzeugt von einer ganzen Reihe mikroskopischer Prozesse, die sich im Detail durch uns gar nicht verfolgen oder beschreiben lassen.

 
Und so kann man die Zeit sehen als eine Folge unseres Nichtwissens über die Details sich ergebender Veränderung.
 
Würden wir all diese Details genau kennen, hätten wir nicht das Empfinden eines Fortschreitens von Zeit.
 
So aber nehmen wir summarisch nur Mittelwerte wahr, aus denen eine neue Vorstellung — die Zeit — hervorgeht.
 
Eben ganz so, wie wir pauschal dort den Eindrück von Wärme verspüren, wo sich die Effekte vieler Bewegungen von Molekülen für uns undurchschaubar aufsummieren. Auf molekularer Ebene erkennen wir kein einziges der Moleküle ist warm.

 
 
Denken wir nochmals an das Unten und das Oben:
 
Es ist nicht das Unten, das die Objekte fallen lässt. Es ist vielmehr das Fallen der Objekte, welches für uns den Begriff » Unten « definiert. Unten ist einfach da, wohin alles fällt, und so ist auch das » Später « einfach nur das Entstehen neuer Zustände. Die Ordnung, in der sie entstehen muss keine lineare sein.

 
Zeit ist demnach nur gedankliches Konzept.
 
In eindeutige Richtung fließt sie nur,
 
soweit uns die Menge aller betrachteten Zustände eines Systems als linear angeordnet erscheinen.

 
 
Note: Rovelli weist explizit darauf hin, dass dies seine Vorstellung von Zeit sei, und dass sein Freund Lee Smolin nicht glaubt, dass das schon die ganze Wahrheit sein könne. Er hat seine Position ausführlich beschrieben in Lee Smolin: Time Reborn (2014).
 


 
Quelle: Carlo Rovelli: Und wenn es die Zeit nicht gäbe? (2018), S. 126-158


 

 Beitrag 0-511
Wie sich relative von absoluter Zeitdilatation unterscheidét

 
 

 
Absolute und relative Zeitdilatation



Siegfried Petry:
 
Bei den relativistischen Effekten der Zeitdilatation (Zeitverzögerung) muss strikt zwischen der absoluten und der relativen Zeitdilatation unterschieden werden.
     
  • Die absolute Zeitdilatation — sie ist die Ursache des so genannten Zwillingsparadoxons — besteht darin, dass bei der Beschleunigung eines Bezugssystems der Gang der darin befindlichen Uhren und allgemein die Geschwindigkeit aller Abläufe verzögert werden.
     
    Sie wurde experimentell bestätigt, indem man eine Atomuhr in einem schnellen Flugzeug auf eine Reise um die Erde schickte (Hafele-Keating-Experiment). Nach der Rückkehr ging die Uhr wieder genau so schnell wie die Vergleichsuhr am Boden, war jedoch retardiert, das heißt, ihre Anzeige war zurückgeblieben:
     
    Die im Flugzeug transportierte Uhr — und mit ihr die Besatzung des Flugzeugs — war weniger gealtert als alles auf dem Boden zurückgebliebene.
     
     
  • Die relative Zeitdilatation dagegen besteht darin, dass für die Beobachter in zwei relativ zu einander bewegten Bezugssystemen die Uhren im jeweils anderen Bezugssystem anders zu gehen scheinen als seine eigene.
     
    Genauer: Den Dopplereffekt herausgerechnet scheint jedem Beobachter jede relativ zu ihm bewegte Uhr umso langsamer zu gehen, je schneller sie sich relativ zu ihm bewegt.
     


 
Quelle: Siegfried Petry: Spezielle Relativitätstheorie (einschließlich aller Rechnung dazu)


 

  Beitrag 2085-1
Zeit im (unterschiedlichen) Sinne von Newton vs Leibniz und Einstein

 
Hallo zusammen,

bisher wusste ich, dass Newton und Leibniz miteinander darüber stritten, wer von beiden die Priorität bei der Erfindung der Infinitesimalrechnung hat.

Nun erfahre ich, dass sie auch über das Wesen der Zeit miteinander gestritten hatten. Newton glaubte an eine absolute Zeit hingegen Leibniz hielt eine absolute, von allen Dingen unabhängige Zeit für ein Unding. Leibniz war damit damals schon näher an Einstein als Newton.

Das geht aus der Buch-Rezension Großer Streit um die Zeit hervor:

Zitat:
Newton postulierte: Das, was zeitgenössische Uhren sukzessive immer genauer messen, ist eine absolute Zeit, die völlig unabhängig von allen physikalischen Zusammenhängen gleichmäßig dahinfließt, sozusagen der Pendelschlag einer vollkommenen Uhr. Die absolute Zeit bildet – zusammen mit dem ebenso absoluten Raum als vollkommenem Metermaß – gewissermaßen die physikalische Bühne, auf der sich alle Vorgänge des Universums abspielen.

Newtons absolute Zeit beherrschte die Physik unangefochten 200 Jahre lang, bis Einstein sie mit der Relativierung der Zeitmessung entthronte. Seither wissen wir: Bewegte Uhren gehen langsamer.

Wie de Padova zeigen will, mutet darum das, was Leibniz unter "Zeit" verstand, heutzutage durchaus modern an. Leibniz hielt eine absolute, von allen Dingen unabhängige Zeit für ein Unding; sie sei vielmehr eine Eigenschaft der Bewegung von Objekten. In einem Weltall ohne ein einziges veränderliches Ding wäre es Leibniz zufolge sinnlos, von Zeit zu sprechen.

Ganz in diesem Sinn sollte Einstein zu Beginn des 20. Jahrhunderts definieren: Zeit ist das, was Uhren messen.

Insofern sind heutige Physiker eher Leibnizianer als Newtonianer.

M.f.G. Eugen Bauhof
 

  Beitrag 1376-1
Was ist Zeit?

 
Auch wenn ich absolut fasziniert von dieser Seite bin ( ein Kompliment an die Verantwortlichen ), habe ich mir schon oft die Frage gestellt: "Was ist Zeit eigentlich?"
Mit den bisher veröffentlichen Theorien und wissenschaftlichen Meinungen kann ich aber leider nicht besonders viel anfangen.
Ich bin dann irgendwann zu meiner persönlichen Erkenntnis gekommen, daß es Zeit NICHT gibt, sondern daß sie nur eine Erfindung der Menschen ist.
Alles bleibt so wie es ist. Der Mensch hat die Zeit erfunden, um sichtbare Veränderungen zu beschreiben, um seinen Tagesablauf zu gestalten oder einfach nur um sein Dasein zu definieren.
 

  Beitrag 1376-43
Zeitablauf ist fortwährende Zustandsänderung

 
Harti aus 1376-42:
Die Vorstellung von Zeit ist eine Folge unserer Fähigkeit, Geschehensabläufe (Veränderungen) zueinander in Beziehung zu setzen. Diese Vorstellung hat sich derartig verselbständigt, dass wir Zeit fälschlich als etwas objektiv Existierendes empfinden.

Das scheint mir eine sehr schöne Erklärung.

Sie kommt mir nur insofern noch nicht so ganz perfekt vor, als sie sich auf den Begriiff Ablauf (in Geschehensabläufe) stützt.
Er aber basiert auf Begriffen wie vorher und nachher, die wiederum nur für den wohldefiniert sind, der die übliche Vorstellung von Zeit hat — womit man sich beim Definieren dann im Kreis bewegt hätte.

Mein Vorschlag also: Den Begriff Ablauf eliminieren indem man von einer durch Ursache und Wirkung gegebenen transitiven Relation auf der Menge aller Ereignisse spricht (von einer Ordnung im Sinne der Mengenlehre).


PS: Ich will nicht verheimlichen, dass das dann sehr gut zu meiner Zeittheorie passen würde.

grtgrt
 

  Beitrag 2050-17
Erklär mir doch einmal die Zeit ...

 
 
Horst in 2050-16:
 
Erklär doch einfach mal – oder mal einfach – wie meßbare(!) "Zeit" dadurch entstehen kann, dass sich etwas(?) fortentwickelt!

Hallo Horst,

die Zeit existiert in wenigstens zweierlei Ausprägung:

  • Als  m e s s b a r e  Größe ist sie einfach nur ein Konzept, welches sich der Mensch so  d e f i n i e r t  hat, dass es ihm nützt.
  • Nur als etwas, das  v e r g e h t  (zwangsweise verbraucht wird) ist die Zeit ein durch die Natur geschaffenes Objekt:
    Die Natur "strickt" sie wie ein großes Tuch, das niemals fertig wird. Wie ich mir dieses "Tuch" vorstelle, habe ich den Forumsteilnehmern hier schon mal beschrieben. Nachzulesen ist all das

Bin gespannt, ob Dir damit gedient ist.

Gruß, grtgrt
 

  Beitrag 2050-23
Ticks der kosmischen Uhr sind die Qanten aller Fortentwicklung (und daher die Zeit im Sinne der Natur)

 
 
Horst in 2050-22:
 
Erkläre mir bitte plausibel und nachvollziehbar – ohne Links – den physikalischen Vorgang, wie durch "fortentwickeln" der Existenz von "etwas"(?) das "Objekt" Zeit entsteht und dessen Struktur.


Hallo Horst,

zunächst sei gewarnt: Wovon ich da spreche ist zunächst mal nur  m e i n e  ganz  p e r s ö n l i c h e  Idee davon, was Zeit für die Natur sein könnte.
Ich habe keine Ahnung, wie viele Physiker diese Idee plausibel fänden, wenn sie davon erführen — vielleicht wirklich niemand.

Leute, die über Schleifenquantengravitation forschen, stellen sich Ähnliches vor (aber keineswegs so Einfaches wie ich). Sie haben mich letztlich auf meine Idee gebracht.


Der physikalische Vorgang, nach dem Du frägst, ist das, was man ein Elementarereignis nennt: Zwei oder mehr Quantensysteme stoßen zusammen, verschmelzen, und teilen sich sofort wieder auf in eine Menge neuer Quanten.


Elementarereignisse in diesem Sinne sind sozusagen die Quanten aller Fortentwicklung von "etwas".

Ich nenne sie » die Ticks der kosmischen Uhr «.



Was alles ein Etwas  a u ß e r h a l b  unseres Universums sein könnte, kann ich dir nicht sagen (da habe ich noch nicht mal eine Ahnung).

I n n e r h a l b  unseres Universums aber ist jenes Etwas schlicht und einfach

die Menge ALLER in ihm gerade existierenden Elementarteilchen mit oder ohne Ruhemasse.


Mehr dazu kann ich Dir nun wirklich nicht kürzer erklären als in meinen schriftlichen Beiträgen, die hier und hier online verfügbar sind.


Gruß, grtgrt
 

  Beitrag 2050-29
Erste nicht-virtuelle Materie

 
 
Horst in 2050-27:
 
Es stellt sich da natürlich die Frage, wie Quantenschwärme vor "Erschaffung" der "Zeit" herumschwärmen konnten.
Wenn es "Zeit" noch gar nicht gab, gab es ja für sie auch noch keine Bewegung um zusammen zu kommen!

Kannst du mir diesen Widerspruch erklären?


Hallo Horst,

Du vergisst, dass virtuelle Teilchen per Quantenfluktuation aus dem Nichts entstehen (wie ich in Beitrag 2050-26 ja auch sagte). Erste solche Fluktuation könnte demnach recht gut der Anfang der » Zeit im Sinne der Natur « gewesen sein.

Wir wissen heute ja sehr gut, dass nicht wirklich  j e d e s  Paar virtueller Teilchen sich zur gegenseitigen Auslöschung wieder zusammenfindet (denk an Hawking Strahlung). Wo das passiert, entsteht nicht-virtuelle Materie.

Gruß, grtgrt
 

  Beitrag 1376-8
Zur eigentlichen Natur der Zeit

 
Meine These:

Zeit entspricht den Kosten, die unsere Bewegung durch den Kosmos verursacht.


Zeit = Verbrauch an Lebensenergie


Mein Beweis:

Wer mit dem Auto unterwegs ist, kennt das: Die Geschwindigkeit v, mit der er sich bewegt, hat zwei Komponenten

v = v(1,2,3) + v(4) ,

derart dass
  • v(1,2,3) seine Geschwindigkeit durch den 3-dimensionalen Raum ist und
  • v(4) die Geschwindigkeit, mit der sich Kosten anhäufen (über Benzinverbrauch und Fahrzeugverschleiß): die Geschwindigkeit also, mit der sich sein Vermögen   dadurch reduziert, dass er sich per Auto bewegt — das Ausmaß, in dem seine Ortsveränderung Resourcen verbraucht.

Nach Einstein bewegt sich jeder Körper ganz grundsätzlich stets mit Lichtgeschwindigkeit c durch ein 4-dimensionales Universum U, das man als Menge von Punkten gesehen (die man Ereignisse nennt) als ein kartesisches Produkt ( R, Z) schreiben kann, worin


R den uns vertrauten 3-dimensionalen Raum bezeichnet und
Z einen weiteren, nur 1-dimensionalen Raum, den Einstein und Minkowski die Zeit nennen.


Die Geschwindigkeit, mit der auch wir uns nach Einstein bewegen, ist demnach darstellbar als Summe


c = v(1,2,3) + v(4) ,

worin
v(1,2,3) den auf R und v(4) den auf Z projezierten Anteil unserer Geschwindigkeit darstellt.


Daraus folgt:

v(4) — das also, was wir als vergehende Zeit empfinden — ist die Geschwindigkeit, mit der unsere Bewegung durch den Kosmos unsere Lebensenergie reduziert, die Geschwindigkeit also, die dem Kostenanteil entspricht: die Geschwindigkeit unseres Alterns.


Diese Interpretation ist voll kompatibel mit der bekannten Auflösung des Zwillingsparadoxons.

Sind A und B die beiden Zwillingsbrüder, und ist E1 das Ereignis "Sie schütteln sich die Hände, bevor der eine, B etwa, mit seinem Raumschiff abreist", und ist ferner E2 das Ereignis "Sie schütteln sich die Hände, nachdem B zur Erde zurückgekehrt ist", so gilt ganz offensichtlich: A und B haben sich beide von E1 nach E2 bewegt, haben hierzu aber unterschiedliche Wege genutzt. Es ist daher kein Wunder, dass sie sich durch ihr Reisen von E1 nach E2 unterschiedlich hohe Kosten eingehandelt haben. Erstaunlich ist nur, dass A — der Bruder also, der sich NICHT durch den Raum bewegt hat — den teuren Weg gegangen ist (für ihn verging mehr Zeit). Wir sehen somit:


Je mehr ein Weg von E1 nach E2 durch die Zeit statt durch den Raum führt, desto teurer kommt die Reise.


grtgrt
 

  Beitrag 1376-13
Lebensbatterie: Wie Zeitquanten entstehen und warum alles stirbt

 
Stueps aus 1376-9:
Hallo Gebhard,

dies erinnert mich stark an das Konzept des "entropischen Zeitpfeils". Lässt sich deine These nicht mit Hilfe der "Zunahme der Entropie" äquivalent beschreiben?

Hi, Stuebs!

Ich glaube auch nicht, dass sich Gebhards Vorschlag mit der "Zunahme von Entropie" beschreiben lässt, und zwar weil meines Erachtens sein Ansatz falsch ist. Wir verbrauchen keine Energie, sondern wir wandeln Energie von schwach entropisch in höher entropisch um, wir "zahlen" keine Zeit, Zeit lässt sich nicht verbrauchen .Denn was die vierdimensionale Raumzeit angeht, Gebhard, so kann sie nicht in raum- bzw. zeitartig getrennt werden (siehe dazu Minkowski, auch wenn ich das Zitat jetzt nicht parat habe, du kennst es sicher), Zeit lässt sich so wenig wie Raum verbrauchen. Ich stimme dir allerdings zu, was die Lichtgeschwindigkeit angeht, jedes Objekt bewegt sich stets mit c.Ich - ich spreche da nur für mich - bin immer noch nicht sicher, ob die Zunahme der Entropie im Kosmos tatsächlich kausal etwas mit dem Zeitpfeil zu tun hat, denn die zugrunde liegenden Gesetze der Physik (Thermodynamik) sind zeitinvariant.
 

  Beitrag 1376-21
Identität eines physikalischen Objekts

 
 
Hi Stueps,

du hast natürlich völlig recht, wenn du sagst, ein Objekt trage keine Energie sondern sei Energie.

Andererseits aber lässt sich ja wohl nicht leugnen, dass ich von einem Objekt erst dann sprechen kann, wenn die Energieportion, aus der es besteht, in einer Form vorliegt, die diesem Objekt einen Typ gibt (und die es irgendwie unterscheidbar macht im Meer aller Energie unseres Universums).

Wir kommen hier wieder auf die schon einmal diskutierte Frage, was genau (bzw. wie konkret genau) ein Objekt denn eigentlich sein muss, um als physikalisches Objekt bezeichnet werden zu können. Reicht eine konzeptuelle Abgrenzung, oder muss das Objekt sich in wirklich beobachtbarer Weise von seiner Umgebung abgrenzen? Wenn jene Abgrenzung aber wahrnehmbar ist, könnte man es dann nicht wirklich auch als Energieträger sehen? In dem Sinne also war meine sprachliche Wendung des "Tragens" von Energie gemeint.

Wie du ferner schon sagst: Energie altert nicht. Und wie der Energieerhaltungssatz uns lehrt, verbraucht sie sich auch nicht — sie kann sich höchstens umverteilen oder sich in unterschiedlicher Form zeigen.

Damit ist klar: Als Energie betrachtet, kann ein physikalisches Objekt (z.B. ein Mensch) auf keinen Fall altern. Altern — und sich mehr oder weniger plötzlich verändern — kann nur die Form, in der sich das Objekt beobachtbar macht, und die dazu führt, dass es sich von anderen physikalischen Objekten abgrenzt und so für uns überhaupt erst zu einem wohldefinierten Objekt wird.

Klar ist auch: Solche Alterung kann nur dann in vielen kleinen Schritten erfolgen, wenn das Objekt kein atomares ist (also kein Elementarteilchen).

Würdest du mir da zustimmen?

Beste Grüße,
grtgrt
 

  Beitrag 1376-24
Die Frage nach noch fehlender Deutung

 
C... aus 1376-23:
Hallo Grtgrt,

Grtgrt aus 1376-15:
Kann mir das jemand deuten? Verursacht Bewegung entlang jener geheimnisvollen 4-ten Dimension vielleicht eine Art zusätzlicher Reibung (zusätzlichen Verschleiß, um im Bilde von Beitrag 1376-8 zu bleiben)?

Eine "Reibung" oder ein "zusätzlicher Verschleiß" ist bei der Bewegung entlang der 4. Dimension gar nicht erforderlich. Denn du hast doch Folgendes festgestellt:

Zitat von Grtgrt:
... bewegt sich jeder Körper ganz grundsätzlich stets mit Lichtgeschwindigkeit c durch ein 4-dimensionales Universum U, ...
c = v(1,2,3) + v(4) ...

Wenn also jemand v(1,2,3) aufnimmt, so muss v(4) automatisch (d.h. auch ohne Reibung) kleiner werden, vorausgesetzt v bleibt konstant c.

Hi C...,

ich habe bewusst NICHT gefragt, ob mir das jemand beweisen kann (den Beweis nämlich erbringt die ART).

Ich habe stattdessen gefragt, ob mir das jemand deuten kann (d.h. ob jemand sich irgend eine plausible Begründung dafür vorstellen kann, dass es nun mal so ist, wie die ART uns sagt, dass es ist; und wir wissen ja: sie HAT recht: es ist wirklich so, wie sie uns sagt).

Beste Grüße,
grtgrt
 

  Beitrag 1376-25
Unberechtigte Kritik

 
Hi Henry,

Henry aus 1376-22:
Was verschiedene Beobachter wahrnehmen ist eine Verlangsamung der Zeit - aber nur auf ein beobachtetes Objekte in einem anderen System bezogen. Entscheidend ist aber, was innerhalb des jeweiligen Systems (also des jeweiligen Beobachters) zu messen ist, und darin unterscheiden sich die einzelne Objekte nicht.

Der Witz ist: Die beiden Systeme können einen nicht leeren Durchschnitt haben. Im Hafele-Keating-Experiment etwa, besteht der darin, dass die beiden Teams — das daheimgebliebene mit seiner Atomuhr, und das andere, das eine identisch gebaute Atomuhr auf ihren Flug um die Erde begleitet hat —, nachdem sie sich wieder treffen, beide gemeinsam auf beide Uhren blicken und feststellen, dass die verschiedene Zeit anzeigen, obgleich sie doch bei Beginn des Experiments synchronisiert waren.


Henry aus 1376-22:
Auch deine Graphen - mit denen du an Feynman erinnerst, oder sehe ich das falsch - sind ...

Mein Zeitgraph hat zunächst gar nichts mit Feynmans Diagrammen zu tun. Man könnte aber jedes von ihnen sehen als eine Präzisierung dessen, was ich ein Elementarereignis nenne (genauer: ein Elementarereignis bestimmten Typs).


Henry aus 1376-22:
... nichts gegen neue Ideen einzuwenden, nur müssen die neuen Ideen kompatibel sein zur ART (da könnte es mit deiner These schwierig werden) und zur Quantenmechanik.

Ich kann nicht erkennen, dass meine Theorie irgendwo im Widerspruch zur ART oder zur Quantenmechanik steht.
Ganz im Gegenteil: Aus der ART kommende Ergebnisse haben sie mir nahegelegt.

Gruß, grtgrt
 

  Beitrag 1376-45
Reaktion (von stueps) auf die Idee der Lebensbatterie

 
Grtgrt aus 1376-28:
Zusammenfassend lässt sich feststellen:

Was ich in Beitrag 1376-15 die Lebensbatterie eines Objektes X im Zustand Z nenne, ist nicht anderes als das Paar

B = ( e(X,Z), e(N) )


worin e(N) eine obere Grenze für im Objekt enthaltene Entropie bezeichnet und e(X,Z) die Entropie von X im Zustand Z ist.

Die Differenz 1 – e(X,Z)/e(N) ist dann zu deuten als die in der Lebensbatterie noch vorhandene Restladung (so normiert, dass 1 der voll geladenen Batterie entspricht).

Dem 2. Hauptsatz der Thermodynamik entsprechend wird sie mit an Sicherheit grenzender Wahrscheinlichkeit bei jedem Zustandsübergang kleiner, so dass klar ist: X wird irgendwann sterben, d.h. wird irgendwann so entstellt sein, dass es seine Identität verliert — eben ganz so, wie auch ein Mensch sich mit zunehmendem Alter mehr und mehr verändert, schließlich in einen Sarg gelegt wird, und dort weiter zerfällt, so dass man irgendwann nicht mehr sagen kann, was man da noch vorfindet sei ein Objekt vom Typ Mensch.


Hallo Gebhard,
ich halte dies zwar für eine interessante Idee, vermute aber auf den ersten Blick irgendwo einen fundamentalen Denkfehler. Ich meine nicht deine Gleichungen, die sind m.E. in sich schlüssig. Ich werde wohl noch eine Weile brauchen, um meine Vermutung zu konkretisieren. Ich bitte um Entschuldigung, da dies unseren Austausch jetzt keinen Deut voranbringt. Aber ich habe das Gefühl, dies trotzdem einmal schon schreiben zu müssen.
Ich muss auch ehrlich gestehen, dass ich wohl deine Idee der "Lebensbatterie" noch nicht ganz durchschaut habe, denn:
Ich kann im Moment nicht erkennen, inwieweit sie Vorteile gegenüber anderen "Zeitideen" bietet. Wenn ich ehrlich bin, kann ich im Moment überhaupt keinen konkreten Nutzen erkennen. Dies ist aber meiner leider nicht grenzenlosen Auffassungsgabe geschuldet (ich steh halt sozusagen "auf dem Schlauch" im Moment). Vielleicht kannst du ja mal schildern, welchen Nutzen und welche konkreten Vorteile deine Ideen bieten.
Nebenbei:
Ich halte zu diesem Thema übrigens C...´ Beiträge auch für sehr wertvoll, je mehr ich seine Ideen verstehe, desto genialer finde ich sie. Sie sind (relativ) schlicht und elegant und ergeben sich allein aus den Rt´s.

Lass dich von meiner "temporären geistigen Schwäche" nicht entmutigen. Denn grundsätzlich:
Es kommt nicht oft vor, dass Leute so hartnäckig wie du versuchen, hinter die "Geheimnisse unseres Seins" zu kommen (oder diesen wenigstens ein kleines Stück näher). Ich begrüße so etwas sehr. Da ist es egal, wenn man sich vielleicht auch mal kurz "verläuft". (Was nicht heißen soll, dass dies hier konkret der Fall ist - hier weiß ich es noch nicht). In diesem Sinne: Weitermachen!

Grüße
 

  Beitrag 1376-52
Unterscheiden wir: Koordinatenzeit und Objektzeit

 
H... aus 1376-48:
Hallo zusammen,

ich betrachte Zeit nicht als etwas Eigenständiges, sondern immer im Zusammenhang mit dem Raum (das passiert, wenn man
einmal die RT "inhaliert" hat).
Und damit kann man dann Zeit als Abstand betrachten, ... Für zeitartige Ergeignisse ist dann also die "Zeit" der Abstand in der Raumzeit.

Hallo H...,

ein fundamentaler Fehler, den man bisher überall macht, scheint mir darin zu bestehen, nicht danach zu fragen, ob die Zeit aus Sicht unseres Alltags denn wirklich genau dem entspricht, was die 4-te — leider auch "Zeit" genannte — Dimension des mathematischen Raumes Raumzeit modelliert. Genauer:

Grtgrt aus 1376-14:

Die 4-te Dimension unseres Universums (Zeitbegriff 1) sollte begrifflich unterschieden werden von dem, was wir als Zeit empfinden (Zeitbegriff 2).

Aus meiner Sicht gilt zunächst nur:
  • Zeitbegriff 1 spricht über eine geheimnisvolle 4-te Dimension unserer Welt,
  • Zeitbegriff 2 aber ist das wegabhängige Ausmaß, in dem wir Reisekosten zu zahlen haben [auf unserem Weg durch die Raumzeit, sprich: das Ausmaß, in dem wir altern].

Wie die ART uns zeigt, stehen diese beiden Begriffe durchaus in Beziehung zueinander — was aber noch lange nicht beweist, dass sie ein und dasselbe sind (!).


Mein Verdacht ist: Das Wesen der Zeit richtig zu verstehen, kann nur dem gelingen, der beide Begriffe peinlich genau auseinander hält.
Damit das gelingt, sollte man zwei verschiedene Namen dafür haben, etwa
  • flüchtige Zeit einerseits und
  • Koordinatenzeit andererseits.

Da nach der ART Objekte X und Y, die sich unabhängig voneinander bewegen, stets auch einen mehr oder weniger verschiedenen Begriff flüchtiger Zeit entwickeln, sollte man noch besser von Zeit( X) bzw. Objektzeit sprechen.

Ich werde das ab sofort mal so versuchen.

Beste Grüße,
grtgrt
 

  Beitrag 1376-54
Definition von: Objektzeit

 
 
H... aus 1376-53:
Hallo grtgrt,

meinst du mit Zeit(X) die sogenannte Eigenzeit (dt = ds/c), die gäbe es ja dann schon ? Oder verbirgt sich dahinter
noch etwas anderes?


Statt dt = ds/c kann man auch schreiben c = ds/dt = v (= Objektgeschwindigkeit durch die Raumzeit) = v(1,2,3) + v(4) = s(1,2,3)/dt + s(4)/dt .

Was ich unter Objektzeit verstehe (so ganz bin ich mir da auch noch nicht sicher), ist dann wohl s(4).

Du könntest jetzt einwenden, dass s(4) doch gerade dt ist — das aber gilt nur im Grenzfall s(4) gegen 0. Wir dürfen ja nicht vergessen, dass die Raumzeit eben kein Vektorraum ist, sondern nur eine differenzierbare Mannigfaltigkeit (ein mathematischer Raum also, der sich nur lokal wie ein 4-dimensionaler reeller Vektorraum darstellt. Lokal in diesem Sinne bedeutet: Mit hinreichend guter Genauigkeit nur in einer hinreichend kleinen Umgebung der jeweils betrachteten Position in der Raumzeit.

Was dabei "hinreichend genau" bedeutet, definieren wir selbst über die Ungenauigkeit, die zu tolerieren wir bereit sind (im Kontext irgend einer konkreten Anwendung).

Beste Grüße,
grtgrt
 

  Beitrag 1894-1
RZQ — die Raumzeit der Quanten: die Zeit als gerichteter Graph

 
 

Wie die Allgemeine Relativitätstheorie uns lehrt, entwickeln physikalische Objekte — sobald sie sich relativ zueinander mit unterschiedlicher Beschleunigung bewegen — einen unterschiedlichen Zeitbegriff. Siehe hier Details dazu.

Daraus, so denke ich, ergibt sich zwingend, dass man die Zeit als einen in die 4-dimensionale Raumzeit der Allgemeinen Relativitätstheorie eingebetteten gerichteten Graphen aufzufassen hat, dessen Knoten Ereignisse im Sinne der Quantenmechanik sind (sprich: Punkte der Raumzeit, an denen Elementarteilchen entstehen, zerfallen, oder sich neu aufteilen in Folge einer Kollision).

Genauer beschrieben findet sich diese Idee auf folgenden Seiten (man sollte sie in eben dieser Reihenfolge lesen):

(1)   Vergangenheit und Zukunft genauer definiert

(2)   RZQ: Die Raumzeit der Quanten (Teil 1)
(3)   RZQ: Die Raumzeit der Quanten (Teil 2)

Ich würde mich freuen, wenn Physiker diese meine Ansicht kommentieren könnten (insbesondere dann, wenn sie glauben, Argumente dagegen zu haben).

 

  Beitrag 1894-3
RZQ — die Raumzeit der Quanten: aus Sicht der Quantenmechanik

 
 
Hi Henry,

die Tatsache, dass die Positionen der Ereignisse nur unscharf definiert sind, ist richtig, spricht aber nicht gegen das Modell.
Im übrigen sind natürlich auch die Kanten des Graphen nur unscharf lokalisierbar.

Noch genauer: Wo konkrete Knoten und Kanten liegen, ist nur durch eine ihnen zugeordnete Wahrscheinlichkeitswelle bestimmt — eben das ist der quantenmechanische Beitrag zum Modell.

Zur Verfeinerung und Konkretisierung des Modells RZQ sollte man wohl nicht die Stringtheorie bemühen, sondern eher die dazu konkurriernde Theorie der Schleifen-Quanten-Gravitation. Die nämlich konstruiert über etwas, was ihre Erfinder "Spin Networks" nennen, einen ganz ähnlichen Graphen.

Gruß, ggreiter

 

  Beitrag 1894-8
RZQ — die Raumzeit der Quanten: scheint plausibel (Fakt 1)

 
 
Hi Harti,

mir geht es nicht um einen gefühlten Zeitbegriff, sondern um jene Struktur der Raumzeit der Allgemeinen Relativitätstheorie (ART), die dafür verantwortlich ist, dass für die diskutierten Zwillinge zwischen Abschied und Wiedersehen ganz unterschiedlich viel Zeit im Sinne absolut identisch funktionierender Uhren vergangen ist (Atomuhren fühlen nicht: Sie zählen nur).

Da jene Diskussion zwei Ereignisse (d.h. zwei Punkte der 4-dimensionalen Raumzeit) betrachtet, die — zeitlich gesehen — eben keinen eindeutig bestimmten Abstand haben, kann die Zeit — als etwas, das alle Ereignisse der Raumzeit besucht — eben nicht einfach nur Pfeil sein.

Mit anderen Worten: Worüber ich nachdenke, sind Fragen, die entstehen angesichts der Tatsache, dass die Raumzeit der ART nur lokal ein metrischer Raum im Sinne der Mathematik ist ( aber keineswegs auch global ).

Zu solchem Nachdenken angeregt haben mich neuere Ergebnisse der Experimentalphysik (solche, die für den Bau von Quanten-Computern relevant sind): Man glaubt jetzt nämlich, erste Beweise dafür zu haben, dass zukünftige Ereignisse vergangene beinflussen können, siehe etwa diesen Bericht.

Mir aber kommt der Verdacht, dass jedes Paar solcher Ereignisse eine gemeinsame Gegenwart haben könnte. Dies würde bedeuten, dass Theoretische Physik die Gegenwart nicht einfach nur als Zeitpunkt verstehen darf, sondern als eine Überlappung von Vergangenheit und Zukunft im Sinne eines geeigneten Zeitmodells sehen muss. Mehr dazu auf meiner Seite Ein Verdacht, ....

Nebenbei: Wenn die Zeit nicht auf einem Pfeil fortschreitet, sondern stattdessen auf einem sich mehr und mehr verzweigenden, ständig wachsendem Ast, wäre es fast schon verwunderlich, wenn die Spitzen all seiner Zweige gleiche Zeitkomponente haben sollten.

Mit anderen Worten: Es könnte gut sein, dass nur die (vielleicht) allzu naive Vorstellung, die Zeit sei ein Pfeil, uns glauben lässt, dass die Gegenwart keine zwei Ereignisse enthalten kann, die zeitlich hintereinander geschehen in dem Sinne, dass eines der beiden erst passieren kann, wenn das andere schon passiert ist.

Gruß, ggreiter

 


 

  Beitrag 1894-9
RZQ — die Raumzeit der Quanten: Definition des Zeitgraphen

 


Entsprechen Quantenereignisse dem Ticken einer Uhr?


Zeit — so sagt man — fließt, indem sich Veränderung ergibt. Nimmt man das ernst, muss jeder Kollaps der Wellenfunktion einen kleinsten Zeitschritt bedeuten.

 
Nach allem, was die Theoretische Physik heute weiß, könnte man unser Universum auffassen als ein Paar ( B, E ), für das gilt:
  • B ist eine Menge elementarer Energiequanten ( Schwingungszustände von Strings im Sinne der Stringtheorie) und
  • E ist eine Menge atomarer Ereignisse, deren jedes sich auffassen lässt als ein Paar X( V,Z), derart dass V und Z Teilmengen von B sind.

Jedes Ereignis X = X( V,Z ) kann als Tor aufgefasst werden, welches aus seiner direkten Vergangenheit V = X.V in seine direkte Zukunft Z = X.Z führt.

Man beachte: X.V ist Input von Ereignis X, wohingegen X.Z seinen Output darstellt.

Da die Menge aller Paare ( v, z ) mit v und z aus E derart, dass der Durchschnitt von v.Z und z.V nicht leer ist, eine zweistellige Relation auf E ist, kann man ihre transitive Hülle bilden. Sie ist ebenfalls zweistellige Relation auf E. Man nennt sie die Zeit (und kann sie sehen als einen gerichteten Graphen, dessen Knoten Ereignisse und dessen Kanten Branen sind).


Die Zeit als Pfeil zu sehen wäre nur dann gerechtfertigt, wenn die sie darstellende transitive Relation eine lineare Ordnung (im Sinne der Mathematik) wäre. Sie scheint aber lediglich Ordnung zu sein.


Frage an alle Physiker unter Euch: Wie kann der so definierte gerichtete Graph — man sollte ihn die Struktur der Zeit nennen — mathematisch mit Einsteins Gleichungen der Allgemeinen Relativitätstheorie verschmolzen (oder zu ihnen in Bezug gesetzt) werden?

 

  Beitrag 1894-11
RZQ — die Raumzeit der Quanten: Durch Henry kommentiert

 
Zitat von ggreiter:
Hi Henry,

es scheint mir nicht richtig, nur das, was sich schon als Materie zeigt, als physikalisches Objekt elementarster Art zu bezeichnen. Unteilbare Bausteine des Kosmos sind ganz sicher schwingende Energie-Portionen (nur einige davon zeigen sich zeitweise im Zustand Materie).

Fakt ist die Äquivalenz von Materie und Energie (wenn man Einstein folgt). Dann ist es aber durchaus zulässig, einzelne Quantenobjekte als "Bausteine" der Raumzeit zu betrachten, jedenfalls als Idee. Der Stand der Dinge ist, dass Quanten als virtuelle Teilchen noch "unterhalb" oder "vor" der Raumzeit auftreten.

Quantenphysikalisch betrachtet ist einem massebehafteten Teilchen über seine Ruhemasse eine Frequenz zugeordnet (über E=mc² bzw. E=vh, also v=m(c²/h)). Es ist eine Frage der Messung, ob im Experiment Teilchen- oder Welleneigenschaften betrachtet werden. Es ließe sich trefflich darüber streiten, ob die Teilchen- oder Welleneigenschaften dem Objekt beide zukommen oder ob die Objekte keines von beiden sind.

Zitat von ggreiter:
Nun zu Ereignissen: Als Position eines Ereignisses kann man nur die Summe aller Punkte verstehen, an denen sich die am Ereignis beteiligten Quanten dann gerade befinden. Es ist aber keineswegs so, dass die Position eines Quantums nur ungenau beobachtbar wäre: Sie ist wirklich auch ungenau definiert.

Diese Definition ist nicht richtig. Ein Ereignis definiert bezogen auf die Raumzeit ist ein Punkt der Raumzeit. Er wird deshalb als Ereignis bezeichnet, weil zu seinen drei Raumkoordinaten stets eine gerichtet Zeitkoordinate gehört. Die Position eines Quantums ist nicht wegen der Definition unbestimmt, sondern wegen der zugrunde liegenden Struktur. Eine Definition kann immer nur die Folge der Realität sein, nicht ihre Ursache.

Zitat von ggreiter:
Damit ist klar: Unter der Position eines Ereignisses muss man sich (aus diesen beiden Gründen) eine kleine Region der Raumzeit vorstellen (eine Punktemenge, die Vereinigung von N Punktemengen ist, wenn am Ereignis N Quanten beteiligt sind) — also nicht einen einzigen Punkt im Sinne der Mathematik, sprich: im Sinne der die Raumzeit modellierenden differenzierbaren Mannigfaltigkeit.

Wovon du hier sprichst, sind Ereignisse bezogen auf einen Konfigurations- bzw. Phasenraum.

Zitat von ggreiter:
Nun zur Frage, ob Wahrscheinlichkeitswellen wirklich existieren: Da Interferenz am Doppelspalt zeigt, dass sie interferieren, müssen sie ja wohl existieren. Eine ganz andere Frage ist, was es denn bedeuten mag, zu existieren. Hierzu hat Niels Bohr mal was sehr Treffendes gesagt (siehe meine Seite Zum Wesen physikalischer Aussagen und dem, was man die Realität nennt).

Das ist keineswegs klar. Was hier interferiert, sind z. B. elektromagnetische Wellen. Unter einer Wahrscheinlichkeitswelle ist aber die mathematische Beschreibung für das Auftreffen eines Photons an einer bestimmten Stelle des Schirmes hinter dem Spalt bzw. der Weg dorthin zu verstehen. Die Philosophie lassen wir mal außen vor.


Zitat von ggreiter:
Zu Stringtheorie (ST) und Schleifen-Quanten-Gravitation (SQG): Zwischen beiden besteht insofern ein ganz gewaltiger Unterschied, als die ST eine hintergrund- abhängige Theorie ist, wohingegen die SQG (ebenso wie die ART) ohne Bezug zu irgend einem Hintergrund formuliert ist. RZQ ist hintergrund- unabhängig, also viel näher an SQG als an ST.

Es geht hier überhaupt nicht um die Frage, ob eine Theorie hintergrundabhängig ist oder nicht. Weder die ST noch die SQG ist bisher verifiziert bzw. falsifiziert worden. Deshalb spreche etwas flapsig davon, dass es Geschmacksache ist, welche man bevorzugt

Zitat von ggreiter:
Ich sehe RZQ als eine Art vergröberte SQG, wobei mir aber nicht klar ist, ob die gröbere oder die feinere Sicht die natürlichere ist.

Das liegt daran, dass ich die Spin-Networks nicht wirklich verstehe. Als gut verständlich (und "anschaulich") würde ich sie nicht einzustufen. Hast du den Artikel, auf den ich oben verweise, gelesen?

Ich habe ihn gelesen.
Gruß
 

  Beitrag 1894-12
RZQ — die Raumzeit der Quanten: Zum Ereignisbegriff

 
 
Hi Henry,

danke für diese Klarstellungen.

Speziell zum Ereignisbegriff sei noch gesagt:

Mir war klar, dass man die Punkte der Raumzeit als Ereignisse bezeichnet. Das schien mir aber nur eine Art Konvention zu sein (eine Sprechweise, die sich halt so eingebürgert hat). Ist das richtig, oder steckt mehr dahinter?

Ein Ereignis in meinem Sinn ist jeder atomare Prozess, in dem
  • genau ein Elementarteilchen (ein virtuelles oder ein reales) entsteht oder aufhört zu existieren
  • oder in dem Teilchen kollidieren und sich infolge solcher Kollision neu aufteilen.
Damit hat jedes Ereignis in meinem Sinn Input und/oder Output (und beides kann als Menge elementarer Teilchen aufgefasst werden: als Menge von Energieportionen, denn Masse ist ja nur eine Zustand von Energie — so etwa, wie Eis ein Zustand von Wasser ist).

Als Konfigurationsraum kann ich so ein Ereignis eigentlich nicht sehen.
Darüber, ob man es als Phasenraum bezeichnen kann, muss ich noch nachdenken (so ganz koscher erscheint mir das im Moment aber nicht).

Warum sollte ich nicht einfach bei meiner Definition bleiben?

Beste Grüße,
grtgrt (= Gebhard Greiter)

 

  Beitrag 1894-32
RZQ — die Raumzeit der Quanten: Zeitquanten (Elementarereignisse)

 
 
Hi Gerhard,

in meinen Augen gilt:
  • Bewegung ist die Veränderung einer Position (im wörtlichen oder auch im übertragenen Sinn).
  • Somit ist Bewegung ein Speziallfall von Veränderung.
  • Für mich ist Zeit äquivalent zu Veränderung.
  • Aus Sicht theoretischer Physik scheint mir Veränderung gegliedert in kleinste Quanten (die ich dann Elementarereignisse nenne und als das Ticken einer kosmischen Uhr sehe).

Beste Grüße,
grtgrt = Gebhard Greiter
 

  Beitrag 1894-35
RZQ — die Raumzeit der Quanten: Einordnung dieser Theorie

 
Stueps aus 1894-13:
Hallo Gebhard,

ich habe mich mal eben auf deiner Seite umgesehen, um mir einen groben Überblick zu verschaffen. Leider ist mir in der kurzen Zeit deine Theorie nicht klar geworden - ich habe leider nicht die Seite gefunden, wo sie vorgestellt wird. Ich kann nur ungefähr ahnen, worauf du mit deinen Gedanken hinaus willst. Die Ansätze, die ich halbwegs verstanden habe, halte ich jedoch für sehr interessant ...
Was mich interessieren würde:

Inwieweit unterscheiden sich deine Gedanken von bestehenden (gut etablierten) Modellen?
Welche Voraussagen machen deine Überlegungen, die andere Erklärungsmodelle nicht machen können?
Wo sind Schwächen?
Und zu guter Letzt die Gretchenfrage (etwas gemein, gebe ich zu): Gibt es Möglichkeiten, alles einmal irgendwie zu prüfen?

Hi Stueps,

meine Idee ergab sich,
  • nachdem ich mir die Grundidee der Schleifen-Quantengravitation habe erklären lassen (Spin Networks)

Was ich als den Zeitgraphen bezeichne, entspricht der Gesamtheit aller Spin Networks (auch sie stellen letzlich einen Graphen dar, der Veränderung bis ins Kleinste hinein strukturiert). Leider verstehe ich Spin Networks aber noch viel zu wenig, und schon gar nicht ihre mathematische Beschreibung.

Es könnte also schon sein, dass mein Ansatz sich irgendwann als so eine Art "gesunder-Menschenverstand-Version" des Ansatzes der Schleifen-Quantengravitation herausstellt.

Auf jeden Fall habe ich keine mathematische Formulierung meiner Idee in Form von Formeln oder Gleichungen.

Ich sehe mich zudem nicht in der Lage, meine Theorie so in mathematischer Form zu präsentieren, dass man dann vielleicht schon auf rein mathematischem Wege z.B. Bezüge hin zu Stringtheorie, Schleifen-Quantengravitation, oder den Gleichungen der Quantenphysik entdecken könnte. (Meine mathematische Vergangenheit liegt schon 30 Jahre zurück, und Physik habe ich nie studiert.)

Einzige Vorhersagen meiner Theorie sind bislang:
  • Die Zeit hat nicht nur eine Dimension.
  • Wenn es überhaupt Sinn macht, der Zeit Dimensionen zuzuordnen, dann sicher nur lokal (auf Seite Zum Dimensionsbegriff (etwa der Physik) habe ich begonnen, mir über verschiedene Dimensionsbegriffe Gedanken zu machen; es wird dort auch kurz auf den Begriff lokaler Dimension eingegangen).

Nebenbei: Meiner Ansicht nach ist den Physikern noch nicht richtig aufgefallen, dass es einen Unterschied gibt zwischen ganz real vorhandenen Dimensionen (d.h. Freiheitsgraden) und Dimensionen, die rein konzeptueller Art sind (und deswegen modellspezifisch und von ihrer Natur her nur unscharf definiert).

Grundsätzlich interessant sein könnte, zu untersuchen,
  • ob (und wenn ja wie) Quantenverschränkung den Zeitgraphen mit bestimmt
  • und wie/ob sich das im Doppelspalt-Experiment beobachtete Interferenzmuster modifiziert, wenn man den Weg über Spalt 1 sehr viel länger macht als den Weg über Spalt 2.

Mit besten Grüßen,
grtgrt = Gebhard Greiter
 

  Beitrag 1894-40
RZQ — die Raumzeit der Quanten: scheint plausibel (Meinung 1)

 
Grtgrt aus 1894-32:
  • Bewegung ist die Veränderung einer Position (im wörtlichen oder auch im übertragenen Sinn).
  • Somit ist Bewegung ein Speziallfall von Veränderung.
  • Für mich ist Zeit äquivalent zu Veränderung.
  • Aus Sicht theoretischer Physik scheint mir Veränderung gegliedert in kleinste Quanten (die ich dann Elementarereignisse nenne und als das Ticken einer kosmischen Uhr sehe).

So ist es vermutlich. Was Zeit ist, verstehe ich nicht, bin aber davon überzeugt, dass Bewegung die Grundlage für Zeit bildet.

Gibt es Quellen im Internet, die gut nachvollziehbar erklären, wie Bewegung im Makrokosmos durch Bewegung im Mikrokosmos hervorgerufen wird? Gibt es Erklärungsmodelle, was Bewegung auf mikrophysikalischer (quantentechnischer) Ebene ist? Weiß jemand einen Link?
 

  Beitrag 1894-45
RZQ — die Raumzeit der Quanten: scheint plausibel (Meinung 2)

 
Hallo Gebhard,

Grtgrt aus 1894-35:
Es könnte also schon sein, dass mein Ansatz sich irgendwann als so eine Art "gesunder-Menschenverstand-Version" des Ansatzes der Schleifen-Quantengravitation herausstellt.

so ich deinen Ansatz intuitiv richtig verstanden habe, stimme ich dir zu. Deshalb finde ich diesen Ansatz durchaus interessant.

Zitat:
Einzige Vorhersagen meiner Theorie sind bislang:
Die Zeit hat nicht nur eine Dimension.
Wenn es überhaupt Sinn macht, der Zeit Dimensionen zuzuordnen, dann sicher nur lokal (auf Seite Zum Dimensionsbegriff (etwa der Physik) habe ich begonnen, mir über verschiedene Dimensionsbegriffe Gedanken zu machen; es wird dort auch kurz auf den Begriff lokaler Dimension eingegangen).

Dies ist natürlich hochinteressant. Hier muss ich mich wieder erst näher auf deinen Seiten erkundigen. Spontan würde ich fragen, auf welchen Ebenen Zeit eine Mehrdimensionalität aufweist. Nur auf mikroskopischer - sprich auf Planck-Ebene, oder auch auf makroskopischer, erfahrbarer Ebene.

Zitat:
Nebenbei: Meiner Ansicht nach ist den Physikern noch nicht richtig aufgefallen, dass es einen Unterschied gibt zwischen ganz real vorhandenen Dimensionen (d.h. Freiheitsgraden) und Dimensionen, die rein konzeptueller Art sind (und deswegen modellspezifisch und von ihrer Natur her nur unscharf definiert).

Oh, da bin ich mir nicht sicher. Aus dem, was ich bisher erfahren habe, sind sich die Wissenschaftler durchaus der Schwierigkeiten mit den vielen Dimensionen bewusst. Und wenn ich es richtig deute, fühlt sich niemand damit so richtig wohl. Ähnlich wie bei dem Begriff "Singularität" bei schwarzen Löchern und dem Urknall. Sie nehmen die vielen Dimensionen so lange in Kauf, so lange ihre Theorien gute Voraussagen machen, und Dinge herleiten, die andere Theorien nicht herleiten können.

Zitat:
Grundsätzlich interessant sein könnte, zu untersuchen,
ob (und wenn ja wie) Quantenverschränkung den Zeitgraphen mit bestimmt
und wie/ob sich das im Doppelspalt-Experiment beobachtete Interferenzmuster modifiziert, wenn man den Weg über Spalt 1 sehr viel länger macht als den Weg über Spalt 2.

Das verstehe ich ehrlich gesagt wieder nicht. Wir haben hier Quantenverschränkungen und Interferenzen schon oft diskutiert, und mir schwirrte jedes mal der Kopf, wenn´s ans "Eingemachte" ging. Bin halt auch nur blutiger Laie.
 

  Beitrag 1894-53
RZQ — die Raumzeit der Quanten: Im Lichte chemischer Reaktionen

 
Henry aus 1894-33:
Vergleiche doch bitte mal zwei identische chemische Reaktionen unter verschiedenen Temperaturen, und zwar ohne einen Begriff der Zeit (also ohne schneller, früher vorher und Dergleichen). Wenn Zeit Veränderung ist, sollte das möglich sein.

Die Reaktion von Sauerstoff und Wasserstoff zu Wasser ereignet sich ganz elementar für jedes einzelne beteiligte Atom. Worauf ich hinaus will ist, dass du erklärst, wie die Veränderung (die du ja mit Zeit gleichsetzt) bei verschiedenen Temperaturen ohne zeitliche Begriffe verglichen werden können. Außerdem, du hast nicht angedeutet, dass Zeit im Makrokosmos etwas anderes sein soll als im Mikro- oder Submikrokosmos.

Eigentlich wollte ich selbst zeitliche Begriffe vermeiden, aber bei verschiedenen Temperaturen laufen chemische Reaktionen verschieden schnell ab.

 
Hi Henry,

Stellen wir uns mal vor, wir würden einen Topf mit Wasser auf den Herd stellen und dann die Kochplatte anschalten, um das Wasser zum Kochen zu bringen.

Vergleichen wir diese Situation mit den Zwillingen aus Beispiel Zw, so sind wir, die wir vor dem Kochtopf stehen, vergleichbar mit dem zuhause gebliebenen Zwilling, während die Wasseratome im Topf seinem fortreisenden Bruder entsprechen:

Die Wasseratome werden durch die ständige Energiezufuhr immer wilder im Topf herumsausen (also beschleunigt werden und zusammenstoßen, was, so denke ich, die Zahl der Elementarereignisse im Topf hochtreiben wird).

Stellen wir uns weiter vor, dass — nachdem das Wasser zum Kochen kam —, wir den Herd abschalten und warten, bis der Topf und das Wasser wieder ebenso kalt ist wie zu Beginn des Experiments.

Dieser Zeitpunkt entspricht dem, an dem der aus dem All zurückkehrende Zwilling seinem daheingebliebenen Bruder wieder die Hand schüttelt.

Kurz: Für die Wasseratome im Topf wird die Zeit weniger schnell vergangen sein als für uns.


Was wir daraus lernen ist:

Der Fluß der Zeit im Kosmos ist (im Großen wie im Kleinen) vergleichbar mit einem breiten Fluß auf unserer schönen Erde, in dem das Wasser (weil der Fluss nicht überall gleich breit und gleich tief ist) an einigen Stellen schnell und an anderen Stellen langsam fließt. Solche Stellen können, wie wir alle wissen, auch recht nahe beinander liegen.


Mathematisch ausgedrückt bedeutet das:

Die kosmische Uhr kann man sich vorstellen als eine Funktion kU, die jedes Tripel ( v,z,w ) auf einen zeitlichen Abstand abbildet, der einer gewissen Zahl zA von Schwingungen einer Atomuhr entspricht:

zA = kU( v, z, w )


Hierbei ist zA der zeitliche Abstand der Ereignisse v und z, den die Atomuhr anzeigt, wenn sie auf dem Weg w von v nach z reist. Nähme sie einen anderen Weg, würde sie andere Zeit anzeigen.

Beste Grüße,
grtgrt = Gebhard Greiter,

PS: siehe auch [2] und dort die letzte Frage.

 

  Beitrag 1894-57
RZQ — die Raumzeit der Quanten: Sie ist verträglich mit Einsteins Theorie

 
Grtgrt aus 1894-53:
Die kosmische Uhr kann man sich vorstellen als eine Funktion kU, die jedes Tripel ( v,z,w ) auf einen zeitlichen Abstand abbildet, der einer gewissen Zahl zA von Schwingungen einer Atomuhr entspricht:

zA = kU( v, z, w )


Hierbei ist zA der zeitliche Abstand der Ereignisse v und z, den die Atomuhr anzeigt, wenn sie auf dem Weg w von v nach z reist. Nähme sie einen anderen Weg, würde sie andere Zeit anzeigen.
 

Interessante Beobachtung:

Der Abstand s zweier Objekte nach Raum und Zeit wird gemessen über die sog. Minkowski-Metrik, die den üblichen Abstandsbegriff im 3-dimensionalen euklidischen Raum in die Zeitdimension hinein verallgemeinert:

ds2 = c2dt2 – ( dx2 + dy2 + dz2 )


Hier steht c für die im 3-dimensionalen Raum beobachtete Lichtgeschwindigkeit im Vakuum.

Wendet man diese Formel — nach versuchter Division durch dt2 — auf Photonen an (aufs Licht also), so zeigt sie, dass Licht sich niemals durch die Zeit bewegt, wenn es sich durchs Vakuum bewegt (sprich: solange es über nur eine Kante im Zeitgraphen reist).

Ich sehe das als eine weitere Bestätigung meiner Theorie, die ja sagt, dass nur Ereignisse die kosmische Uhr ticken lassen.

Klar wird hier auch, dass das in den Formeln der Allgemeinen Relativitätstheorie auftretende t für lokale Zeit steht.

 

  Beitrag 1894-58
Zeit und Zeitbegriff sind nicht dasselbe!

 
 
Gerhard-28 aus 1894-27:
Was ist der Unterschied von "Zeit" und "Zeitbegriff"?

Ich sehe den Unterschied so:
  • Die Zeit ist ein (recht geheimnisvoller) Teil der Natur.
  • Jeder Zeitbegriff aber ist ein Modell, an dem wir uns klarmachen, wie jener Teil der Natur sich uns zeigt (sprich: wie wir ihn wahrnehmen).

Siehe auch was Niels Bohr uns dazu sagt.

Nebenbei: Es scheint mir naheliegend zu sagen, die Zeit sei jener Teil der Natur, der dafür verantwortlich ist, dass sie sich uns als etwas Lebendiges zeigt — als etwas, das sich ständig wandelt.
 

  Beitrag 1823-146
RZQ — die Raumzeit der Quanten: Zum Motor der kosmischen Uhr

 
Okotombrok aus 1823-7:
Bernhard Kletzenbauer aus 1823-6:
Keine Prozesse, keine Vergleichsmöglichkeiten = keine Zeit
So klar hab' ich das für mich noch nicht.
Wenn wir das Gedankenexperiment weiterdenken, und annehemen, es bewegt sich wieder was, heißt das dann, die Erstarrung hätte gar nicht stattgefunden?

Ja, Zeit ist Veränderung. ABER: Der Fall, dass es keine Prozesse gibt, existiert gar nicht. Genauer:

Einstein sagt:

Zitat:
Die Zeit ist, was man von der Uhr abliest.

Dazu kommt, was ich gelernt zu haben glaube (siehe Zur Struktur der Zeit):

Zitat von grtgrt:
Die einzige umfassende Uhr ist der Kosmos selbst.


Jeder Tick dieser Uhr ist ein atomares Ereignis, in dem eine kleine Portion schwingender Energie entsteht, vergeht, oder mit anderen solcher Portionen kollidiert (was zu einer Verschmelzung und Neuaufteilung dieser Portionen noch im Ereignis selbst führt).

Dass diese kosmische Uhr niemals stehen bleibt, ergibt sich aus dem, was Lisa Randall (auf Seite 238 und 225 ihres Buches Warped Passages ...) schreibt:

Zitat:
Quantum contributions to physical processes arise from virtual particles that interact with real particles.

Virtual particles, a consequence of quantum mechanics, are strange, ghostly twins of actual particles. They pop in and out of existence, lasting only the bares moment. They borrow energy from the vacuum — the state of the universe without any particles.

Da virtuelle Partikel spontan entstehen (ohne beobachtbaren Grund also), ist nicht damit zu rechnen, dass das jemals endet. Sie also sind der Motor, der die kosmische Uhr treibt und die Zeit vergehen lässt.

Mfg, grtgrt
 

  Beitrag 1823-148
Die einzige umfassende Uhr ist der Kosmos selbst

 
Zitat von grtgrt:
Die einzige umfassende Uhr ist der Kosmos selbst.

Jeder Tick dieser Uhr ist ein atomares Ereignis, in dem eine kleine Portion schwingender Energie entsteht, vergeht, oder mit anderen solcher Portionen kollidiert (was zu einer Verschmelzung und Neuaufteilung dieser Portionen noch im Ereignis selbst führt).

Hallo Grtgrt,

man kann jede beliebige Veränderung zur Begründung unserer Vorstellung von Dauer und damit Zeit heranziehen. Ein Rückgriff auf Vorgänge im Mikrokosmos ist nicht nur unzweckmäßig, sondern auch wenig realitätsgerecht. Meine Vorstellung von "Zeit" ist jedenfalls nicht durch Vorgänge im Elementarbereich entstanden. Es werden zwar atomare Ereignisse zur Konstruktion von Uhren genutzt, nämlich die Schwingungen von Cäsiumatomen,
die von Dir genannten Vorgänge sind nach meiner Meinung aber kaum zur Konstruktion von Uhren geeignet, das sie meines Wissens nicht als periodische Ereignisse feststellbar sind. (siehe Deine weitere Äußerung)

Zitat:
Da virtuelle Partikel spontan entstehen (ohne beobachtbaren Grund also), ist nicht damit zu rechnen, dass das jemals endet. Sie also sind der Motor, der die kosmische Uhr treibt und die Zeit vergehen lässt.

Man kann diese Vorgänge vielleicht als "elementarste" ( dies ist eine von mir erfundene Steigerungsform) bezeichnen, zur Normierung von Zeit eignen sie sich aber nicht.

MfG
Harti
 

  Beitrag 1823-149
Globale Zeitnormierung erscheint unmöglich

 
 
Hi Harti,

meine Aussage ist nicht, dass jene elementarsten Vorgänge die Zeit normieren — ich behaupte lediglich, dass sie das verursachen, was wir Menschen als die Zeit bezeichnen und (in stark lokalisierter Form) auch wahrnehmen.

Eine Normierung der Zeit kann sie stets nur lokal normieren (z.B. über die Beobachtung der Schwingungen eines Atoms).

Globale Zeitnormierung scheint unmöglich, da die Allgemeine Relativitätstheorie uns beweist, dass unterschiedlich beschleunigte Objekte ganz unweigerlich einen unterschiedlichen Zeitbegriff entwicklen (und das selbst dann noch, wenn sie Uhren identischer Funktionsweise und Bauart mit sich führen).

Beste Grüße,
grtgrt
 

  Beitrag 1823-153
RZQ — die Raumzeit der Quanten: Genaueres zum Zeitgraphen

 
 
Seid gegrüßt, Henry und E...,

mir ist schon klar, dass virtuelle Teilchen möglicherweise nur sehr selten mit realen Teilchen interagieren. Es reicht aber, wenn sie es selten tun. Genauer:

Dein Einwand, Henry, hat mir klar gemacht, dass der von mir in Beitrag 1894-9 definierte Zeitgraph wohl keine Ordnung darstellt, sondern nur eine transitive Relation. Dann aber kann der Graph aus mehreren Zusammenhangskomponenten bestehen (aus Teilgraphen also, die keine Kante miteinander verbindet).

Wenn nun also ein Elementarereignis ein virtuelles Teilchen erzeugt, welches nie mit einem realen Teilchen interagiert, so wird die in diesem Ereignis beginnende Zusammenhangskomponente des Zeitgraphen halt nur aus einer einzigen Kante bestehen. Sie ist dann zwar uninteressant, spricht aber nicht gegen meine Theorie der Zeit.

Wirklich interessant sind die Zusammenhangskomponenten des Zeitgraphen, die Ereignisse enthalten, in denen Teilchen sich durch Zusammenstoß neu aufteilen.

Wie der Casimir-Effekt hier eine Rolle spielt, habe ich nicht verstanden (bin halt doch zu wenig ein Physiker).

Beste Grüße,
grtgrt (= Gebhard Greiter)

 

  Beitrag 1213-19
Zeitreisen in die Vergangenheit sind NICHT möglich

 
 
Wrentzsch aus 1213-18:
Die Vergangenheit ist der Zustand aus dem sich der Jetztzustand entwickelte.
Die Zukunft ist die Weiterentwicklung des Jetztzustandes.
Damit ist eine Reise in die Vergangenheit unmöglich, weil dieser Zustand nicht mehr existiert.

Eben so sehe ich das auch.

grtgrt
 

  Beitrag 1213-21
Warum Vergangenheit und Zukunft nicht austauschbar sind — und daher die Zeit nicht umkehrbar

 
 
Hans-Peter-Dürr, schreibt in seinen Reflexionen einen Quantenphysikers:
 
Zitat:
 
Aus der Sicht der Quantenphysik ist die Zukunft prinzipiell offen, prinzipiell unbestimmt.

Die Vergangenheit dagegen ist festgelegt, durch Fakten, die durch irreversible Prozesse erzeugt werden.

Die Gegenwart bezeichnet den Zeitpunkt, wo Potentialität zur Faktizität, Möglichkeit zur Tatsächlichkeit gerinnt.

 

 

  Beitrag 1213-20
Sind Raum und Zeit verschiedene Formen derselben Größe?

 
Gregor Lämmer aus 1213-14:
Raum und Zeit waren noch nie miteinander verbunden.

Hallo Gregor Lämmer,

ich würde die Sache folgendermaßen sehen: Eine Bewegung besteht aus zwei Komponenten, einer räumlichen Veränderung und einer zeitlichen (die Bewegung dauert). Zur vollständigen Erfassung einer Bewegung muss man beide Veränderungen in die Betrachtung einbeziehen.
Dies kann auf zwei verschiedene Arten geschehen:
1) Auf herkömmliche (alltägliche) Art, indem man Zeit und Raum trennt. Beispiel: Ich gehe 2 Stunden spazieren (zeitliche Betrachtung) und lege 8 Kilometer zurück (räumliche Betrachtung).
2) Auf relativistische Art, indem man das Ereignis des Losmarschierens und das Ereignis der Ankunft, in einem vierdimensionalen Koordinatensystem als raumzeitliche Veränderungen beschreibt, wobei die Ereignisse durch eine Weltlinie verbunden sind.

Zitat:
Wir sollten nicht länger den Fehler machen, unsere subjektive Wahrnehmung von Phänomenen, die zeitlichen und räumlichen Charakter haben, als verbunden zu betrachten.

Weder die eine noch die andere Art der Betrachtung sind fehlerhaft.
In der herkömmlichen Betrachtung stellen wir zwischen Raum und Zeit eine Beziehung her in Form von Geschwindigkeit (Geschwindigkeit = Strecke/Zeit). Wären Strecke und Zeit vollständig unabhängig voneinander, wären unbegrenzt große und unbegrenzt kleine Geschwindigkeiten möglich.
Es gibt aber eine Geschwindigkeit, nämlich die elektromagnetischer Wellen, bei der die räumliche und zeitliche Komponente der Bewegung, nämlich die elektrische und die magnetische Wirkung nicht zu unterscheiden sind. Der Quotient Strecke/Zeit bzw. Zeit/Strecke ist deshalb konstant 1. Daraus folgt die Konstanz der Lichtgeschwindigkeit. Eine Beziehung, deren Wert nicht von ihrem Kehrwert zu unterscheiden ist, hat den Wert 1.
Dass wir diese Geschwindigkeit tatsächlich mit ca. 300000 Km/sec messen, liegt daran, dass wir bei der Betrachtung die von uns willkürlich gewählten Maßeinheiten Km und sec zugrunde legen. Würden statt km z.B. Meter verwenden, betrüge die Lichtgeschwindigkeit ca. 3 Millionen m/sec.

MfG
Harti
 

  Beitrag 1878-7
Was es bedeutet, in die Zukunft zu reisen

 
 
 
Wie man (prinzipiell wenigstens) in beliebig ferne Zukunft reisen kann

Auf Seite Verschieden schnell durch die Zeit reisen wird anhand von drei Beispielen gezeigt, wie man in die Zukunft reisen kann.

Speziell das dritte Beispiel dort ist interessant.

Wer es nachgerechnet hat, wird sein Ergebnis vorliegen haben in der Form

          S = F( A, Z )

wo F eine Formel ist, A der Abstand vom Schwarzen Loch, in dem der Astronaut eine gewisse Zeit Z wartet, bis er umdreht und zur Erde zurückkehrt,
und S die Zahl der Jahre, die auf der Erde vergangen sein sollen, wenn der Astronaut zurückkehrt.

Natürlich werden die in der Formel auftretende Koeffezienten davon abhängig sein, welches Schwarze Loch der Astronaut zu besuchen gedenkt, und wie heftig er plant, auf seiner Reise zu beschleunigen oder abzubremsen.

Interessant aber ist: Da Vergrößern von A den Sprung S in die Zukunft verkleinert, und da umgekehrt Vergrößern der Parkdauer Z den Sprung S vergrößert, lässt sich praktisch jeder Wert von S erreichen.

Das einzige Problem des Verfahrens ist, dass der Astronaut sterben kann, noch bevor er wieder die Erde erreicht. Man sollte ihn also besser durch einen robusten Roboter ersetzen (oder Raumschiff und Astronaut durch einen Forschungssatelliten).


PS: Reisen in die Vergangenheit sind grundsätzlich NICHT möglich (da Zeit eine Richtung hat, die von Ursache zu Wirkung führt).

 

  Beitrag 52-14
Wir wissen nicht, in welchem Sinne unsere Zeit unendlich sein könnte

 
 
Quante in 52-8:
 
Denn auch Gott muss es ja schon seit ewiger Zeit und für ALLE Zeit geben, denn ansonsten sei die Frage seiner Entstehung hiermit gestellt.


Das Problem ist, dass wir ja noch nicht mal wissen, was Zeit (in unserem Sinne) wirklich ist.

So wie man unendlich große mathematische Räume (Unendlichkeiten also) in immer noch größere einbetten kann — sogar in solche, die in einem Sinne unendlich groß, in einem anderen aber von endlichem Durchmesser sind — könnte es doch auch im Spezialfall rein zeitlicher Dimensionen sein.

Es könnte also sehr gut sein, dass sich jede Ewigkeit in eine noch weit größere Ewigkeit als Teilintervall "endlicher" Länge einbetten lässt.


Als Ewigkeit bezeichne ich alles, was aus zeitlicher Sicht unendlich groß ist (in dem Sinne, dass es dort zu jedem Ereignis E immer noch ein ihm zeitlich vorausgehendes gibt).

Beispiel einer Menge, die unendlich groß ist, aber doch nur endlichen Durchmesser hat, wäre die Menge aller reellen Zahlen, die größer als 0 und kleiner als 1 sind.
Warum sollte Zeit in unserem Sinne nicht diesem Zahlintervall entsprechen können und Zeit im Sinne der Schöpfung z.B. der Menge  a l l e r  reellen Zahlen?

 

  Beitrag 2073-12
Die mit Sicherheit sinnvollste Möglichkeit, den Fluss der Zeit zu definieren

 
 
Struktron in 2073-11:
Hallo alle miteinander,

Mich würde interessieren, ob man Ereignisse als etwas Grundsätzlicheres als Zeit ansehen könnte?


Hallo Struktron,

mich hat stets irritiert, dass manche Physiker "Ereignis" nennen, was Mathematiker unter einem "Punkt" der Raumzeit verstehen.

Aus meiner Sicht sollte man einen Punkt der Raumzeit nur dann ein Ereignis nennen, wenn dort Elementarteilchen entstehen, sich auslöschen oder miteinander kollidieren.
Hierfür aber scheint der Begriff Elementarereignis gebräuchlich zu sein (obgleich Versicherungsmathematiker und Stochastiker darunter ganz was anderes verstehen).


Kurz: Ein Elementarereignis im Raum ist ein an einer bestimmten Stelle des Raumes eintretendes  a t o m a r e s  Ereignis in dem Sinne, dass dort etwas passiert (oder passiert ist), was spontane, um diesen Punkt herum Wirkung zeigende Abänderung der Wellenfunktion des Universums bedeutet.

Elementarereignisse sind insofern grundsätzlicher als Zeit, als es Sinn macht, die Zeit — den Fluss der Zeit — zu sehen als

die transitive Hülle der einzigen zweistelligen, gerichteten Relation » früher als « auf der Menge aller Elementarereignisse, so dass:

" E1 früher als E2 " genau dann, wenn E2 eine aus E1 kommende Potentialwelle zerstört oder modifiziert.


Mit anderen Worten:
  • Vergangenheit von Ereignis E ist alles, was direkt oder indirekt zu E geführt hat.
  • Zukunft von E ist jedes Ereignis, in dessen Vergangenheit E liegt.

Sie hierzu auch Zum Wesen der Zeit.


 
Struktron in 2073-11:
 
Nach der üblichen Interpretation des Mainstreams sind Ereignisse Punkte der Raumzeit.
Das wäre eine dichte Menge, die überabzählbar wäre (die Mathematiker hier sollten das am besten formulieren können).

Handelt es sich bei den elementarsten Ereignissen dagegen um eine diskrete abzählbare Menge von Ereignissen, welche beispielsweise durch Stöße diskreter Elemente definiert werden, könnte (bzw. müsste) man wohl die Zeit damit definieren?


Ja, man könnte über diese elementarsten Ereignisse die Zeit sinnvoll definieren (wie oben erklärt).

Gruß, grtgrt
 

  Beitrag 2073-10
Unser Gehirn — auch nur ein Computer?

 
 
Harti in 2073-9:
Grtgrt in 2073-8:
 
Auf jeden Fall ist Zeit nichts, mit dem irgend ein Beobachter interagieren könnte. Zeit ist, in welchem Kontext auch immer, ein Abstraktum.

Hallo Grtgrt,

könnte man die Vorstellung (den Begriff) "Zeit" auch als ein Gedankenkonstrukt bezeichnen ?
Oder in Anlehnung an einen Computer unser Gehirn als Hardware und die Vorstellung von Zeit als Software ?

Unsere Vorstellung (unser Gefühl von) Zeit wäre dann ein Programm unseres Gehirns, mit dem wir die Veränderungen in der Natur erfassen. Ein anderes Programm wäre die Vorstellung von Raum.
 


Nun, Harti,

so habe ich es noch nie gesehen — dennoch scheint mir das durchaus Sinn zu machen, denn:
  • ein Abstraktum ist etwas durch Gedanken Konstruiertes (ein Gedankenkonstrukt),
  • und unser Gehirn scheint mir schon mit einen Computer vergleichbar:
    • Gehirnmasse ist ganz klar Hardware.
       
    • Als seine Software würde ich darin gespeicherte Erfahrungen sehen,
       
    • wobei gedankliche Konstrukte — als Denkergebnisse — zunächst mal Rechenergebnissen entsprechen (die man natürlich selbst wieder als Erfahrung nutzen kann. Und tatsächlich: Auch bei einem normalen Computer kann Software ja in beiden Rollen auftreten: erst als Rechenergebnis eines Programmgenerators, später dann dann als Programm, welches etwas errechnet).

Denken und Rechnen könnten so tatsächlich von gleicher Qualität sein — wenn da nicht noch Kreativität wäre, von der wir nicht wissen, wie sie in unserem Kopf zustande kommt. Bis das geklärt ist, muss man wohl annehmen:

Denken   =   Rechnen (Regelabarbeitung)   +   Zulassen von Ergebnis-Mutation (Kreativität)

 
Gruß, grtgrt
 

  Beitrag 2068-21
Was ist Zeit denn nun eigentlich wirklich?

 
Hallo Horst,

Horst in 2068-20:
Die Frage ist nach wie vor, anhand welchen Vorgänge im Raum kann man behaupten oder gar beweisen, dass der Raum "Zeit" enthält und wie auch immer mit ihm "verbunden" ist, wenn doch niemand weiß, was "Zeit" denn ist ?

Für mich persönlich ist Zeit nicht im Raum, somit enthält er auch keine Zeit.
Dass niemand das Wesen der Zeit erklären kann, heißt nicht, dass es sie nicht gibt. Typischer Fehlschluss, mit dem du oft argumentierst: "Was ich nicht sinnvoll erklären kann, existiert nicht."
Für mich ist die Kausalität in der Makrowelt (die Dinge eines Prozesses passieren stets nacheinander) ein empirischer Beweis für die Zeit.

Du kannst die Zeit "Dauer und Anzahl der Bewegungen" nennen, oder auch "Peter", das beeinflusst jedoch nicht die Tatsache, dass du auf meinen Beitrag erst nachdem ich ihn hineingestellt habe, Bezug nehmen kannst.

Gruß
 

  Beitrag 1209-9
Zur Idee einer nie versagenden Uhr: John Barrows Krümmungsuhr — kann sie definieren, was Zeit ist?

 
Bernhard Kletzenbauer in 1209-6:
In einem begrenzten Universum ist diese Aussage falsch. Das Licht, das ein Teleskop in 10 Mrd. Lj. Entfernung von der Erde auffängt, ist dann erst 3,7 Mrd. Jahre alt (mal die Undurchsichtigkeitsphase und die kosmische Expansion außer acht lassend). Denn vom Zeitpunkt "Urknall" ist es ja erst 3,7 Mrd. Jahre unterwegs. Das gilt für den Fall, daß das Ur-Licht niemals erlosch. .

Hallo Bernhard,

auch in einem endlichen Universum ist jeder Punkt im Universum gleich alt. Das kann man sich anhand des Ballon-Modells durch die nachstehende Grafik veranschaulichen:


Das in sich selbst zurückgekrümmte Universum besitzt an jeder Stelle den gleichen Krümmungsradius R(t). In jedem der eingezeichneten Galaxien A, B, C, D besitzt der Weltraum den gleichen globalen Krümmungsradius R(t). Deshalb spricht man in der Literatur auch von einer "Krümmungsuhr".

Krümmungsuhr heißt, die Größe des globalen Krümmungsradius R(t) ist ein Maß für die verflossene Zeit t von Urknall bis heute.

Ganz gleich, wo man auch das Teleskop "hinbeamt", dem Urknall kommt man nicht näher. Der Urknall fand an jeder Stelle des Universums gleichzeitig statt.

M.f.G. Eugen Bauhof
 

  Beitrag 2068-62
Was es bedeutet, die Zeit zu beobachten

 
 
Horst in 2068-59:
Stüps im Beitrag 2068-58

Zitat:
Wenn du Bewegung beobachtest, beobachtest du auch die Zeit.

Nee Stüps, das kann doch nicht wahr sein, du kannst Zeit beobachten?

Außerdem ignorierst du ... Beitrag 2073-8.
Demnach interagiert Zeit nicht mit einem Beobachter – wieso kannst du Zeit trotzdem beobachten?
Hat sich da Grtgrt geirrt?

Gruß Horst


Zeit ist ein Abstraktum. Einen Aspekt davon — das Voranschreiten der Zeit — bildet der Mensch sich ab z.B. auf den Zeigerstand einer Uhr.

Dass er sich so wirklich nur  e i n e n  Aspekt der Zeit in ein Bild verwandelt hat, zudem noch in ein oft recht unvollkommenes, erkennt man z.B. daraus, dass dieses Bild die Zeit aus seiner persönlichen, relativen Sicht zeigt. Dieses Bild der Zeit kann sich — der SRT wegen, aber z.B. auch dann, wenn die Uhr falsch gestellt wurde oder zu langsam geht — gravierend vom Bild anderer unterscheiden.

Mit anderen Worten: Nicht die Zeit selbst, aber Analogien dazu kann man beobachten.

Mathematiker würden sagen: Was man beobachtet, sind homomorphe Bilder der Zeit (deren mögliche Unvollkommenheit mal ignoriert).

 

  Beitrag 2068-36
Das Multiversum, in dem wir zu Hause sind (bzw: Raumzeit in dreierlei Bedeutung)

 
 
E... in 2068-33:
Grtgrt in 2068-31:
 
... Die Vermischung passiert nicht im Konstrukt "Raumzeit", sondern erst beim Übergang in ein anderes Exemplar davon.
 
Guten Morgen Grtgrt.

Wieviel Exemplare "Raumzeit" bist Du denn bereit zu bieten, in diesem, unseren Universum?

Mit einem kann ich Dir dienen, aber wo willst Du eine weitere "Raumzeit" herholen in die "übergegangen" werden kann?

Mit gespannten Grüßen.
E....


Hallo E...,

man sollte sich darüber klar sein, dass, wer von der Raumzeit spricht, von 3 ganz verschiedenen Dingen sprechen kann. Dies sind
    (1)   die Raumzeit als mathematisches Modell dessen, was wir unser Universum nennen,
    (2)   die Raumzeit als das, was unser Universum tatsächlich ist, und
    (3)   der jeweils beobachterspezifischen Sicht auf dieses Universum modelliert durch ein Exemplar von Typ (1).

Keine zwei Exemplare vom Typ (3) sind identisch.

Sie sind vergleichbar mit der Sicht eines Menschen, der sich in einem Segelboot weit draußen auf dem Meer herumtreibt in einer Gegend, wo — fern am Horizont — gerade noch einige Inseln zu sehen sind. Der Erdkrümmung wegen wird er i.A. nicht alle sehen, und wenn er sich ihnen nähert, wird die Zahl derer, die er zu sehen bekommt, immer größer. Kurz: Die ihm sichtbare Welt wird anders aussehen je nachdem, an welchem Ort er sich gerade befindet.

Dennoch ist unbestritten: Diese Sichten sind nicht disjunkt zueinander. Sie sind wie Karten, deren Inhalt sich überlappt aber doch nicht identisch ist.

Wenn ich also von mehreren Exemplaren der "Raumzeit" spreche, meine ich damit ortsabhängige Sichten (3) auf unser Universum modelliert über je ein Exemplar (1).
Jede ist zwangsläufig begrenzt durch den Zeithorizont eines Beobachters, der sich am jeweils betrachteten Ort befindet — ist also ein kugelförmiges, begrenztes Universum.

Die Vereinigung aller Universen in diesem Sinne nenne ich das Multiversum, in dem wir zu Hause sind.

Es ist Vereinigung sich überlappender oder nicht überlappender kugelförmiger Teil-Universen, die — wenn beobachterspezifisch definiert — sich zudem noch ständig gegeneinander verschieben, wenn jene Beobachter sich gegeneinander bewegen.

Wir, als ein solcher Beobachter, bewegen uns wenigstens durch die Zeit, und deswegen modifiziert sich unser Universum ständig (und nicht nur deswegen, weil der Raum expandiert).

Gruß, grtgrt
 

  Beitrag 2068-42
Was genau ist die beobachterspezifische Zeitskala?

 
 
Henry in 2068-37:
 
Natürlich sieht gerade ein Photon aus SEINER Sicht, das es für eine Strecke von 300000 km eine Sekunde braucht, ...


Da das Licht keine Zeit kennt, kennt es auch keine Sekunde ....


Henry in 2068-37:
 
Der "jemand, der nicht das Proton selbst ist" hingegen sieht für das Photon ( gemessen mit einer Uhr auf dem Photon! ) die Zeit still stehen.


Ja: Was die Uhr des Photons zeigt, ist die Sicht des Photons.

Die beobachterspezifische Sicht ist — per definitionem — stets die, die der Beobachter von seiner eigenen Uhr abliest.
Und das muss auch so sein, da sie sonst ja gar nicht eindeutig wäre (sie würde abhängig vom Objekt, auf dessen Uhr er blickt).

 

  Beitrag 2068-50
Wie der Beobachter den Zeigerstand einer relativ zu ihm bewegten Uhr errechnet

 
 
Henry in 2068-49:
 
Dein Zitat: » Die beobachterspezifische Sicht ist — per definitionem — stets die, die der Beobachter von seiner eigenen Uhr abliest.
Und das muss auch so sein, da sie sonst ja gar nicht eindeutig wäre (sie würde abhängig vom Objekt, auf dessen Uhr er blickt).
«

Lassen wir das Extrem des Photons selbst mal beiseite, es geht hier auch gar nicht in erster Linie um das Photon, sondern darum, dass es bzgl. der Zeitdilatation immer um die Sicht der Uhr aus einem System heraus in ein anderes geht, DAS stellt der jeweilige Beobachter fest und es geht NICHT um die eigene Uhr des jeweiligen Beobachters, sondern darum, welche Zeit er auf der Uhr des jeweils anderen System abliest.

Das war der Grund, warum ich den Link eingestellt habe.


Du scheinst mir ausweichen zu wollen, Henry, denn:

  • Es ging durchaus um das Extrem des Photons.
  • Dennoch gebe ich Dir recht: Es geht auch darum, wie denn nun genau die Wahrnehmungen von Beobachter (einerseits) und beobachtetem Objekt (andererseits) sich unterscheiden.
    Hierzu aber gilt:
    • Der Beobachter kann die Uhr des beobachteten Objekt gar nicht einsehen.
       
    • Die SRT allerdings sagt ihm, wie er deren Zeigerstand herrechnen kann aus dem Zeigerstand seiner eigenen Uhr und der durch ihn beobachteten Geschwindigkeit des Objekts.
       
    • Die entsprechende Regel lautet:
       
      Eine aus Sicht des Beobachters mit Geschwindigkeit v bewegte Uhr geht – aus seiner Sicht – um den Faktor  (1 – v2/c2)–1/2  langsamer als seine eigene.


Mit besten Grüßen,
grtgrt
 

  Beitrag 2080-24
Was ist eine Sekunde?

 
 
Horst in 2080-21:
 
Aber mit dem Urknall ist ja - Gott sei Dank - die Zeit mit entstanden, da frag ich dich mal, nach welchem Takt läuft denn diese Zeit ab?

Das ist eine ernsthafte Frage!


Die offizielle Antwort darauf scheint immer noch die Sekundendefinition von 1967 sein:

Zitat:
 
Eine Sekunde ist das 9 192 631 770 -fache der Periodendauer der dem Übergang zwischen den beiden Hyperfeinstrukturniveaus des Grundzustandes von Atomen des Nuklids 133Cs entsprechenden Strahlung.

Nukleid 133Cs ist das nicht radioaktive Caesium-Isotop mit der Massenzahl 133.
 

 

  Beitrag 2080-37
Warum die Zeit vielleicht doch ein Teil der Natur sein könnte — also mehr als nur Abstraktion

 
 
Hallo Horst,

mir kommen Zweifel, ob meine sarkastische Bemerkung in Beitrag 2080-34 berechtigt war.

Zu wissen, wie spät es ist, bedeutet letztlich nichts anderes, als zu wissen, wo andere Taktgeber — seien es Uhren, die Sonne oder z.B. auch Züge — eben jetzt stehen.

Vielleicht sollte man die Zeit deswegen vor allem als etwas sehen, welches uns erlaubt, aus der Geschichte eines Objektes (z.B. meiner Uhr) auf die Geschichte anderer Objekte zu schließen. Auch in dem Sinne, dass man auf die Frage » Wie könnte Objekt X jetzt aussehen? « eine Antwort erhält, die i.A. wenigstens grob richtig sein wird (aber natürlich nicht unbedingt richtig sein muss — ein Zug etwa könnte ja inzwischen mit einem anderen zusammengestoßen sein).

Damit ist ein Taktgeber ein Mechanismus, der uns erlaubt, in gewissem Umfang wissen zu können, in welchem Zustand andere Objekte sich jetzt befinden unter der Voraussetzung, dass sie sich entwickelt konnten, wie "geplant" war (und diese Planung uns bekannt ist).

Genauer: Nicht der Taktgeber als solcher erlaubt uns das, sondern es ist die Natur, die ihn zwingt, synchron mit gleich gebauten anderen Objekten zu sein. Damit, so scheint mir jetzt, ergibt sich, dass die Zeit ein in die Natur eingebauter Synchronisationsmechanismus ist.


Die Zeit, so könnte man sagen, ist so eine Art Feldwebel, der die Kompanie physisch existierender Objekte versucht, im Gleichschritt zu halten.
Sein Gegenspieler sind quantenphysikalische Ereignisse, die absolut zufällig eintreten.

Gruß,
grtgrt
 

  Beitrag 2085-9
Die Zeit aus Sicht der Menschen ...

 
Grtgrt in 2085-8:
Mir scheint, dass das seitens Einstein nur eine flapsig dahingeworfene Verlegenheitsantwort war — es dürfte ihm nämlich durchaus bewusst gewesen sein, dass auch er nicht in der Lage war, den eigentlichen Ursprung dessen zu erklären, was der Mensch als » Zeit, die verstreicht « kennt.

Hallo Grtgrt,

das kann gut sein. Andererseits hat Einstein sinngemäß geäußert, dass Vergangenheit und Zukunft keine Bestandteile des wissenschaftlichen Zeitbegriffs sind. Eugen Bauhof wird das entsprechende Zitat sicherlich wörtlich mitteilen können.

Ich habe mir schon mal Gedanken darüber gemacht, wie unsere Vorstellung von Zeit im Sinn von Dauer einer Veränderung zustande kommt.

Um feststellen zu können, dass ein Vorgang V1 dauert, brauchen wir zumindest einen weiteren Vorgang V2, der währen des Vorgangs V1 stattfindet. Aus Erfahrung wissen wir, dass während des Vorgangs V1 unzählige weitere Vorgänge stattfinden. Daraus folgt unser allgemeines Zeitgefühl. Wenn wir den Vorgang V1 genau bestimmen wollen, verwenden wir als Vorgang V2 eine Uhr. Eine Uhr zeigt normierte Dauer ( Maßeinheiten von Dauer) in Form von Sekunden, Minuten etc. an. Der zugrundeliegende Vorgang einer Uhr ist nicht das Zeigersystem, sondern eine Schwingung, die z.B. durch ein Pendel, eine Unruh oder atomare periodische Vorgänge erzeugt wird und deren bestimmte Anzahl eine Zeiteinheit ausmacht. Periodische Bewegungsvorgänge sind zur Darstellung von Dauer besonders gut geeignet, weil die räumliche Veränderung, die eine Bewegung auch beinhaltet, nach Durchlaufen einer Periode Null ist. Deshalb haben die Menschen auch schon relativ früh periodische Vorgänge wie Erdumdrehung (Tag) Mondrotation (Monat) etc. zur Zeitbestimmung benutzt, auch wenn sie nicht wußten, wie die entsprechenden Bewegungsabläufe tatsächlich waren.

Unsere Vorstellung von Zeit kommt deshalb letztlich dadurch zustande, dass wir Vorgänge (Geschehensabläufe) zueinander in Beziehung setzen, miteinander vergleichen.
Dieser Vergleich hat sich verselbständigt und zu einem allgemeinen Gefühl entwickelt, dass Vorgänge nicht instantan erfolgen, sondern dauern. Wir meinen sogar, dass Zeit im Sinn von Dauer eine objektive, außerhalb unserer Vorstellung existierende Naturerscheinung ist. Dies sehe ich aus den dargelegten Gründen anders. Es gibt nicht nur dort, wo es keine Objekte/Veränderung gibt keine Zeit, sondern auch dort, wo es keine Menschen gibt. In welchem Umfang Tiere ein Zeitgefühl haben, lasse ich mal unberücksichtigt.

MfG
Harti
 

  Beitrag 2085-21
... und auch aus noch anderer Sicht

 
 
Horst in 2085-19:
 
Was oft beim Thema Zeit nicht bedacht wird ist doch der prinzipielle Unterschied zwischen der physikalisch/mathematischen Anwendung des Begriffes Zeit zu Messzwecken mit entsprechend willkürlich gewählten Maßeinheiten und die rein menschliche, also subjektive Empfindung dessen was man Zeit nennt.

Und diese Empfindung ist nach dem Tode ebenso verschwunden wie die einer Seele, und damit auch die Vorstellung von Zeit nach dem verschwinden der Menschheit.

Die Frage "Was ist Zeit"? kann man also beantworten: "Zeit ist nicht!"


Hallo Horst,

Du sprichst hier auch nach meiner Überzeugung einen ganz wichtigen Punkt an:


Es gibt wohl einen Unterschied zwischen gefühlter, subjektiver Zeit

und dem Zeitbegriff, von dem Einsteins Gleichungen sprechen (dem der Natur).



In Beitrag 2085-14 ist das schon angesprochen worden, doch habe ich dort noch nicht danach gefragt,
  • wie — und auch in welchem Ausmaß denn überhaupt — sich diese vielen individuellen Zeitbegriffe zueinander in Beziehung setzen lassen. Dieser Frage nachzugehen scheint mir wichtig, denn schließlich sind Uhren dazu da, die durch uns gefühlte Geschwindigkeit des Vergehens "unserer" Zeit immer wieder zu synchronisieren mit dem Voranschreiten der Zeit im Sinne der Natur.
  • Warum z.B. vergeht uns "unsere" (individuelle) Zeit umso schneller, je beschäftigter wir sind? Liegt es daran, dass wir uns, wenn wir beschäftigt sind, weniger oft mit der Stand der Zeit der Natur synchronisieren? Wenn ja, könnte das bedeuten, dass "unsere" Zeit ganz grundsätzlich eher langsamer vergeht als die der Natur? Könnte es sein, dass nur solche Synchronisation unsere individuelle Uhr überhaupt eine Schritt tun lässt? Wenn ja: Würde das nicht bedeuten, dass es eigentlich doch  n u r  die Zeit der Natur gibt?


Gruß, grtgrt

PS: Siehe auch Zum Rätsel der Zeit.
 

  Beitrag 2085-22
Ohne wohldefinierten Zeitbegriff keine wohldefinierten Weltlinien (Geodäten)

 
 
Bauhof in 2085-16:
Grtgrt in 2085-14:
 
Das Problem mit diesem Definitionsversuch scheint mir zu sein, dass die Begriffe » Weltlinie « und » Eigenzeit « den Begriff der Zeit ja schon voraussetzen, d.h. sie bauen auf ihm auf und können deswegen schon rein technisch  n i c h t  Teil seiner Definition sein.

Hallo Gebhard,

nur eine Frage: Inwiefern setzt der Begriff der Weltlinie den Begriff der Zeit voraus? Die Raumzeit ist zunächst mal nur ein abstraktes vierdimensionales Gebilde. Dieses Gebilde hätte man statt Raumzeit auch "Schlossgespenst" taufen können.

Die senkrecht auf dem Raum stehende vierte Dimension ist nicht die Zeit, sondern eine imaginäre Geschwindigkeit: V4 = i•c•t.


Nun, Eugen, schon in der Formel  V4 = i•c•t  tritt ja nun explizit die Zeit auf (das t steht für sie).

Auf den zweiten Blick aber sieht man, dass das Produkt  c•t  tatsächlich nur eine Länge ist — eine Länge allerdings, die nur als wohldefiniert gesehen werden kann, unter der Voraussetzung, dass man die Lichtgewschwindigkeit c kennt: eine  G e s c h w i n d i g k e i t  also — und somit etwas, das zu definieren ohne einen schon bekannten Zeitbegriff gar nicht erst möglich ist.

Die Weltlinien schließlich sind genau die Wege durch die Raumzeit, die Objekte nehmen, die keinerlei Beschleunigung ausgesetzt sind.  B e s c h l e u n i g u n g  aber ist die zweite Ableitung von Ortsveränderung nach Zeit, und so kann auch von Weltlinien (basierend auf Abwesenheit von Beschleunigung) erst derjenige sinnvoll sprechen, der die Zeit als schon definiert sieht.

Das also war mit meiner Bemerkung gemeint.


Gruß, grtgrt
 


 

  Beitrag 2085-39
Die Zeit als Phänomen gesehen

 
 
Stueps in 2085-37:
 
... Meines Erachtens machst du im Denken einen ganz entscheidenden Fehler:

Du versuchst, Zeit als ein Ding zu postulieren (in deiner Antithese). Dies kann m.E. natürlich nicht funktionieren.

Ich fasse Zeit als Phänomen auf. Nun kann man sich fragen: SInd Phänomene real? Ich sage: Ja klar! Ich möchte es mit einem Beispiel versuchen:

Altern ist kein Ding an sich. Ich könnte nämlich im "Horst-Stil" fragen: Aha, Altern existiert also real? Dann zeige mir doch "Altern an sich"! Sage mir, wieviel es wiegt, wo ich es erwerben und anfassen kann! Wie sieht "Altern" denn konkret aus, welche Farbe, Größe und Form hat denn das real existierende "Altern"? Wie wechsel­wirkt "Altern" denn mit Elektronen oder Quarks?

Du siehst also:
    1. Jeder wird das Altern als Ding an sich leugnen.
    2. Niemand wird jedoch das Altern als real existierendes Phänomen leugnen.
     
    3. Jeder wird Zeit als Ding an sich leugnen.
    4. Niemand wird jedoch Zeit als real existierendes Phänomen leugnen.

Es wird meines Erachtens also von den meisten Leuten, die Zeit nicht als real existierend betrachten, nicht zwischen Ding und Phänomen/Prozess unterschieden. Und das führt natürlich zu den unterschiedlichsten Meinungen und Missverständnissen untereinander.
 


Hi Stueps,

durch diese deine Erklärung wird jetzt klar, was mich an der ganzen Diskussion um die Natur der Zeit bislang (mir selbst kaum bewusst) irgendwie gestört hat:

Die Zeit im Sinne der Natur
  • als » Phänomen « zu sehen
  • statt als etwas, das sich ähnlich verhalten und bemerkbar machen könnte wie z.B. Kraftpotential,
— darauf war ich bisher einfach nicht gekommen.


Danke,
mit besten Grüßen,
grtgrt

PS: Man müsste sich jetzt natürlich fragen, was Phänomene verschiedener Art von unterschiedlicher Natur sein lässt und wie solcher Unterschied gedanklich greifbar zu machen ist. Denn mit Sicherheit kann ja auch Kraftpotential, ja sogar alles, was wir beobachten können, als Phänomen eingestuft werden.

 

  Beitrag 2085-191
Die Zeit als etwas, das man nicht anhalten kann

 
 
Hans-m in 2085-189:
 
Zitat von Quante:
Insofern existiert Zeit, aber sie existiert nicht als "eigenständiges Ding Zeit".

Richtig, Zeit existiert nicht als eigenständiges Ding, sondern zusammen mit dem Raum, das nennt man Raumzeit.


Ich finde es gar nicht so offensichtlich, dass man das Phänomen

» Zeit, die man nicht aufhalten kann «


tatsächlich so ohne weiteres mit der 4-ten Dimension des mathematischen Objekts, welches wir » die Raumzeit der ART « nennen, identifizieren kann.

Die Tatsache, dass Einsteins Feldgleichungen das Verhalten des Inhalts unseres Universums auf Basis dieses mathematischen Modells so verdammt gut vorhersagen, bedeutet ja noch keineswegs, dass es nicht noch eine völlig andere, viel genauere Basis geben könnte und die vielleicht gar keine wohldefinierten Dimensionen (im ganz Kleinen) kennt, da sie — wie nicht nur die Vertreter des Modells der Schleifenquantengravitation ja schon annehmen — netzartig gestrickt sein könnte.

Dass Einsteins Modell irgendwann durch ein genaueres ersetzt werden muss, beweist allein schon die Tatsache, dass die Raumzeit der ART Singularitäten kennt — Stellen also, in denen ein oder mehrere physikalische Größen — glaubt man dem Raumzeit-Modell der ART — unendlich großen Wert haben müssten, obgleich das ja nun definitiv nicht der Fall sein kann (!).

 

  Beitrag 2085-106
Hawkings imaginäre Zeit — eine Versionierung der Raumzeit

 
C... in 2085-94:
Hallo, Bauhof,

Bauhof in 2085-90:
Wenn sich die Oberfläche der 5-D-Kugel aus 3 Dimensionen des Raumes und einer imaginären Dimension (imaginäre Zeit) zusammensetzt, wo kann man dann die reelle Zeit der klassischen vierdimensionalen Raumzeit verorten?

Ich denke, Hawking hat die imaginäre Zeit als Alternative zur reellen Zeit konzipiert, weil erstere die Realität besser beschreibt (in dem Sinne, dass sich damit Unendlichkeiten/ Singularitäten vermeiden lassen). Genau das beschreibt Hawking auf S. 177, 2. Absatz, seines Buchs, wobei er die "Realität" der imaginären Zeit zwar vermutet, sich aber nicht zu einer entsprechenden Behauptung hinreißen lässt, da jedes mathematische Modell letztlich ja nur Phänomene beschreibt.

Falls du die reelle Zeit dennoch in diesem Modell "verorten" möchtest:

Wie wäre es, wenn du die 5. Dimension der Kugel (deren Durchmesser) als die reelle Zeit betrachtetest?
Dabei wären nämlich folgende Voraussetzungen erfüllt:

- Die reelle Zeit stünde senkrecht auf der imaginären Zeit.
- Die reelle Zeit wäre eine Strecke endlicher Länge (sie hätte einen Anfang und -zumindest im geschlossenen Universum- auch ein Ende).

In diesem Sinne entsprächen der Schar aller mögliche Durchmesser der Kugel die möglichen reellen Zeitverläufe, entsprechend der möglichen Bewegungszustände im 3-D-Raum(!).

Hallo C...,

gehen wir mal davon aus, das du recht hast mit deinem Hinweis, dass die Breitenkreise auf der 5-D-Kugel das dreidimensionale Universum repräsentieren.

Und dass die reelle Zeit in Hawkings Modell senkrecht auf der imaginären Zeit steht, dem kann ich auch zustimmen. Denn das schreibt auch Stephen Hawking in seinem Buch [1]:

Zitat:
Doch wie ich in Kapitel 2 gezeigt habe, gibt es noch eine andere Art der Zeit, die imaginäre Zeit, die rechtwinklig zur gewöhnlichen Zeit verläuft – zu jener Zeit, die wir verstreichen fühlen. Die Geschichte des Universums in der reellen Zeit bestimmt seine Geschichte in der imaginären Zeit und umgekehrt, aber die beiden Arten von Geschichte können sehr verschieden sein. Insbesondere muss das Universum in der imaginären Zeit weder einen Anfang noch ein Ende haben. Die imaginäre Zeit verhält sich einfach wie eine weitere Raumrichtung. Daher kann man sich die Geschichten des Universums in der imaginären Zeit als gekrümmte Flächen wie Kugeln, Ebenen oder Sattelflächen vorstellen, nur mit vier Dimensionen anstelle von zweien.

Das heißt, die reelle Zeit stünde senkrecht auf der imaginären Zeit und damit gleichermaßen senkrecht auf der 5-D-Kugel. Der Radius der 5-D-Kugel aber ändert sich nicht, weder mit der reellen noch mit der imaginären Zeit.

Es ändert sich zwar nicht der Radius der 5-D-Kugel, aber der vierdimensionale Radius des 3-D-Universums ändert sich. Damit passieren zwei Dinge:

1. Der Umfang des 3-D-Universums wird größer (was ja durch die Universum-Expansion beobachtet wird.)

2. Durch die Vergrößerung des vierdimensionalen Radius des 3-D-Universums (der reellen Zeitdimension) wird neue Zeit "generiert", Zeit entsteht. Oder in deiner Sprechweise: Fortschreiten in der Zeit.

Soweit damit einverstanden?

M.f.G. Eugen Bauhof

[1] Hawking, Stephen
Das Universum in der Nußschale.
Hamburg 2001
ISBN=3-455-09345-0
 

  Beitrag 2090-3
Warum Greensnipers These zur Zeit jeder Grundlage entbehrt

 
 

Mein Argument gegen Greensnipers These, Zeit sei die Eigenschaft eines Sinnfeldes



Greensniper in 2090-1:
 
Zeit ist eine Eigenschaft eines Sinnfeldes, eines Systems.

    Um das zu verstehen fange ich mit einem simplen Beispiel an. Schauen wir uns eine Sonne mit einem darum kreisenden Planeten an. Die Zeit, die der Planet braucht um die Sonne zu kreisen, definieren wir ein "Jahr". Fertig. Mehr brauchen wir in diesem System nicht. Wenn es keinen Beobachter gibt, ist das auch ziemlich egal, wie lange so ein Jahr dauert.
    Aber nun stellen wir uns vor, auf dem Planeten leben denkende und fühlende Wesen. Auch für sie ist die Länge dieses Jahres erst einmal ziemlich egal, denn bisher haben wir nur eine einfache Welt gedanklich erschaffen. Aber denken wir weiter: Der Planet hat eine Neigung zur Sonne und so gibt es 4 Jahreszeiten, die Felderträge auf dem Planeten schwanken. Die Bewohner können nur eine gewisse Zeit lang ohne Felderträge überleben. Und damit kommt hier die zweite Welt, das zweite Sinnfeld ins Spiel, nämlich die Dauer der Zeit, die diese Wesen ohne Nahrung auskommen können. Wir haben nun zwei Sinnfelder, die sich überlappen: Die Umlaufzeit des Planeten und die Hungerzeit der Bewohner.
    Hat der Planet auch eine Eigendrehung, kommt noch die Tageszeit hinzu. Gibt es einen Mond mit Gezeiten, kommt der Monat hinzu.

All diese Zeitwelten (Sinnfelder) überlagern und beeinflussen sich teilweise. Aber es gibt trotzdem in diesem Szenario keine absolute Zeit.

Nun mag man anmerken, dass die Welt komplexer ist. Es gibt Evolution, Zerfall, Entropiezunahme,... Es gab wohl einen Beginn.
Dennoch muss man das alles in einem eigenem Kontext mit einer jeweils eigenen Zeit betrachten. Wir können nicht einfach den Urknall als Anfang "aller Zeit" betrachten, sondern nur der des Universums. Aber der hat nichts mit unserem Alltag zu tun.

Was hieße denn "Alle Zeit"? Was sind denn diese angeblichen 14 Milliarden Jahre?


Hallo Greensniper,

deine 3 Beispiele beschreiben durch  T a k t g e b e r  kreierte Zeitbegriffe. Allein schon die Tatsache, dass sie sich ineinander umrechnen lassen, zeigt ganz klar, dass Zeit etwas Grundlegenderes sein muss, als nur » das, was man von der Uhr abliest « (wie Einstein mal flapsig sagte).

Die Möglichkeit der Umrechnung zeigt: Dein Szenario  k e n n t  absolute Zeit.

Dass es sie dennoch nicht gibt — wie die ART uns zeigt — steht auf einem anderen Blatt. Dein Szenario jedenfalls ist zu einfach, die Nichtexistenz absoluter Zeit zu beweisen.


Gruß, grtgrt
 

  Beitrag 2090-4
Entsteht die Zeit, da kein unteilbares Ereignis Mitursache seiner selbst sein kann?

 
 
Greensniper in 2090-1:
 
Zeit ist der Abstand zwischen zwei sich wiederholenden Ereignissen.

Solange die Wiederholungen konstant sind, wie z.B. einer Schwingung eines Atoms, dürfte hier jeder mitgehen.
Schwierig wird es, wenn sich die Wiederholungen ändern, z.B. wenn ein Planet seine Bahn ändert und somit die Umlaufintervalle.
Und da wir verschiedene Systeme haben muss man die Zeiten miteinander vergleichen. Wie verschiedene (teilweise schwankende) Währungen werden nun Jahre mit Monaten mit Tagen mit Atomschwingungen verglichen. Aber wer sagt denn, dass Atome immer gleich stark schwingen? Und was ist mit der Relativitätstheorie? Immerhin hängt die Zeit ja auch von der Geschwindigkeit im All ab. Aber Geschwindigkeit ist immer relativ zu anderen Objekten zu sehen.


So allgemein kann man das auf keinen Fall sagen: Das Ereignis etwa


» Ich merke, dass mein Zug Verspätung hat «


wiederholt sich oft, definiert aber ganz sicher keinen Zeitbegriff — und schon gar nicht das Wesen der Zeit oder gar die Zeit selbst.


Dass relativ zueinander bewegte Beobachter zeitliche Abstände als verschieden lang wahrnehmen (was richtig ist), bedeutet noch lange nicht, dass diese unterschied­lichen Sichten auf vergehende Zeit die Zeit selbst — wenn sie denn überhaupt etwas Konkreteres als nur eine beobachterspezifische Wahrnehmung sein sollte — irgendwie beinflusst.

Wir können sicher nur sagen: Zeit muss eine Folge der Tatsache sein, dass auf der Menge aller unteilbaren Ereignisse eine transitive, aber nirgendwo symmetrische Relation

Ereignis X ist Ursache oder Mitursache für Ereignis Y

existiert. Meine Vermutung ist:


Zeit entsteht, da kein unteilbares Ereignis sich selbst zur Ursache — oder auch nur Mitursache — haben kann.



Wer, ausgehend von einem Ereignis A, zu einem Ereignis B kommt, wird auf  s e i n e m  Weg dorthin gewissen Wirkungsquanten ausgesetzt. Die Summe all dieser kleinsten Wirkungen ist, was er als seine Eigenzeit empfindet. Dies gilt für Lebewesen ebenso wie für unbelebte Objekte.

Zeit könnte ein Netz von Zuständen sein, deren jeder durch genau ein atomares Ereignis geschaffen und durch genau ein anderes atomares Ereignis zerstört wird. Von der Dauer eines solchen Zustandes zu sprechen, würde dann keinerlei Sinn machen.

Was immer auch die kleinsten Bausteine unseres Universums sein mögen: Jeder könnte genau ein solcher Zustand sein (bzw. ihn darstellen).

 

  Beitrag 2090-5
Zeitgefühl — eine Art Uhr

 
 
Harti in 2090-2:
 
Zeitgefühl, ein allgemeines Wissen, dass dauernd zahlreiche Vorgänge stattfinden.


Dieser Aussage kann ich voll zustimmen.

Es freut mich, dass sie zudem klar macht: Zeitgefühl ist was ganz anderes als die Zeit selbst.

Zeitgefühl ist eine uns von der Natur mitgegebene Uhr ohne jede Garantie bestimmter Genauigkeit.

 

  Beitrag 1708-102
Zweiter Hauptsatz der Thermodynamik: Warum schätzt er die Vergangenheit falsch ein?

 
 
Bauhof in 1708-100:
Hans-m in 1708-99:
Was mich an dem Experiment interssiert ist, ob nach dem Energieerhaltungssatz ein endloser Kreislauf möglich wäre? rein hypothetisch.

Hallo Hans-m,

Der Energieerhaltungssatz ist der Erste Hauptsatz der Thermodynamik. Er besagt nur, dass Energie nicht erzeugt, sondern nur umgewandelt werden kann. Er sagt aber nichts darüber, für welche Richtungen diese Energie-Umwandlungen erlaubt sind.

Diese Einschränkung der Richtung beschreibt der Zweite Hauptsatz der Thermodynamik. Er besagt, dass z.B. Wärme nur vom heißen Ort zum kalten Ort fließt und nicht umgekehrt. Deshalb ist kein endloser Kreislauf möglich.


Hallo Hans-m,

was Eugen Bauhof dir hier sagt, ist nur grob richtig:

Wie Brian Greene im Abschnitt "Entropy: Past and Future" seines Buches "The Fabric of the Cosmos" klarstellt, impliziert der Zweite Hauptsatz der Thermodynamik keineswegs die absolute  S i c h e r h e i t , sondern nur eine über alle Maßen große  W a h r s c h e i n l i c h k e i t  dafür, dass Wärme nur vom heißen Ort zum kalten Ort fließt und nicht umgekehrt.

Greene weist zudem noch auf die merkwürdige Tatsache hin, dass der Zweite Hauptsatz der Thermodynamik für die Vergangenheit als besonders wahrscheinlich erachtet, was tatsächlich so gut wie  n i e  wirklich geschah:


Zitat von Brian Greene, p. 161:
 
As it's usually described, the second law of thermodynamics implies that entropy increases toward the future of any given moment. [ But: ]

Since the known laws of nature treat forward and backward in time identically, the second law actually implies that entropy increases both towards the future  a n d  toward the past from any given moment.

That's the essential lesson. It tells us that the entropic arrow of time is double-headed. From any specific moment, the arrow of entropy increase points toward the future  a n d  toward the past. And that makes it decidedly awkward to propose entropy as the explanation of the one-way arrow of experiential time.

Think about what the double-headed arrow of time implies in concrete terms: If it's a warm day and you see partially melted ice cibes in a glass of water, you have full confidence that half an hour later the cubes will be more melted, since the more melted they are, the more entropy they have. But [according to the second law of thermodynamics] you should have  e x a c t l y  the same confidence that half an hour  e a r l i e r  they were also  m o r e  melted, since exactly the same statistical reasoning implies that entropy should increase toward the past. ...

The troubling thing ist that half of these conclusions seem to be flatout wrong. Entropic reasoning yields accurate and sensible conclusions when applied in one time direction, toward what we call the future, but gives apparantly inaccurate and seemingly ridiculous conclusions when applied to what we call the past.
 


Gruß, grtgrt
 

 Beitrag 0-453
Wie extrem genau Uhren die Relativität der Zeit bestätigen

 
 

 
Wie extrem genaue Atomuhren

die durch SRT und ART vorausgesagte Zeitdilatation bestätigen

 
 
In einem der Bücher des theoretischen Physikers Jörg Resag liest man:
 


Jörg Resag (2012):
 
Eine sich bewegende Uhr läuft für einen ruhenden Beobachter langsamer als eine ruhende Uhr (spezielle Relativitätstheorie). Ebenso laufen ruhende Uhren in einem statischen Gravitationsfeld umso langsamer, je weiter unten sie sich befinden (allgemeine Relativitätstheorie).
 
Mittlerweile gibt es Uhren, die so präzise sind, dass sich diese Effekte auch bei alltäglichen Geschwindigkeiten und Gravitationsfeldern messen lassen.
 
So haben James Chin-Wen Chou, Dave Wineland und Kollegen am National Institute of Standards and Technology (NIST) in Boulder (Colorado) Uhren verwendet, die auf nur einem einzigen Aluminium-Ion in einer sogenannten Paul-Falle basieren und die in 3,7 Milliarden Jahren nur eine Sekunde falsch gehen.
    Einfach unglaublich, dass man heute schon Materie derart präzise kontrollieren und manipulieren kann!
     
    Das Aluminium-Ion wird dabei in einem elektromagnetischen Feld festgehalten und mit Lasern gekühlt. Ein anderer Laser feuert nun auf das Ion,
    wobei seine Frequenz sehr präzise auf eine Absorptionsfrequenz des Ions abgestimmt wird. Diese Frequenz ist nun der Taktgeber der Uhr. Bei dieser Frequenzabstimmung spielt ein weiteres Ion (ein Magnesium- oder Beryllium-Ion) eine Rolle, dessen Quantenzustand mit dem des Aluminium-Ions verschränkt ist (ähnlich wie die beiden Spin-1/2-Teilchen im Einstein-Podolsky-Rosen-Experiment miteinander verschränkt sind, siehe Kapitel 2.8).

Lässt man nun das Ion bei einer Durchschnittsgeschwindigkeit von nur 10 m/s (= 35 km/h) hin- und heroszillieren, so beobachtet man eine Verlangsamung der darauf basierenden Uhr um etwa den Faktor 10-16.
 
Auch bei anderen Geschwindigkeiten entspricht die Zeitdilatation genau dem Wert, wie ihn die spezielle Relativitätstheorie vorhersagt.
 
Ebenso gelingt es, den Zeitunterschied zweier solcher Uhren im Gravitationsfeld der Erde nachzuweisen, die nur etwa 17 cm Höhenunterschied aufweisen. Die untere Uhr läuft dabei um etwa den Faktor 4 × 10-17 langsamer als die obere — das entspricht grob einer zehnmillionstel Sekunde in 80 Jahren.
 
 
Mehr dazu in: Relativity with a human touch sowie Handwerkszeug Relativität, Physik Journal 9 (2010) Nr. 11, S. 16.

 


 
Quelle: Jörg Resag: Die Entdeckung des Unteilbaren, Kap 7


 

 Beitrag 0-212
Bewegung ist nur teilweise relativ

 
 

 
Unterschiedlich relative Bewegungen

 


Rüdiger Vaas (in seinem Buch Jenseits von Einsteins Universum, 2016, S. 192 u. 209):
 
Im Gegensatz zu Einsteins Ausgangspunkt und langjähriger festen Überzeugung erwiesen sich gleichförmige und beschleunigte Bewegung  n i c h t  als gleichermaßen relativ:
     
  • Mit welcher Geschwindigkeit — sich zwei Körper bewegen (und ob sie sich überhaupt bewegen), lässt sich immer nur im Hinblick auf ein bestimmtes, aber beliebiges Bezugssystem sagen.
     
  • Nur ob und wie stark eine Objekt aber beschleunigt ist, steht fest unabhängig von jeder solchen Perspektive.

 
Das bedeutet (z.B.): Gäbe es nur ein Raumschiff im ansonsten leeren Weltraum,
     
  • so könnten seine Bewohner nicht sagen, wohin und wie schnell sie sich bewegen und ob überhaupt.
     
  • Beschleunigen sie aber, so macht sich das ihnen bemerkbar durch dann auftretende Trägheitskräfte — sie sind  n i c h t  relativ.

 



 

 Beitrag 0-6
Beobachtete Geschwindigkeiten kleiner c (im Sinne der SRT) sind niemals wirkliche Geschwindigkeit

 
 

 
Beobachtete Geschwindigkeiten sind stets nur Scheingeschwindigkeit

 
 
Mit vB( X ) bezeichnen wir die Geschwindigkeit, mit der sich ein Objekt X relativ zu einem Beobachter B bewegt.
 
Mindestens im Rahmen Spezieller Relativitätstheorie ist dann stets
 
vB( X )   =   vX( B )

 
Sind nun aber B1 und B2 relativ zueinander bewegte Beobachter ein und desselben Objektes X, so haben vB1( X ) und vB2( X ) meist unterschiedlich hohen Wert.
Aus dieser Tatsache folgt ganz klar, dass
 
 
beides stets nur  S c h e i n g e s c h w i n d i g k e i t e n  sind.
 
 
 
Weil das so ist, kann vB( X )   in Ausnahmefällen auch größer als c  sein ( Beispiele dafür diskutiert der Physiker Domenico Guilini ).


 

  Beitrag 2103-3
Beispiel einer Pseudobewegung (einer nur  v i r t u e l l e n  Bewegung)

 
 
Harti in 2103-2:
 
Auch ich kann mir Überlichtgeschwindigkeiten vorstellen, der keine "echte" Bewegung zugrunde liegt.
Beispiel: Der Lichtkegel einer sich um die eigene Achse drehenden Lichtquelle erscheint auf einem hinreichend großen Radius als überlichtschnell bewegt.


Hallo Harti,

dieses Beispiel ist interessant, da es zeigt, dass nicht alles, was wir als Geschwindigkeit bezeichnen, denn auch wirklich Geschwindigkeit ist.

Es ist ja nicht so, dass sich auf dem Kreis um die Lichtquelle, auf der der Beobachter sitzt, irgend etwas bewegen würde. Was sich in den Punkten dieser Kreislinie in schneller Folge ändert, ist lediglich ihr Zustand "beleuchtet" bzw. "nicht beleuchtet" — ein Zustand also.

Ein noch interessanterer Aspekt deines Beispiels ist, dass — falls der Beobachter weiß, wie weit die Lichtquelle von ihm entfernt ist — das schnelle Wandern der Schatten­grenze auf der Kreislinie Information trägt: Die Information nämlich, wie schnell sich die Lichtquelle um ihre eigene Achse dreht. Der Weg aber, auf dem diese Information ihn erreicht, ist KEIN Teil der Kreislinie, sondern Teil der Geodäte, auf der Lichtquelle und Beobachter sich befinden. Nach meiner Definition aus Beitrag 2103-1 ist die sich ständig umdefinierende Grenze zwischen Licht und Schatten also KEIN Signal — und so darf es auch nicht erstaunen, dass sie mit unbegrenzt hoher Geschwindigkeit die Kreislinie entlang wandern kann.

Wer die Kreislinie ebenso wie die Geodäte mit Straßen vergleicht, über die Fahrzeuge kommen um die Kreuzung "Beobachter" zu überqueren, wird feststellen, dass sich ständig irgendein Fahrzeug auf der Kreuzung befindet, aber doch nur eine einzige dieser zwei Straßen befahren ist: die Geodäte. Nichts bewegt sich auf der anderen Straße (obgleich all deren Punklte wieder solch stark frequentierte Kreuzungen sind).

Damit ist bewiesen: Die Grenze zwischen Licht und Schatten, die sich auf der Kreislinie zu bewegen scheint, ist kein physisches Objekt, sondern nur ein rein virtuelles im Sinn der Definition aus Beitrag 2102-26:


Zitat:
 
Virtualität ist die Eigenschaft einer Sache, nicht in der Form zu existieren, in der sie zu existieren scheint, aber in ihrem Wesen oder ihrer Wirkung einer in dieser Form existierenden Sache zu gleichen. Das Wort führt über den französischen Begriff virtuel (fähig zu wirken, möglich).

Virtualität spezifiziert also eine gedachte oder über ihre Eigenschaften konkretisierte Entität, die zwar nicht physisch, aber doch in ihrer Funktionalität oder Wirkung vorhanden ist.

Somit ist "virtuell" nicht das Gegenteil von "real" – obwohl es fälschlicherweise oft so verwendet wird – sondern von "physisch".

 


Gruß, grtgrt
 

  Beitrag 2103-4
Zwei weitere Beispiele für Überlichtgeschwindigkeit

 
 

Nun noch zwei Beispiele für   Überlichtgeschwindigkeit  ,

bei denen — mir wenigstens — nicht mehr wirklich klar ist, ob man es mit realer oder nur virtueller Bewegung zu tun hat:



Zitat von Giolini (nicht ganz wörtlich):
 
Wenn man versucht, den Begriff der Geschwindigkeit auch einer Welle zuzuordnen, so gibt es dazu mehrere Möglichkeiten:

Mathematisch wird eine Welle beschrieben als Überlagerung unendlich ausgedehnter rein harmonischer Wollen von jeweils fester Frequenz und Wellenlänge. Die Phasen dieser Partialwellen breiten sich jeweils mit der sog. » Phasengeschwindigkeit « aus. Sie beträgt c/n, wo n der Brechungsindex des Mediums ist, in dem die Ausbreitung stattfindet.

Nun hängt n aber i.A. von der Frequenz der Partialwelle ab, so dass folglich die Partialwellen auch unterschiedliche Phasengeschwindigkeit haben können.
Dieses Phänomen bezeichnet man als Dispersion. Genauer: Man spricht von normaler/anormaler Dispersion, wenn n mit der Frequenz steigt/fällt (die Phasen­geschwindigkeit also fällt/steigt).


Mit den Phasen einer harmonischen, unendlich weit ausgedehnten Welle kann man aber keine Signale übertragen, also darf auch die Phasengeschwindigkeit größer als c werden, was in Frequenzbereichen, in denen anormale Dispersion vorliegt, oft passiert, da dort n < 1 werden kann.

Aus harmonischen Partialwellen kann man durch Überlagerung lokalisierte Modulation oder Wellengruppen bilden, deren Schwerpunkte sich mit der sog.
» Gruppengeschwindigkeit « ausbreiten. Sie kann in nur eingeschränktem Maße zur Signalübermittlung verwendet werden, denn:
    Die Dispersion führt u.U. zum Zerfließen solcher Wellenpakete, so dass Signalübermittling nur so lange funktionieren kann, wie das Zerfließen nicht zu sofortiger Zerstörung der das Signal darstellenden Wellengruppe führt. Genauer:
    Der mathematische Ausdruck für die Gruppengeschwindigkeit existiert zwar, doch kommt ihm in Regionen eines zu schnellen Zerfließens physikalisch nicht mehr die Bedeutung einer Signalübermittlung zu. Er kann dort größer als c werden, so dass Messungen von Gruppengeschwindigkeiten oberhalb der Licht­geschwindigkeit immer wieder zur physikalisch nicht korrekten Behauptung führen, es seien Signale — ja sogar ganze Mozart-Symphonien — mit Überlicht­geschwindigkeit übertragen worden (siehe etwa [ G. Nimtz: ein Tunneleffekt? ]).

Uneingeschränkt zur Signalübertragung geeignet ist jeder Einschaltvorgang. Er breitet sich mit der sog. » Frontgeschwindigkeit « aus. Das ist die Geschwindigkeit, mit der sich der Wellenkopf bewegt, jene Stelle also, an der die Welle von einer schon einige Zeit andauernden Nullamplitude zu einer von Null verschiedenen Ampli­tude abhebt. Man denke etwa an das Morsen, wo Strich und Punkt mehr oder weniger lange Signale sind, die durch Pausen absoluter Ruhe getrennt sind.

Es gibt dann noch die » Energiegeschwindigkeit «, mit der sich Energie in einer Welle ausbreiten kann. Sie muss mit keiner der oben genannten Geschwindigkeiten übereinstimmen und kann niemals größer als c sein.


FAZIT:
  • Phasen- und Gruppengeschwindigkeiten größer c widersprechen der SRT nicht.
  • Anders liegt die Sache bei Signal- und Energiegeschwindigkeiten: Sie würden, wenn sie größer als c sein könnten, die SRT in ernsthafte Schwierigkeiten bringen.


Quelle: Domenico Giolini: Spezielle Relativitätstheorie, 2004, Seite 109-111

 

  Beitrag 2103-1
Überlichtgeschwindigkeit — eine Klarstellung und 2 Beispiele

 
 


Überlichtgeschwindigkeit – es gibt sie auch in SRT und ART



Ein weit verbreiteter Irrtum besteht darin, zu glauben, dass es keine Geschwindigkeiten höher als die des Lichts gibt.

Tatsächlich gibt es beliebig hohe Geschwindigkeiten auch im Rahmen der beiden Relativitätstheorien. Beide — die SRT und auch die ART — beweisen lediglich, dass kein Signal sich schneller als das Licht bewegen kann.



Signal in diesem Sinne ist alles,
das sich auf der Geodäte der Raumzeit bewegt, die den Beobachter mit dem beobachteten Objekt verbindet


( in der SRT also auf einer Geraden, die durch den Beobachter führt ).



Eine gut verständliche Begründung — und auch zwei Beispiele — finden sich auf den Seiten 106 bis 111 des Buches Spezielle Relativitätstheorie des Physikers Domenico Giulini (erschienen 2004 im Fischer Taschenbuch Verlag):

Zitat von Giulini:
 
Wird die Frage » Gibt es Überlichtgeschwindigkeiten? « in dieser Allgemeinheit gestellt, so muss sie mit einem klaren JA beantwortet werden:

Nur bestimmten Ausbreitungsphänomenen setzt die SRT den Wert c als Grenze. Dies betrifft ... allgemein alle Prozesse, die zumindest im Prinzip einer Signalübertragung dienen können, wobei man hier allerdings eine sorgfältige Definition von » Signal « anfügen müsste. Auf jeden Fall würde eine Signalausbreitung mit Überlichtgeschwindigkeit innerhalb der SRT zu Widersprüchen in Kausalitätsverhältnissen führen.
 


Hier nun die beiden Beispiele für tatsächlich  b e o b a c h t b a r e  Überlichtgeschwindigkeit, die Giulini im Detail diskutiert:

Vorsicht aber:  B e o b a c h t e t e  Geschwindigkeiten sind Scheingeschwindigkeiten, die für zueinander bewegte Beobchter  u n t e r s c h i e d l i c h  hohen Wert haben (als mit Sicherheit keine reale Geschwindigkeit darstellen).


Beispiel 1 (nachgerechnet auf Basis der SRT):
    Giulini betrachtet einen Beobachter B, der im Ursprung eines Inertialsystems sitzt und ein Objekt X beobachtet, das sich auf einer Geraden mit gleichförmiger Geschwindigkeit bewegt. Diese Gerade führe nicht durch den Beobachter, wohl aber durch einen Punkt P. Bezeichnet nun a den spitzen Winkel zwischen dieser Geraden und der Strecke von B nach P, so kommt Giulini über einige wenige Rechenschritte hin zur Aussage, dass für die durch B beobachtete Geschwindigkeit v(B) des Objekts X
    v(B) = c • s(a,v)
    gilt, wenn man
    s(a,v)   =   ( sin(a) v/c ) / ( 1 – cos(a) v/c )
     
    setzt und hierbei unter v die Geschwindigkeit versteht, mit der sich X als Signal seiner Bahn entlang durch den 3-dimensionalen Raum bewegt.
    Für ein festes Verhältnis v/c und variablen Winkel a erreicht die Funktion s(a,v) ihr Maximum bei cos(a) = v/c . Dort ist dann also
     
    v(B)   =   c • sin(v/c) / ( 1 – (v/c)2 )   ,
     
    was für v gegen c nach unendlich strebt, so dass also tatsächlich v(B) auch weit größer als c werden kann.


Beispiel 2 (auf Basis ART, da durch astronomische Beobachtung entdeckt):

 
Zitat von Giulini:
    Heute kennt man in der Astronomie zahlreiche Beispiel für diesen Effekt [ beobachteter Überlichtgeschwindigkeit ].
    Ein besonders eindrucksvolles liefert die Galaxie M87, die sich in einer Entfernung von 60 Mio Lichtjahren von uns im Virgohaufen befindet. Aus ihrem Zentrum werden Gasströme entlang sog. Jets auf einer Länge von 5000 Lichtjahren ins All geschleudert, deren gemessene visuelle Geschwindigkeit v(B) die 6-fache Lichtgeschwindigkeit erreichen!.
    Getrieben werden diese Jets wahrscheinlich durch ein im Zentrum der Galaxie vorhandenes supermassives Schwarzes Loch. Man schätzt die eigentliche Geschwindigkeit v der Gasströmung [ die also, die sie als Signal hat ] auf höchstens 98% der Lichtgeschwindigkeit.


PS 1: Diese Beispiele zeigen einmal mehr, dass — worauf ich hier im Forum schon mehrmals zu sprechen kam — durch Beobachter gemessene Geschwindigkeiten stets nur  S i c h t  auf eine Signalgeschwindigkeit sein können (niemals aber wirklich reale Geschwindigkeiten sind). Mit anderen Worten:

Es gilt zu unterscheiden zwischen
  • scheinbarer ( d.h. beobachteter ) Geschwindigkeit einerseits und
  • Signalgeschwindigkeit andererseits (nur sie kann niemals höher als Lichtgeschwindigkeit sein; ihre Richtung ist stets tangential zu einer Geodäte).

Erst wer das verstanden hat, kann verstanden haben, wie die SRT denn nun wirklich argumentiert und was ihre Aussagen tatsächlich bedeuten.


PS 2: Man sollte sich zudem klar machen, dass die Raumzeit — ausgestattet mit der Minkowski-Metrik — nur eine einzige  r e a l e  Geschwindigkeit kennt: die des Lichts.
Denn alles, was man sonst noch so an Geschwindigkeiten kennt, sind einfach nur beobachter-spezifische  S i c h t e n , die sich ergeben
  • entweder als Projektion von Signalgeschwindigkeit in der Raumzeit auf den 3-dimensionalen Raum
  • und/oder als Schrumpfen oder Wachsen von Abständen (pro Zeiteinheit) zweier vom Beobachter ins Auge gefasster Objekte aus der spezifischen Perspektive des Beobachters und seiner Uhr heraus.

 

  Beitrag 1991-1
Kernaussagen der Speziellen Relativitätstheorie

 
Hallo zusammen,

bitte hier keine Beiträge einstellen!

Warum? Es soll nur der erste Beitrag hier verbleiben, den ich immer auf den neuesten Stand nach Absprache mit Zara.t. bringe. Er soll eine Wissensplattform zur SRT für alle werden, damit jeder darauf zurückgreifen kann, wenn Fragen entstehen. Hier eingehende Beiträge werden weder von mir noch von Zara.t. beantwortet.

Wenn ihr darüber diskutieren möchtet, dann bitte im Thread Basics spezielle Relativitätstheorie! Vielen Dank im Voraus.

M.f.G. Eugen Bauhof




Forenteilnehmer, deren Beiträge bis jetzt hier eingebracht wurden (in alphabetischer Reihenfolge):
Bauhof, Grtgrt, Henry, Okotombrok, Zara.t.



Themen, die demnächst in den SRT-Basics diskutiert werden sollen:

(A) Begriff des Beobachtens.
(B) Maxwellsche Gleichungen.
(C) E = mc2.
(D) Die Lorentz-Transformationen, hergeleitet allein aus dem Relativitätsprinzip.
(E) Transformationen, die das Linienelement ds2 = (c dt)2 - dx2 - dy2 - dz2 invariant lassen.


Arbeitsplattform SRT



Inertialsysteme
1. Verschiedene Inertialsysteme bewegen sich gegeneinander geradlinig und gleichförmig. Alle Inertialsysteme bewegen sich relativ zueinander mit einer Geschwindigkeit v<c.

2. Sich drehende oder anderweitig beschleunigte Bezugssysteme sind keine Inertialsysteme.

3. Auch gleichförmig beschleunigte Systeme sind keine Inertialsysteme. Eine gleichförmige Beschleunigung ist keine gleichförmige Bewegung und somit nicht der Ruhe äquivalent.

4. Ein Teilchen, das sich mit Lichtgeschwindigkeit bewegt, hat keine Ruhemasse. Deshalb gibt es kein Inertialsystem, in dem Photonen in Ruhe wären.

5. Ein System, in dem sich jeder kräftefreie Körper gleichförmig geradlinig bewegt, wird als Inertialsystem bezeichnet.

6. Das Zusammentreffen von Kräftefreiheit und gleichförmiger Geradlinigkeit sämtlicher Bewegungen gibt es nur in Inertialsystemen.

7. Es kann kein Inertialsystem geben, das sich relativ zu einem zweiten Inertialsystem mit c bewegt. Grund: alle lichtartigen Weltlinien haben die Länge Null.

8. Wenn ein Lichtstrahl im Koordinatenursprung O(0,0,0) startet. dann gilt:
S2 = (ct)2 - x2 - y2 - z2 ; für Licht gilt: x2 + y2 + z2 = (ct)2 ==> S2 = (ct)2 - (ct)2 = 0; S = 0
Wichtig: S ist für alle (also auch für die nicht lichtartigen) Weltlinien invariant unter Lorentztransformationen. Das heißt: die Eigenzeit eines Systems ist eine absolute Größe, sie ist die Zeit, die eine mit diesem System fest verbundene Borduhr anzeigt.

Relativitätsprinzip
1. Die Gesetze, nach denen sich die Zustände der physikalischen Systeme ändern, sind unabhängig davon, auf welches von zwei relativ zueinander in gleichförmiger Translationsbewegung befindlichen Koordinatensystemen diese Zustandsänderungen bezogen werden.

2. Alle Inertialsysteme sind gleichberechtigt, wenn es gilt Naturgesetze zu formulieren. Naturgesetze müssen kovariant formuliert werden können.

3. Alle Beobachter, die sich mit gleichförmiger Geschwindigkeit relativ zueinander bewegen, sind gleichermaßen zu der Behauptung berechtigt, sie befinden sich in Ruhe.

Absolute Ruhe
Die Physik kennt keine absolute Ruhe. Die Gesetze der Physik lauten in allen Inertialsystemen gleich und zeichnen keines aus (Relativitätsprinzip). Es gibt nur relative Ruhe, Ruhe in bezug auf ein spezielles Inertialsystem, das momentane Ruhsystem.

Absolutgeschwindigkeit
Mit keinem Experiment kann eine Absolutgeschwindigkeit eines Inertialsystems bestimmt werden; kein Experiment lässt die Unterscheidung von Ruhe und gleichförmiger Bewegung zu. Damit war auch das vor Einstein der als absolut ruhendes Lichtwellen-Trägermedium angesehene "Lichtäther" gegenstandslos, den "Lichtäther" gibt es nicht.

Beobachten
Unter Beobachten versteht man die Feststellung der Koordinaten von Ereignissen in einem Inertialsystem. Wenn man das wirkliche Beobachten meint, muss man die endliche Laufzeit des Lichtsignals vom Gegenstand zum Betrachter berücksichtigen.
Den Begriff des Beobachtens möchte Zara.t. noch diskutieren. Er ist von zentraler Wichtigkeit um das Denken und die Arbeitsweise Einsteins zu verstehen.

Lichtgeschwindigkeit
1. Die Lichtgeschwindigkeit ist eine physikalische Naturkonstante und hat einen exakt definierten Wert.

2. Man kann c=1 setzen. Um hier im Forum Missverständnisse zu vermeiden, wird folgendes vereinbart:

Lichtgeschwindigkeit im Vakuum (im SI-System): c = 299 792 458 m/s

3. Die Existenz einer invarianten Grenzgeschwindigkeit folgt aus dem Relativitätsprinzip. Der Wert dieser Grenzgeschwindigkeit wird von den Maxwellschen Gleichungen festgelegt und durch alle Messungen bisher bestätigt.

4. Aus den Maxwellschen Gleichungen ergibt sich die Ausbreitungsgeschwindigkeit c elektromagnetischer Wellen aus zwei experimentell herausgefundenen Konstanten:
c = 1/sqrt(ε0 • ε0)
ε0 = magnetische Feldkonstante.
ε0 = elektrische Feldkonstante.

Eigenzeit einer Uhr
Die Eigenzeit einer Uhr entspricht immer der Länge ihrer Weltlinie.

Länge der Weltlinie eines bewegten Objekts
Die Länge der Weltlinie eines ruhenden Objekts (keine Änderung der Ortskoordinaten) ist die abgelaufene Zeit. Das bleibt auch bei einer Bewegung des Objekts so, wir müssen uns nur darauf beschränken, wieder nur die Uhr abzulesen, die das Objekt unmittelbar begleitet. Dieses Zeitmaß heißt Eigenzeit... Der Ablauf der Eigenzeit wird nun nicht mehr nur von der Änderung der Zeitkoordinate, sondern auch von der Änderung der Ortskoordinate bestimmt.

Maxwellsche Gleichungen
Die Maxwellschen Gleichungen sagen in einfachen Worten: Die Änderung eines elektrischen Feldes induziert ein magnetisches Feld, die Änderung des magnetischen Feldes induziert ein elektrisches Feld, das sich ändernde elektische Feld induziert ein Magnetfeld, usw....dieser ständige Wechsel breitet sich wellenförmig mit c (im Vakuum) aus. Die Maxwellschen Gleichungen sind früher als die SRT unabhängig von der SRT entstanden. An ihrem Beispiel könnten wir zeigen, was eine kovariante Formulierung ist. So wie Maxwell seine Gleichungen formulierte, waren sie nämlich noch nicht kovariant. Man kann ihnen aber eine kovariante Form verpassen.

Energie-Impuls-Beziehung
Die Energie-Impuls-Beziehung ist eine invariante Größe:

E2 – (pc)2 = (mc2)2 || = constant

E..................Gesamtenergie, sie ist bezugssystemabhängig.
p...................Impuls, er ist bezugssystemabhängig.
mc2:=E0.......Ruheenergie sie ist bezugssystemunabhängig.

Nachdem die Ruheenergie bezugssystemunabhängig ist, ist auch die Masse m invariant.




SRT-Glossar

Äther
Ein Medium, in dem sich Licht und andere elektromagnetische Wellen fortpflanzen sollten. Diese Vorstellung wurde, vor allem nach der Entdeckung der Speziellen Relativitätstheorie, durch eine Feldbeschreibung ersetzt, nach der sich elektromagnetische Wellen auch ohne jedes Medium im leeren Raum fortpflanzen können. Die Vorstellung vom Äther wurde durch die Relativitätstheorie widerlegt.

Beobachter
Idealisierte Person oder Apparatur, oft hypothetischer Natur, die relevante Eigenschaften eines physikalischen Systems misst. Jeder Beobachter, der sich inertial bewegt, darf annehmen, dass er selbst in Ruhe sei. Er beobachtet lediglich, dass alle anderen Beobachter sich relativ zu ihm bewegen oder in Ruhe sind. Mehr kann er nicht beobachten, insbesondere kann er keine Eigenbewegung aus den Bewegungen anderer ableiten. Für ihn sieht es so aus, als würde er ruhen und alle anderen Beobachter bewegen sich.

Beschleuniger
Ein Instrument, das die Geschwindigkeit, und damit die Energie, geladener Teilchen erhöht. Beschleunigt werden können stabile Teilchen wie Elektron, Proton und Ionen.

Beschleunigung
Eine Veränderung der Geschwindigkeit, das heißt des Geschwindigkeitsbetrags und/oder der Bewegungsrichtung eines Objekts.

Bezugssystem
Die räumliche Beschreibung von Vorgängen braucht ein Bezugssystem (Koordinatensystem, Koordinaten) In einem Bezugssystem kann man die Bewegung eines Körpers beschreiben, ebenso aber auch die Bewegung eines anderen Bezugsystems, z. B. eine gradlinig gleichförmige Bewegung eines Bezugssystems in einem anderen, oder auch eine Drehung, eine krummlinige Bewegung oder eine beliebige Beschleunigung. Die Spezielle Relativitätstheorie diskutiert die Frage, wie sich zwei Beschreibungen desselben Vorgangs zueinander verhalten, die zu Bezugssystemen gehören die gegeneinander gleichförmig bewegt sind. Alle Bezugsysteme, die sich relativ zu einem Inertialsystem gleichförmig und geradlinig bewegen, sind ebenfalls Inertialsysteme.

Dilatation
Das Auseinanderziehen, die Verlängerung. Hier gebraucht im Zusammenhang mit Zeitdilatation, d.h. die Verlängerung der Zeiten in einem bewegten System gemäß der Speziellen Relativitätstheorie.

Doppler-Effekt-Verschiebung
Verschiebung der Spektrallinien einer Strahlung durch die Bewegung ihrer Quelle relativ zur Sichtlinie. Annäherung verursacht Blauverschiebung, Entfernung Rotverschiebung.

Eigenzeit
Eigenzeit ist die Zeit, gemessen von einer Uhr, welche die Bewegung eines Beobachters mitmacht. Uhren in relativer Bewegung zu einem Beobachter messen, dass seine und die Eigenzeit verschieden schnell vergehen. Die Länge der Weltlinie eines ruhenden Objekts (keine Änderung der Ortskoordinaten) ist die abgelaufene Zeit. Das bleibt auch bei einer Bewegung des Objekts so, wir müssen uns nur darauf beschränken, wieder nur die Uhr abzulesen, die das Objekt unmittelbar begleitet. Dieses Zeitmaß heißt Eigenzeit. Der Ablauf der Eigenzeit wird nun nicht mehr nur von der Änderung der Zeitkoordinate, sondern auch von der Änderung der Ortskoordinate bestimmt.

Elektrodynamik
Theorie der elektrischen und magnetischen Erscheinungen, zusammengefasst in der einheitlichen Elektrodynamik am Ende des 19. Jahrhunderts durch Maxwell. Dazu gehört vor allem die Theorie der elektromagnetischen Wellen, wie z. b. Radiowellen, Licht oder Gammastrahlen.

Elektromagnetische Welle
Eine wellenartige Störung in einem elektromagnetischen Feld. Alle diese Wellen breiten sich mit Lichtgeschwindigkeit aus. Sichtbares Licht, Röntgenstrahlen, Mikrowellen und Infrarotstrahlung sind Beispiele dafür.

Energie
Physikalische Grundgroße, für die ein strenger Erhaltungssatz gilt. Sie kann verschiedene Formen annehmen, z. B. die potentielle Energie eines im Schwerefeld hochgehobenen Gewichts, die kinetische Energie eines bewegten Körpers, die Wärmeenergie, in die schließlich jede Energie anderer Formen übergeht, oder die als Masse vorhandene Energie gemäß der Speziellen Relativitätstheorie. ("Trägheit der Energie")

Ereignis
Ein Weltpunkt in der Raumzeit. Beschrieben durch vier Koordinaten: {x, y, z. t}.

Frequenz
Die Zahl vollständiger Wellenzyklen, die eine Welle pro Zeiteinheit durchläuft.

Grobes Missverständnis der SRT.
Etliche Kritiker der SRT in den letzten 100 Jahren glaubten, die relativistische Zeitdilatation sei ein rein "perspektivischer" Beobachtungs-Effekt und allein durch die Lichtlaufzeiten zum Beobachter zu erklären. Ein ganz grobes Missverständnis der SRT.

Geschwindigkeit
Der Geschwindigkeitsbetrag und die Bewegungsrichtung eines Objekts, zu einer Größe zusammengefasst. Die Geschwindigkeit ist differentiell definiert als Ortsänderung pro Zeiteinheit.: v = dx/dt. Wenn eine punktförmige Lichtquelle angeschaltet wird, entsteht eine Kugelwelle, deren Radius sich mit 299 752 458 m/sec vergrößert. Ihr Durchmesser aber vergrößert sich doppelt so schnell. Das ist kein Widerspruch zur SRT, da die Rate, mit der sich ein Abstand vergrößert, nicht als Ortsveränderung eines physikalischen Objekts verstanden werden kann. Diese Vergrößerung des Durchmessers kann nicht mit der Einsteinschen Geschwindigkeitsadditions-Formel berechnet werden, weil es sich bei der Verdoppelung des Durchmessers nicht um eine Geschwindigkeit im Sinne der SRT handelt.

Gleichzeitigkeit
In der klassischen Mechanik ist die Gleichzeitigkeit von Ereignissen unabhängig von jedem Bewegungszustand und von der Entfernung definiert. In der speziellen Relativitätstheorie hängt die Definition der Gleichzeitigkeit von der Relativ-Geschwindigkeit und von der Entfernung ab (Einsteins Relativität der Gleichzeitigkeit).

Gleichzeitigkeits-Definition von Einstein
"Die für A und B gemeinsame Zeit kann so definiert werden, indem man durch Definition festsetzt, dass die Zeit, welche das Licht braucht, um von A nach B zu gelangen, gleich ist der Zeit, welche es braucht, um von B nach A zu gelangen."

Für entfernte Punkte kann man zu einer Gleichzeitigkeits-Definition also überhaupt nur gelangen auf dem Wege eines konstruktiven Postulats, in dem gefordert wird, dass die Ausbreitung des Lichtes, gemessen in der noch zu definierenden gemeinsamen Zeit, ein Vorgang mit konstanter Geschwindigkeit ist. Dieses Postulat ist entscheidend an der Konstruktion des Begriffs der gemeinsamen Zeit beteiligt.

Ideale Uhren.
Zur Herleitung der Theorien (der SRT und der ART) legt man sogenannte "ideale" Uhren zugrunde. So eine ideale Uhr wird realisiert durch Atomuhren. Reale Experimente, welche die SRT belegen, werden mit den Taktraten von Atomuhren durchgeführt

Inertialsystem.
Ein Inertialsystem ist ein Bezugssystem, in dem sich kräftefreie Körper geradlinig gleichförmig bewegen. Das bedeutet, dass es sich bei einem Inertialsystem um ein nicht rotierendes und nicht beschleunigtes Bezugssystem handelt. Alle Bezugsysteme, die sich relativ zu einem Inertialsystem gleichförmig und geradlinig bewegen, sind ebenfalls Inertialsysteme. Somit bewegen sich Körper, auf die keine Kräfte wirken, in allen Inertialsystemen auf Geraden. Eine gleichförmige Rotationsbewegung wäre demzufolge nichtinertial. Jeder nichtrotierende Beobachter, der sich gleichförmig und unbeschleunigt bewegt, befindet sich in einem Inertialsystem. Auf ihn wirken keine Kräfte.
Es gibt kein ausgezeichnetes Inertialsystem, von dem man behaupten könnte, es sei absolut ruhend. Es gibt keinen Punkt im Universum, von dem man sagen könnte, das sei allein der "absolut ruhende Pol", auf dem man jede andere Bewegung beziehen könnte. Insbesondere gibt es auch keinen "absoluten Raum", auf dem man alle Bewegungen beziehen könnte. Sowohl in der klassischen Mechanik wie in der Speziellen Relativitätstheorie sind alle Inertialsysteme für die Beschreibung gleichberechtigt.

Isotropie
Unabhängigkeit von Richtung oder Winkel. In einem isotropen Universum sind sämtliche messbaren Größen in allen Richtungen gleich.

Invarianz
Die Unveränderlichkeit bestimmter Größen oder Beziehungen, auch wenn andere Charakteristika transformiert werden. Invarianz-Überlegungen der Physik beziehen sich gewöhnlich auf die Invarianz von Größen gegenüber Transformationsgruppen.

Invarianz der Lichtgeschwindigkeit.
Unter der Invarianz der der Lichtgeschwindigkeit versteht man z.B. folgendes: Für das Licht wird immer die gleiche Geschwindigkeit gemessen, unabhägig davon, ob sich die Lichtquelle auf uns zu- oder von uns fortbewegt. Das sehen alle Beobachter so, gleichgültig, ob sie auf der Lichtquelle sitzen oder ob sie sich außerhalb befinden und die Lichtquelle als bewegt wahrnehmen. Daraus folgt: Die Lichtgeschwindigkeit ist in allen Inertialsystemen gleich groß.

Kinematik der Relativitätstheorie
Die Grundlagen der Relativitätstheorie können durch geometrische Intuition erschlossen werden. Die Kinematik der Relativitätstheorie kann daher als die Geometrie der Raumzeit-Union angesehen werden.

Kinetische Energie
ist die mit der Bewegung verknüpfte Energie; sie ist gleich der Arbeit, die geleistet werden muss, um einen Körper der Masse m aus dem Ruhezustand in einen Bewegungszustand mit der Geschwindigkeit v zu bringen. Sie beträgt in der klassischen Mechanik 1/2•m•v2.

Konstanz der Lichtgeschwindigkeit
Darunter versteht man die Unabhängigkeit von der Ausbreitungsrichtung und die Unveränderlichkeit ihrer Größe bei Zusammensetzung.
Unabhängigkeit von der Ausbreitungsrichtung heißt zum Beispiel: Wenn ein Lichtstrahl senkrecht auf eine Spiegelfläche mit der Geschwindigkeit v=c trifft, dann hat auch der reflektierte Strahl die Geschwindigkeit v=c. Strahlen, die nicht senkrecht auftreffen, werden ebenfalls mit der Geschwindigkeit v=c reflektiert.

Kovariant
Die Beschreibung eines physikalischen Systems ist kovariant, wenn bei einer Transformation des Systems die transformierte Beschreibung des transformierten Systems dieselben Ergebnisse liefert wie die ursprüngliche Beschreibung des ursprünglichen Systems.

Lichtlaufzeiten.
Mit den Lichtlaufzeiten zum Beobachter sind die relativistischen Effekte nicht erklärbar. Das schließt nicht aus, dass man zum besseren Verstehen Gedankenexperimente macht, in denen die Lichtlaufzeiten mit einbezogen werden.

Lichtuhr
Eine hypothetische Uhr, welche die verstrichene Zeit misst, indem sie zählt, wie viele vollständige Rundreisen ein einzelnes Photon zwischen zwei Spiegeln zurücklegt.

Lorentz-Kontraktion
Besonderheit, die sich aus der speziellen Relativitätstheorie ergibt. Ein bewegtes Objekt erscheint einem ruhenden Beobachter, der dessen Länge misst, in Richtung der Bewegung verkürzt.

Minkowski-Raumzeit
Minkowski erkannte, dass die Kinematik der SRT graphisch darstellbar wird, wenn man neben den altbekannten kartesischen Koordinatenachsen x, y, z noch die vierte Achse ict einführt und sich bei der Veranschaulichung relativistischer Raum-Zeit-Verhältnisse auf jeweils zwei Achsen, typischerweise die Achsen x und ict, beschränkt. Für die Abstände zwischen Weltpunkten, die jetzt Raum- und Zeitkoordinaten hatten, galt die alte euklidische Regel quadrierter Koordinaten-Differenzen, allerdings mit einem durch i2 = –1 veränderten Vorzeichen des zeitlichen Anteils:

ds2 = dx2 + dy2 + dz2 + d(ict)2

Die Lorentztransformationen konnten dann als Drehung in dieser vierdimensionalen Raumzeit verstanden werden, bei der diese Größe ds2 invariant blieb ebenso wie bei Drehung im dreidimensionalen Raum der dreidimensionale Abstand zweier Körper unverändert bleibt. Effekte wie die Lorentzkontraktion und die Zeitdilatation konnten nun als durch Projektion von Längen- und Zeitanteil von eigentlich längen- und zeitbehafteten Größen auf zueinander um den Winkel ß = arctan(v/c) gedrehte Koordinatenachsen anschaulich interpretiert werden.

Mittelpunkt im Universum.
Im Universum ist kein Ort feststellbar, der als Mittelpunkt im Universum bezeichnet werden könnte.

Newtonsche Bewegungsgesetze
Gesetze, welche die Bewegung von Körpern unter Kräfteeinfluss beschreiben, wobei sie voraussetzen, dass Raum und Zeit absolut und unwandelbar sind. Diese Gesetze galten unverändert, bis Einstein die spezielle Relativitätstheorie entdeckt hatte, die zeigt, dass die Newtonsche Theorie nur eine Näherung für den Grenzfall kleinerer Geschwindigkeiten ist.

Photon
Kleinstes Paket des elektromagnetischen Felds. Botenteilchen der elektromagnetischen Kraft. Kleinstes Lichtpaket.

Pseudoeuklidische Geometrie
Während in einem euklidischen Raum das Quadrat des Abstandes zweier verschiedener Punkte P1 und P2 stets größer als Null ist, kann es in der Minkowski-Welt auch kleiner oder gleich Null sein. Die Raumzeit-Union der Relativitätstheorie besitzt also keine euklidische Maßbestimmung. Vielmehr ist die Maßbestimmung die vierdimensionale Verallgemeinerung der pseudo-euklidischen Geometrie. Dies ist der mathematische Ausdruck dafür, dass die Zeitkoordinate eine andere Qualität als die 3 Raumkoordinaten besitzt.

Raum und Zeit als untrennbare Einheit.
Man kann die SRT nicht auf der Grundlage der klassischen Betrachtung mit Trennung von Raum und Zeit verstehen. Man kann die SRT erst dann verstehen, wenn man Raum und Zeit als untrennbare Einheit ansieht.

Raumzeit
Die drei physikalischen Dimensionen des Raums werden mit der Zeit, die als vierte Dimension aufgefasst wird, verknüpft und ergeben so das Raum-Zeit-Kontinuum, das den grundlegenden Rahmen in der Relativitätstheorie darstellt.

Relativgeschwindigkeit.
Jede Geschwindigkeit (eines Körpers oder eines Inertialsystems) ist immer nur eine Relativgeschwindigkeit.

Relativität der Bewegung
Relativ bedeutet, dass die Bewegung nur in Bezug auf ein anderes Objekt festgestellt werden kann. Die Relativität der Bewegung gleichförmig bewegter Objekte ist keine Entdeckung der SRT, sondern wurde bereits von Galileo erkannt.

Relativität der Gleichzeitigkeit.
Das ist die erste Konsequenz der Konstanz der Lichtgeschwindigkeit. Einstein argumentierte etwa wie folgt: Auch die Zeit ist relativ. Es gibt keine absolute Zeit, die für alle Inertialsysteme gültig ist. In jedem Inertialsystem läuft die Zeit anders ab, abhängig davon, wie schnell sich die Inertialsysteme relativ zueinander bewegen. Und das ist ein totaler Umsturz unseres Weltbildes, weil seit Newton die Zeit "überall gleich verfließt". Und genau das tut sie nicht.

Was für einen Beobachter gleichzeitig ist, ist für einen anderen bewegten Beobachter nicht gleichzeitig. Ob zwei räumlich getrennt liegende Ereignisse gleichzeitig sind oder nicht, kann nicht dadurch entschieden werden, dass man mit Hilfe einer Messung die Gleichzeitigkeit zweier Ereignisse feststellen könnte. Der Gleichzeitigkeits-Begriff getrennt liegender Ereignisse erlangt erst durch eine Definition seine Bestimmtheit. Gleichzeitigkeit ist objektiv unbestimmt.

Relativitätsprinzip
Zentraler Begriff der Relativitätstheorie, nach der die physikalischen Gesetze für alle Beobachter, die mit konstanter Geschwindigkeit relativ zueinander bewegt sind, die gleiche Form haben; daher ist jeder dieser Beobachter gleichermaßen zu der Behauptung berechtigt, er befinde sich in Ruhe. Dieses Prinzip wird in der allgemeinen Relativitätstheorie zum Äquivalenzprinzip erweitert. Bereits in der Newtonschen Mechanik waren alle Inertialsysteme gleichberechtigt. Die Newtonschen Axiome der Mechanik zeichnen kein Inertialsystem gegenüber dem anderen aus. Der Übergang von einem Inertialsystem in ein anderes Inertialsystem wird in der Newtonschen Mechanik durch eine Galilei-Transformation vollzogen.

Relativistischer Dopplereffekt
Der relativistische Dopplereffekt hängt nur von der Relativgeschwindigkeit von Sender und Empfänger ab.

Ruheenergie
ist die aus der Beziehung E0 = mc2 berechnete Energie , wobei m die Masse des Teilchens und c die Lichtgeschwindigkeit ist. Die Ruhenergie wird nur dann vollständig freigesetzt, wenn ein Teilchen mit seinem Antiteilchen zerstrahlt.

Ruhesystem.
Man kann sich als Beobachter in jedes beliebige Inertialsystem gedanklich hineinsetzen kann, um dieses Inertialsystem dann zum Ruhesystem zu erklären.

Spezielle Relativitätstheorie
Einsteins Gesetze von Raum und Zeit in Abwesenheit von Gravitation.

SRT als Sonderfall der ART
Die SRT lässt sich als Sonderfall der ART interpretieren, sie ist gültig für einen massefreien Raum. Bei einem masseerfüllten Raum lässt sich die SRT lokal näherungsweise anwenden.

Teilchenbeschleuniger
Anlage, die Teilchen fast auf Lichtgeschwindigkeit beschleunigt und sie dann mit anderen Teilchen zusammenstoßen lässt, um ihren materiellen Aufbau zu ermitteln.

Vektorraum
In der Mathematik eine Gesamtheit von Vektoren, d.h. Objekten, die sich addieren und mit Zahlen multiplizieren lassen. Vektorräume unterscheiden sich nach der Art dieser Zahlen (z. b. reelle oder komplexe Zahlen) und nach ihrer Dimensionszahl. Zum Beispiel bilden die möglichen Geschwindigkeiten im Raum einen dreidimensionalen reellen Vektorraum.

Vierdimensionaler Abstand zwischen zwei Ereignissen.
Für alle inertialen Beobachter ist der vierdimensionale Abstand zwischen zwei Ereignissen in der Raumzeit verbindlich. Alle messen den gleichen Wert. Man sagt, dieser Abstand ist das "Raumzeit-Intervall" und dieses ist invariant für alle Beobachter. Wenn man die zwei Raumdimensionen y und z weglässt und nur eine Raumdimension x und die Zeitdimension t betrachtet, dann ergibt sich das Raumzeit-Intervall s2 zwischen zwei Ereignissen für zwei verschiedene Systeme wie folgt:

s2 = (ct')2 - (x')2 = (ct)2 - (x)2.

Wellenlänge
Elektromagnetische Strahlung wird durch ihre Wellenlänge oder ihre Frequenz charakterisiert, deren Produkt gleich der Lichtgeschwindigkeit ist. Die Wellenlänge ist die Entfernung zwischen aufeinanderfolgenden Wellenfronten, und die Frequenz ist die Zahl der Wellenfronten, die an einen gegebenen Punkt in einer Sekunde vorbeilaufen. Im sichtbaren Bereich hat das Licht eine Wellenlänge von 400 bis 700 Nanometer und eine Frequenz von 7 x 1014 bis 4 X 1014 Hertz.

Weltlinie
Eine Abfolge von Weltpunkten.

Weltpunkt
Ein Ort in der Raumzeit. Er ist bestimmt durch vier Koordinaten: {x, y, z, t}

Zwillingsparadoxon.
Gegen Einsteins Zwillingsexperiment wird meist mit der Symmetrie der Zeitdilatation argumentiert:

Wenn Zwilling B einerseits die Uhr von Zwilling A langsamer gehen sieht, dann muss umgekehrt doch Zwilling A auch die Uhr von B langsamer gehen sehen. Das ist zwar richtig, gilt aber nur, solange sich Zwilling A und B jeweils in einem Inertialsystem bewegen. Doch Zwilling B muss umkehren, um zu A zurückkehren zu können, d. h., er beschleunigt und befindet sich daher nicht in einem Inertialsystem, im Gegensatz zu B.

Diese Antwort wiederum hat dazu geführt, dass oftmals die Beschleunigungsphasen von B für die Verjüngung verantwortlich gemacht werden. Aber auch das ist nicht richtig, wie sich leicht zeigen lässt, indem man aus dem Zwillingsparadoxon ein Drillingsparadoxon macht. In der nachstehenden Skizze sind die Weltlinien der Drillinge A, B und C dargestellt, wobei C identische Beschleunigungsphasen (gekrümmte Kurvenstücke, durchgezogen gezeichnet) durchläuft, allerdings ist seine Reise kürzer.



Nach Zusammenkunft aller drei ist A älter als C und C älter als B. Die Beschleunigungsphasen können also nicht für den Altersunterschied verantwortlich gemacht werden. Es ist die physikalische Geometrie der relativistischen Raumzeit: Der Weg von B ist kürzer als der von C, und dieser wiederum ist kürzer als der Weg von A.

Zwillingsexperiment als Quintessenz mehrerer Fach-Autoren.
1. Die Symmetrie der Zeitdilatation wird gebrochen, sobald der reisende Zwilling umkehrt.

2. Die Umkehr ist gleichbedeutend mit einem Inertialsystemwechsel.

3. Der Inertialsystemwechsel bei der Umkehr lässt die auf seiner Reise erarbeitete Zeitdilatation des reisenden Zwillings manifest werden.

4. Die verflossene Eigenzeit, die der jeweilige Zwilling auf seiner Uhr abliest, entspricht der jeweiligen Länge der Weltlinie im Minkowski-Raum.

5. Die Beschleunigungsphasen des reisenden Zwillings sind nicht die Ursache der Zeitdilatation. Man kann die Beschleunigungsphasen durch ein Differenzexperiment zum Verschwinden bringen. Die Zeitdilatation tritt dann trotzdem auf. Die Zeitdilatation ist im wesentlichen abhängig von der Reisedauer und der Reisegeschwindigkeit des reisenden Zwillings.

6. Manche meinen, dass nur die ART das scheinbare Paradoxon auflösen könnte, weil Beschleunigungen im Spiel sind. Mal ganz abgesehen davon, dass man mit der SRT auch Beschleunigungen beschreiben kann, ist die ART zu Auflösung des Zwillingsparadoxons nicht notwendig. Es kann allein mit Mitteln der SRT gelöst werden. Die ART wird nur dann benötigt, wenn die Gravitation ins Spiel kommt.



Herleitung der Lorentz-Transformationen mit Hilfe einer Koordinatensystemdrehung im Minkowski-Raum.

Zunächst wird die gewöhnliche Koordinatentransformation im euklidischen Raum in Skizze 1 dargestellt, so wie sie in jeder Formelsammlung zu finden ist:


Skizze 1


Der Mathematiker Hermann Minkowski Minkowski ging von der Invarianz des Linienelementes aus und suchte allgemeine Transformationsgleichungen, die dies gewährleisten. Er hat dabei herausgefunden, dass allgemein jede Transformation zwischen relativ zueinander bewegten Inertialsystemen durch eine Koordinatensystemdrehung um einen imaginären Winkel in einem vierdimensionalen Raum darstellbar ist.

Gegeben sind zwei Inertialsysteme S und S', die sich relativ zueinander mit der Geschwindigkeit v bewegen. Ersetzt man w durch ict und ß durch den imaginären Winkel (iß), dann wird aus der reellen Drehung im euklidischen Raum eine imaginäre Drehung im Minkowski-Raum (x, w=ict). Die imaginäre Einheit in w=ict gewährleistet die pseudoeuklidische Metrik der Minkowski-Raumzeit.

Das ist vorläufig eine willkürliche mathematische Festsetzung ohne physikalische Begründung. Durch diese Festsetzung wird auch der Drehwinkel imaginär. Einstein stand ursprünglich den mathematischen Ideen Minkowskis zur SRT reserviert gegenüber: Er sagte: "Das bringt doch physikalisch nichts Neues!" Später erwies sich Minkowskis imaginärer Ansatz als sehr hilfreich zur späteren mathematischen Herleitung der ART-Gleichungen. Einsteins ursprüngliche Herleitung der ART-Gleichungen war nämlich sehr kompliziert und für einen mathematischen Laien nicht nachvollziehbar. Siehe Skizze 2:


Skizze 2

Der Raumzeit-Abstand O-E bleibt durch die Drehung invariant. Das Ereignis E, das im System S durch den Weltpunkt E(x, ict) dargestellt wird, ist im System S’ der Weltpunkt E(x’, ict’). Bei dem Weltpunkt E handelt sich im System S und im System S’ um das gleiche Ereignis, nur die Koordinaten dieses Ereignisses haben im System S andere Werte als im System S’.

Die Transformationsgleichungen nehmen deshalb folgende Gestalt an:

(3) x' = x•cos(iß) + ict•sin(iß); i2 = – 1
(4) ict' = ict•cos(iß) – x•sin(iß)

Zu jedem Zeitpunkt bewegt sich S' mit der Geschwindigkeit v = x/t in Richtung der positiven x-Achse; daraus folgt:

(5) x = v•t

Betrachtet man im 2. Quadranten des Systems S einen beliebigen Punkt (– x, ict), der auf der auf der Ordinate w=ict' liegt, dann lässt sich für den Winkel (iß) der Tangens bestimmen:

(6) tan(iß) = – x/ict; mit x=v•t ergibt sich:
(7) tan(iß) = – v•t/ict
Für alle t ungleich Null ergibt sich:
(8) tan(iß) = – v/(ic); quadriert ergibt:
(9) tan2(iß) = – v2/c2

Der imaginäre Drehwinkel (iß) repräsentiert somit – über den Tangens dieses Winkels – die Relativgeschwindigkeit zwischen den beiden Inertialsystemen S und S' als Bruchteil der Lichtgeschwindigkeit. Aus einer Formelsammlung entnehme ich die Umformungen (10) und (11):

(10) sin(iß) = tan(iß)/sqrt[1 + tan2(iß)
(11) cos(iß) = 1/sqrt[1 + tan2(iß)
Mit der Kurzschreibweise G = 1/sqrt[1 + tan2(iß) ergibt sich:
(12) sin(iß) = G•tan(iß)
(13) cos(iß) = G; (12) und (13) in (3) eingesetzt ergibt:
(14) x' = x•G + ict•G•tan(iß); mit tan(iß) = – v/(ic) ergibt sich:
(15) x' = x•G + ict•G•[– v/(ic)
(16) x' = x•G – t•G•v
(17) x' = G(x – v•t); aus (4) und (12) und (13) ergibt sich:
(18) ict' = ict•G – x•G•tan(iß); mit tan(iß) = – v/(ic) ergibt sich:
(19) ict' = ict•G – x•G•[ – v/(ic)
(20) ict' = G[ ict – x•[ – v/(ic) ]
(21) ict' = G[ict + x·v/(ic); dividiert durch (ic) ergibt:
(22) t' = G(t – x•v/c2)

Wenn man die Beziehung (9) in die Kurzschreibweise
G = 1/sqrt[1 + tan2(iß) einsetzt, dann ergibt sich:

(23) G = 1/sqrt(1 – v2/c2)

Diese Gleichung in (17) und (22) eingesetzt ergeben die speziellen Lorentz-Transformationen:

x' = (x – v•t) / sqrt(1 – v2/c2)
t' = (t – v•x/c2) / sqrt(1 – v2/c2)


Neue Ergänzungen



13.03.2013, 19:99 Die Lorentz-Transformationen, hergeleitet aus der Drehung eines Koordinatensystems in der Minkowski-Raumzeit {x, y, z, w=ict}.
06.04.2013, 11:34 Relativistische Masse gelöscht.
06.04.2013, 11:34 Energie-Impuls-Beziehung neu eingebracht.
16.04.2013, 14:25 Lichtgeschwindigkeits-Defintion.
21.04.2013, 17:45 Relativität der Bewegung + SRT als Sonderfall der ART.
02.05.2013, 16:40 Begriff der Geschwindigkeit ergänzt.
05.10.2013, 11:34 Zwillingsexperiment als Quintessenz mehrerer Fach-Autoren.




Beitrag zuletzt bearbeitet von Bauhof am 05.10.2013 um 11:35 Uhr.

 

  Beitrag 1985-229
Erste Frage

 
 
Bauhof aus 1991-1:
 
Inertialsysteme
Verschiedene Inertialsysteme bewegen sich gegeneinander geradlinig und gleichförmig.
Alle Inertialsysteme bewegen sich relativ zueinander mit einer Geschwindigkeit v<c.


Frage an Dich, Eugen (bzw. an alle):

Ist die SRT auch noch anwendbar auf Bezugssysteme, die sich geradlinig und gleichförmig mit Lichtgeschwindigkeit zueinander bewegen?

Mir ist zwar klar, dass die Formeln der Lorentz-Transformation dann undefiniert werden. Aber könnte man sie nicht als Grenzwert v gegen c an der Stelle v = c dennoch irgendwie sinnvoll anwenden?

Gruß,
grtgrt

Siehe dazu auch die mögliche, in Beitrag 2035-1 skizzierte Anwendung.
 

  Beitrag 1981-1
Zweite Frage

 
 

Eine der wichtigsten Fragen, die uns Physiker noch NICHT beantworten können, ist die Frage nach der Natur der Zeit. Bisher weiß man nur:
Es gibt KEINEN universellen zeitlichen Abstandsbegriff (und auch keine aus Sicht aller Betrachter gegebene Gleichzeitigkeit zweier Ereignisse).

Wie aber steht es mit einem universellen räumlichen Abstandsbegriff?

Hat sich jemand diese Frage überhaupt schon mal gestellt?

 

  Beitrag 1981-2
Die klassische Antwort

 
Hallo Grtgrt,

Grtgrt aus 1981-1:
Eine der wichtigsten Fragen, die uns Physiker noch NICHT beantworten können, ist die Frage nach der Natur der Zeit. Bisher weiß man nur:
Es gibt KEINEN universellen zeitlichen Abstandsbegriff (und auch keine aus Sicht aller Betrachter gegebene Gleichzeitigkeit zweier Ereignisse).

Wie aber steht es mit einem universellen räumlichen Abstandsbegriff?

ich denke, es gibt ebenso keinen universellen Abstandsbegriff.
Ich verstehe die SRT so, dass sowohl gemessene Zeitintervalle wie auch gemessene Raumintervalle abhängig vom gewählten Inertialsystem sind.
Erst Raumzeitintervalle sind unabhängig vom Beobachter invariant. Ein Indiz dafür, dass das Universum erst verstanden werden kann, wenn man die Raumzeit als eine Einheit begreift.
Das zeigt doch schon die Lorentztransformation:
Entfernungen schrumpfen in Bewegungsrichtung im Verhältnis zum Beobachter, welcher zum Ziel der Bewegung ruht.
Ein Beobachter, welcher zu einem entfernten Planeten ruht, wird eine größere Entfernung ermitteln, als ein Beobachter, welcher sich auf den Planeten zubewegt.

mfg okotombrok
 

  Beitrag 1981-4
Die vielleicht bessere (neue) Deutung

 
 
Okotombrok aus 1981-2:
 
Entfernungen schrumpfen in Bewegungsrichtung im Verhältnis zum Beobachter, welcher zum Ziel der Bewegung ruht.
Ein Beobachter, welcher zu einem entfernten Planeten ruht, wird eine größere Entfernung ermitteln, als ein Beobachter, welcher sich auf den Planeten zubewegt.

Hallo Okotombrok,
hallo Henry:

So wirklich gut erklärt ist die Situation hierdurch keineswegs. Man kann sich ja durchaus fragen:

Was genau wollen wir denn eigentlich unter der Entfernung des Beobachters vom Planeten verstehen, wenn der Beobachter auf den Planeten zurast?
Schließlich und endlich benötigt die Messung ja Zeit, und während dieser Zeit kommen Planet und Beobachter einander näher.

Als was also soll der Abstand der beiden dann definiert sein? Als der zu Beginn der Messung oder als der zu Ende der Messung? Oder als Durchschnitt davon?
Und können sich Beobachter und Planet denn überhaupt auf einen gemeinsamen Zeitpunkt "Beginn der Messung" verständigen, wo wir doch wissen, dass es einen wohldefinierten Begriff "gleichzeitig" nach der ART gar nicht geben kann?


Die Situation wird vielleicht klarer, wenn wir noch andere Beispiele betrachten:

    Sei dazu A eine Person, die an einer Straße sitzt, auf der ein Lastwagen fährt, auf dessen Ladefläche eine Person B sitzt, die eine Lichtuhr festhält. Diese Uhr bestehe aus einem 2 Meter hohen, senkrecht neben ihr auf einer Lichtquelle Q stehenden Glasstab, der ganz oben in einem Spiegel endet. Wenn Q ein Photon aussendet, wandert das durch den Stab nach oben, wird dort am Spiegel reflektiert, und kommt so zurück zur Lichtquelle. Aus Sicht von B hat dieses Lichtquant dann also zwei Mal ein und dasselbe Stück einer Geraden durchlaufen: einen geschlossenen Weg Q nach Q von insgesamt genau 4 Metern Länge.
    Relativ zu A aber hat Q sich bewegt, und deswegen hat das Photon aus Sicht von A einen etwas längeren Weg zurückgelegt: Er war nicht geschlossen, denn er war gegeben durch die beiden etwas über 2 Meter langen Seiten eines gleichschenkeligen Dreiecks, dessen Höhe 2 Meter betrug.

Der Konstanz der Lichtgeschwindigkeit wegen wird diese Situation üblicherweise so gedeutet, dass man sagt, aus Sicht von A vergehe für B die Zeit langsamer als für A.

Stimmt das aber wirklich?

Muss man nicht genauer sein und sagen, dass A das Photon auf einem Weg von Q nach Q hat wandern sehen, der ganz andere Form und deswegen auch andere Länge hatte als der Weg, auf dem B es wandern sah.

Diese Interpretation ist sicher auch richtig, denn sie wird gestützt durch die Tatsache, dass die Länge beider Wege über dieselbe Metrik gemessen wurde (über die auf jedem reellen 3-dimensionalen Vektorraum gleich definierten euklidischen Metrik).

Wer das so sieht, wird nicht mehr sagen, dass die Zeit oder die Metrik des Raumes relativ sei. Man würde stattdessen anerkennen, dass der Weg, den das Photon nahm, aus Sicht von A ein anderer war als aus Sicht von B.


Diese Interpretation macht auch dann noch Sinn, wenn der Lastwagen sich mit zunehmender oder abnehmender Geschwindigkeit bewegt (beschleunigt also).

Sie macht auch Sinn, wo es statt um räumliche um zeitliche Abstände geht. Hier ein Beispiel dazu:

    Wir nehmen zwei identisch gebaute Atomuhren, stellen sie nebeneinander und garantieren, dass beide gleichzeitig starten (Ereignis E1).
    Eine dieser Uhren lassen wir stehen, die andere fliegen wir mit einem schnellen Flugzeug einige Male um die Erde, um sie danach wieder an ihren alten Platz neben die erste zu stellen und dann beide gleichzeitig anzuhalten (Ereignis E2).
    Wie die ART uns lehrt — und wie mehrfache Durchführung dieses Experiments ja auch tatsächlich bewiesen hat — werden die Uhren dann NICHT mehr gleiche Aussage machen: Auf der um die Erde geflogenen wird weniger Zeit vergangen sein.

Man interpretiert das üblicherweise so, dass man sagt, für die bewegte Uhr sei die Zeit langsamer vergangen.

Und wieder kann man umgekehrt auch sagen: Beide Uhren gingen gleich schnell, haben aber unterschiedliche Wege durch die Zeit von E1 nach E2 gestoppt.

Diese Interpretation des Versuchsergebnisses entspricht den Tatsachen besser, denn der Weg durch die Raumzeit, den die eine Uhr ging, hat mehr durch die Zeit und weniger durch den Raum geführt als der Weg, den die andere Uhr nahm (jene, die im Flugzeug ganz anderen Beschleunigungen ausgesetzt war).


Fazit also:

Wo bei Messung von zeitlichen oder räumlichen Abständen unterschiedliche Beobachter zu widersprüchlichen Aussagen gelangen, sind solche Widersprüche nicht auf unterschiedliche Metrik zurückzuführen, sondern einfach nur darauf, dass jede der beiden Messungen einen anderen Weg zwischen den beiden betrachteten Punkten der Raumzeit zum Gegenstand hatte.

Und wirklich: In jedem Raum, der mehr als nur eine Dimension hat, gibt es zwischen je zwei Punkten diesen Raumes beliebig viele unterschiedlich lange Wege.
Dies nicht zu vergessen und sich Rechenschaft darüber zu geben, von welchem dieser Wege man im konkreten Fall spricht, kann nicht schaden.


Gruß, grtgrt
 

  Beitrag 1981-39
Beschleunigungskräfte — und nur sie — machen Uhren langsamer

 
 
Tatsachen sind:

Beschleunigungskräfte (und nur sie) machen Uhren langsamer.


Zueinander gleichförmig bewegte Uhren haben lediglich den Eindruck, die jeweils andere würde langsamer gehen:

Tatsächlich aber gehen sie synchron.


 

  Beitrag 1981-55
Was ist Gravitation?

 
 
Bauhof aus 1981-47:
 
dass Beschleunigungskräfte (und nur sie) Uhren langsamer machen, schrieb nicht Henry, sondern Grtgrt.

Ungeachtet dessen ist die Behauptung falsch.

Hi Eugen,

ich bin fest davon überzeugt, dass meine Behauptung richtig ist, mehr noch: dass Gravitation und Beschleunigung ein und dasselbe sind.

Beschleunigungskräfte kennen wir als
  • die   G r a v i t a t i o n s k r a f t   (sie erzeugt Beschleunigung)
  • die   T r ä g h e i t s k r a f t   (sie entsteht als Folge von Beschleunigung)
  • die   F l i e h k r a f t   (sie entsteht ebenfalls als Folge von Beschleunigung).

Je größer die Summe dieser Kräfte ist, desto langsamer vergeht die Zeit,

und umgekehrt: Dehnung der Zeit erzeugt Gravitation.


Zitat von Joachim Schulz:
 
Um zu verstehen, warum Dehnung der Zeit zu Gravitation führt, muss man von der Vorstellung von drei Raum- und einer Zeitdimensionen auf eine 4-dimensionale Vorstellung wechseln. In dieser Vorstellung bewegt sich alles durch die Zeit. Selbst ein ruhendes Objekt eilt im Sauseschritt durch die Dimension Zeit. Und diese Bewegung durch die gleichmäßig vergehende Zeit ist eine gerade Strecke. Versucht nun ein Objekt in der gekrümmten Zeit auf einer Geraden zu laufen, so wird daraus eine Kurve. Eine Kurve in der Raum-Zeit ist aber kein Ruhezustand mehr sondern eine beschleunigte Bewegung. So kommt die Beschleunigung eines Objektes in einer gekrümmten Raum-Zeit zustande. Und das nicht nur für schwere Objekte, sondern auch für Lichtwellen und alles, was sich durch Zeit und Raum bewegt.
 


Wie Gravitation (Beschleunigungskräfte also) die Raumzeit krümmen, versucht man meist über das sog. Gummituchmodell zu illustrieren.

Zitat von Joachim Schulz:
 
Das Gummituchmodell erklärt aber eines gar nicht: Warum krümmt sich der Raum eigentlich?

Ist es die Ruhemasse der Elementarteilchen, die sich in einem Objekt befinden? Das kann nicht sein, denn dann könnte Licht nicht an der Gravitation beteiligt sein. Die Elementarteilchen des Lichts, die Photonen, sind bekanntlich masselos, der Einfluss von Gravitation auf Licht lässt sich aber leicht nachweisen. Ist es also die Gesamtmasse inklusive Bewegungsenergie, die den Raum krümmt? Das würde der allgemeinen Relativitätstheorie, die unabhängig von Beobachtern formulierbar sein muss, nicht gerecht werden. Bewegungsenergie ist eine relative Größe.

Tatsächlich ist es eine etwas kompliziertere Größe, die den Raum krümmt: Der Energie-Impuls-Tensor.

Kennt man von einer Materieverteilung das Feld von Energie-Impuls-Tensoren, so weiß man alles über die Massen-, Energie-, Impuls- und Druckverteilung in dieser Masse.
 

Es macht deswegen Sinn zu sagen:

Gravitation = das Feld aller Energie-Impuls-Tensoren (in unserem Universum)


grtgrt
bezugnehmend auf  Joachim Schulz  und dessen Artikel » Gravitation - Was krümmt den Raum? «

 

  Beitrag 1981-9
Einsteins Definition von Gleichzeitigkeit

 
Grtgrt aus 1981-4:
Als was also soll der Abstand der beiden dann definiert sein? Als der zu Beginn der Messung oder als der zu Ende der Messung? Oder als Durchschnitt davon?
Und können sich Beobachter und Planet denn überhaupt auf einen gemeinsamen Zeitpunkt "Beginn der Messung" verständigen, wo wir doch wissen, dass es einen wohldefinierten Begriff "gleichzeitig" nach der ART gar nicht geben kann?

Hallo Grtgrt,

in der Relativitätstheorie gibt es einen wohldefinierten Begriff der Gleichzeitigkeit, nämlich die Einsteinsche Gleichzeitigkeitsdefinition.

Nach Albert Einstein kann man kann die Gleichzeitigkeit entfernter Ereignisse nicht erkennen, sondern nur definieren. Diese Definition ist willkürlich: man kann irgendeine Festsetzung darüber treffen, ohne dass ein Fehler entsteht.

Nach der Einsteinschen Gleichzeitigkeitsdefinition gelten zwei Ereignisse an verschiedenen Orten A und B als gleichzeitig, wenn ein auf dem Mittelpunkt M der Verbindungsgeraden platzierter Beobachter Lichtsignale von beiden Ereignissen gleichzeitig bei sich eintreffen sieht.

Dass das Licht zum Durchlaufen des Weges A -> M und zum Durchlaufen der Strecke B -> M dieselbe Zeit brauche, ist keine Voraussetzung oder Hypothese über die physikalische Natur des Lichtes, sondern eine Festsetzung, die man nach freiem Ermessen treffen kann, um zu einer Definition der Gleichzeitigkeit zu gelangen.

M.f.G. Eugen Bauhof
 

  Beitrag 1981-10
Warum Einsteins Definition wenig nützt

 
 
Hallo Eugen,

danke für den Hinweis. Und tatsächlich:

Auf den ersten Blick schien mir Einsteins Definition von Gleichzeitigkeit recht sinnvoll.


Bei genaueren Hinsehen aber erkenne ich sie als zu wenig nützlich. Das liegt daran, dass es im konkreten Fall ja gar nicht so einfach ist zu sehen, wo der Mittelpunkt zwischen zwei Orten A und B denn nun genau liegt.
  • Er ist nämlich selbst zeitabhängig (da A und B sich ja i.A. relativ zueinander bewegen),
  • und selbst die A und B verbindende Geodäte der Raumzeit ist ständigem Wandel unterworfen (z.B. ganz gravierend dann, wenn ein großer Stern gerade im Begriff ist, irgendwo in ihrer Nähe vorbeizuziehen).

Mit anderen Worten: Sich einfach eine A und B verbindende gerade Strecke vorzustellen, deren Mittelpunkt man als genau definiert sehen könnte, geht nicht.

Sinn machen kann Einsteins Definition also höchstens im Kontext der für viele Anwendungen zu ungenauen Speziellen Relativitätstheorie (oder wo A und B nicht besonders weit voneinander entfernt sind).

Gruß, grtgrt
 

  Beitrag 1981-16
Was nicht beobachtbar ist, existiert auch nicht (Bohrs Prinzip)

 
 
Bauhof aus 1981-9:
 
Nach Albert Einstein kann man kann die Gleichzeitigkeit entfernter Ereignisse nicht erkennen, ...

Hi Henry,

mir kommt es darauf an, hier bestätigt zu sehen, dass ich in der Sache einer Meinung mit Einstein zu sein scheine.

Gruß, grtgrt
 

  Beitrag 1981-81
Freier Fall im Sinne der Allgemeinen Relativitätstheorie

 
 
Okotombrok aus 1981-78:
 
Der Satellit befindet sich nun im freien Fall um die Erdkugel herum.

Da braucht es keine Flieh- oder Gravitationskräfte, die daran zerren und auch die Insassen können nicht unterscheiden, ob sie im intergalaktischen Raum fernab jeglicher Massen schweben, sich in einem frei fallenden Fahrstuhl befinden oder vom Dach fallen.
Sie können es nicht weil es ununterscheidbar ist.

Es geht letztendlich um die Metrik der Raumzeit und nicht um Kräfte.
 

Hi Okotombrok,

im Prinzip bin ich da deiner Meinung. Und deswegen ist mir klar, dass hier ganz sicher keine Fliehkraft mehr wirkt.

Dass sich Erde und Satellit aber auch nicht mehr per Gravitationskraft anziehen sollten, finde ich schwer einzusehen. Frage also:

Muss so ein Satellit, wenn man will, dass er die gewünschte Umlaufbahn wirklich über Jahrtausende hinweg einhält, nicht doch hin und wieder ein klein wenig beschleunigt werden?

Gruß, grtgrt
 

  Beitrag 2058-16
Ursache der Relativität von Zeit und Raum

 
 
Henry in 2058-13:
 
Und Raum und Zeit sind relativ? Es geht doch um die Raumzeit als Gesamtheit, und die ist nicht relativ.


Wie kommst Du auf die Idee, dass räumliche und zeitliche Abstände in der ART nicht ebenso relativ sein sollen wie in der SRT?

Meiner Ansicht nach besteht der Unterschied beider Theorien in dieser Hinsicht nur darin, dass
  • unbeschleunigte Bewegung im Minkowski-Raum stets eine geradlinige Bewegung ist,
  • wohingegen sie in der Raumzeit entlang einer (gekrümmten) Geodäte vor sich geht — und deswegen weit schwieriger zu beschreiben ist.

Die Relativität von Zeit und Raum (d.h. die relative Wahrnehmung der Länge von örtlichen oder zeitlichen Abständen) geht einzig und alleine zurück auf die beiden Axiome:
  • Die Lichtgeschwindigkeit ist endlich und
  • Information über eingetretene Ereignisse breitet sich grundsätzlich mit Lichtgeschwindigkeit aus.

Nicht mal die Konstanz der Lichtgeschwindigkeit ist für Relativität notwendig. Dass sie tatsächlich gegeben ist, vereinfacht die Rechnungen natürlich erheblich.

Gruß, grtgrt
 

  Beitrag 2065-1
Zur Relativität kosmischer Abstände (in zeitlicher und örtlicher Hinsicht)

 
 


Warum Alter und Größe kosmischer Objekte beobachterabhängig sein müssen



In Jukka Maalampi: Die Weltlinie – Einstein und die moderne Physik findet sich in Abschnitt 5.6 ein interessanter Vergleich:


Zitat von Maalampi (nicht wörtlich):
 
Einsteins Annahme, dass die Lichtgeschwindigkeit konstant ist, mag unschuldig aussehen, hat aber dramatische Folgen:

Wenn ich mich z.B. dadurch beschreibe, dass ich 182 cm groß und 57 Jahre alt bin, hat diese Beschreibung keine absolute Bedeutung, denn:
  • Aliens, deren Raumschiff unser Sonnensystem mit 80% Lichtgeschwindigkeit durchquert, würden sehen, dass ich nur etwa 100 cm groß und 83 Jahre alt bin.
  • Würden sie gar mit Lichtgeschwindigkeit reisen, sähen sie mich als unendlich klein und unendlich alt.


Das Beispiel finde ich interessant, denn wenn wir davon ausgehen, dass unser Universum unendlich groß, homogen und isotrop sowie in ständiger Ausdehnung begriffen ist, muss es darin Materie geben, die sich von uns mit jeder nur denkbaren Geschwindigkeit entfernt, insbesondere mit fast Lichtgeschwindigkeit.

Würde das dann aber nicht bedeuten, dass es z.B. Materie gibt, aus deren Sicht der Durchmesser unserer Milchstraße kleiner als 1 Meter und ihr Alter weit größer als 20 Mrd. Jahre ist?

Schlimmer noch: Wie können wir bei solch unvermeidbarer Relativität denn überhaupt noch davon sprechen, dass unser Universum ein bestimmtes Alter habe? Müsste dann nicht vielmehr das durch uns beobachtbare Alter durch uns beobachtbarer Galaxien umso größer sein, je weiter sie von uns entfernt sind (bzw. je schneller sie sich relativ zu uns bewegen)?

Anders gefragt:

Ist – so gesehen – der Begriff » Alter des Universums « denn überhaupt wohldefinierbar?


 

  Beitrag 2065-2
Lichthorizont und Zeithorizont

 
 

Lichthorizont und Zeithorizont



Mal angenommen, wir können unserem Universum tatsächlich ein wohldefiniertes Alter zuordnen (was dann derzeit bei 13.81 Mrd. Jahren sieht, was nach meiner Argumentation in 2065-1 aber vielleicht doch mit einem grundsätzlichen Fragezeichen versehen sein müsste).

Sollte unser Universum unendlich groß sein, gibt es dann zwei gedachte, unterschiedlich große Kugeloberflächen um uns herum, die äußerst interessant sind:

  • Die eine nenne ich unseren Lichthorizont — er grenzt den uns im Prinzip beobachtbaren Teil des Universums ab von dem, der uns nicht einsehbar sein kann in dem Sinne, das uns von dort kein Licht erreichen kann, da es, uns zu erreichen, länger unterwegs sein müsste als das Alter unseres Universums ihm gestattet.
  • Die andere nenne ich unseren Zeithorizont — er grenzt den Teil unseres Universums, den wir zeitlich einordnen können (genauer: in dem unser Zeitbegriff überhaupt anwendbar ist) ab vom unendlich großen Rest des Universums. Die entsprechende Kugel hat endlichen, derzeit mit Lichtgeschwindigkeit wachsenden Durchmesser, und doch hat sie — in zeitlicher Hinsicht — unendlich großen Durchmesser (da alles, was kurz vor diesem Horizont liegt, aus unserer Sicht fast unendlich großes Alter hat).
    Würde unser Universum irgendwann mal statt zu expandieren schrumpfen, würde der Radius des Zeithorizontes mit Lichtgeschwindigkeit kleiner werden, jene Kugel in Sinne unserer Zeit aber dennoch weiter unendlich großen Durchmesser haben.
    Ein Schrumpfen ebenso wie ein Expandieren unserer Raumzeit modifiziert von uns wahrgenommene zeitliche Abstände: Sie schrumpfen bzw. wachsen im selben Umfang, in dem unser Universum schrumpft oder sich aufbläht.

 
Ich würde mich freuen, wenn jemand versuchen wollte, in dieser Argumentation Fehler zu entdecken.

Sie setzt nur voraus, dass auf der Geodäte, die uns mit irgendeinem Ort im Universum verbindet — auf dem Weg also, den das Licht nimmt —, für zueinander gleichförmig bewegte Beobachter alle Voraussetzungen der SRT gegeben sind.

Gebhard Greiter (grtgrt)
 

  Beitrag 2066-1
Warum Information sich niemals schneller als das Licht ausbreiten kann

 
 

Hin und wieder frage ich mich, warum man so sicher ist,

dass Information sich niemals schneller als das Licht ausbreiten kann.


In Jukka Maalampi: Die Weltlinie – Einstein und die moderne Physik findet sich in Abschnitt 6.2 die Begründung dafür:



Zitat von Maalampi (gekürzt, nicht wörtlich):
 
Die Relativitätstheorie in Kombination mit der vernünftig erscheinenden Zusatzvoraussetzung, dass Informationsübertragung sämtliche Kausalitätsbeziehungen erhalten wird, führt zur Erkenntnis, dass Information sich niemals schneller als das Licht ausbreiten kann.

Wenn zwei Ereignisse an verschiedenen Orten stattfinden und keinerlei kausalen Zusammenhang aufweisen, hängt ihre zeitliche Reihenfolge von der Relativgeschwindigkeit des Beobachters in Bezug auf das betreffende Ereignispaar ab. Die chronologische Reihenfolge, von Ereignissen, die unabhängig voneinander an verschiedenen Orten stattfinden, ist relativ.

Die Erhaltung der Ursache-Folge-Beziehung gilt in der Reletivitätstheorie aber nur unter der Voraussetzung, dass sich ein Effekt mit höchstens Lichtgeschwindigkeit auf Folgen davon überträgt. Ginge es nämlich schneller, dann könnte sich aus der Sicht einiger Beobachter die Folge vor der Ursache ereignen.
 

 

  Beitrag 2068-1
Fragen zur Relativität Schwarzer Löcher

 
 

In Jukka Maalampis Buch » Die Weltlinie – Einstein und die moderne Physik (2008) « liest man:

Zitat von Maalampi (S. 128):
 
Ein Schwarzes Loch ist ein extremes Beispiel für die von der Relativitätstheorie vorhergesagte Gravitations-Rotverschiebung: Je näher sich das in Bewegung setzende Licht am Ereignishorizont befindet, desto größer wird seine Wellenlänge. Die Wellenlänge des vom Horizont ausgehenden Lichtes dehnt sich ins Unendliche aus. So etwas ist gar keine Welle mehr, und auch die Energie geht gegen Null, so dass dort Strahlung aufhört Strahlung zu sein.


Da frägt man sich nun unwillkürlich, wie sich denn dort Längen und Horizont (des Schwarzen Loches) relativieren:

FRAGEN also:

Wenn wir uns einen Beobachter vorstellen, der direkt vor dem Ereignishorizont eines Schwarzen Loches sitzt,
  • wie groß ist seiner Wahrnehmung nach der Radius des Schwarzen Loches bzw. sein Horizont?
  • Ist es richtig, dass aus Sicht eines sich dem Horizont näherenden Beobachters beide gegen Null gehen?
  • Und was genau sieht die ART als singulär (nur den Mittelpunkt des Schwarzen Lochs oder tatsächlich die gesamte Region im Inneren seines Horizonts)?

Schon auf dem Horizont jedenfalls steht die Zeit offenbar still, denn:

Zitat von Maalampi (S. 129, gekürzt, nicht wörtlich):
 
Wenn jemand frägt, wie man die Zeit anhalten — bzw. aus Sicht anderer ewig jung bleiben — könne, gibt es mindestens eine Antwort: Man begebe sich an den Horizont eines Schwarzen Loches. Wenn für jemand in unmittelbaren Nähe eines solchen Horizonts Wochen, Tage, oder gar nur Stunden vergehen, entspricht das auf der Erde Jahrtausenden.
 

 

  Beitrag 1997-80
Erst unterschiedlich beschleunigt zu werden gibt Zwillingen unterschiedliches Alter (?)

 


Beschleunigung — und wirklich nur sie — dehnt Eigenzeit

Henry in 1997-67:
 
Um nicht immer aus Wikipedia zu zitieren:

» Durch Einstein wurde unser Verständnis von Raum und Zeit radikal neu gestaltet: Phänomene wie Zeitdilatation und Lorentz-Kontraktion und die Verschmelzung von Raum und Zeit im Raum-Zeit-Kontinuum sind eine natürliche Konsequenz der Speziellen Relativitätstheorie. «


Ein Zitat aus http://www.wissenschaft-online.de/astrowissen/ von Dr. Andreas Müller.

Der Autor Dr. Andreas Müller ist Astrophysiker und wissenschaftlicher Koordinator im Exzellenzcluster "Origin and Structure of the Universe" der Technischen Universität München.


Hallo Henry,

diese Aussage Müllers ist richtig — aber eben  n u r  wegen der Vergröberung, mit der sie formuliert ist. Diese Vergröberung (ein Ausblenden wirklich wichtiger Details) besteht darin, dass diese Formulierung den qualitativen Unterschied zwischen der Raumzeit einerseits und beobachterspezifischen  S i c h t e n  darauf andererseits völlig ignoriert.

Tatsache ist:

Eine Dilation der Zeit und zu ihr korrelierte Kontraktion von Längen gibt es NUR im Sinne der  S i c h t e n ,
aber keineswegs im Sinne der Struktur der Raumzeit (der SRT) selbst.


Dass dem wirklich so ist, erkennt man sehr schön, wenn man sich zwei Personen X und Y vorstellt, die sich mit gleichförmiger Geschwindigkeit zuerst voneinander ent­fernen um dann, wenn jeder den jeweils anderen in einer Entfernung von genau 100 km vermutet, mit eben derselben Geschwindigkeit wieder aufeinander zuzufliegen.
Bitte beachte:
  • Dies ist ein absolut symmetrisches Szenario.
  • Aus Sicht der einen Person geht die Uhr der jeweils anderen langsamer,
  • und doch werden beide Uhren — wenn die beiden Personen sich wieder treffen — genau gleiche Zeit anzeigen (das folgt aus der Symmetrie des Szenarios und ist deswegen so, weil X und Y dann ja auch wieder dasselbe Bezugssystem haben).

Wir sehen also: Das Zwillingsparadoxon gibt es  n u r  in der Raumzeit der ART, aber  n i c h t  in der Raumzeit der SRT.

In der SRT sind beobachtete Unterschiede wirklich  n u r  darauf zurückzuführen, dass Beobachter, die solche Unterschiede feststellen, aus unterschiedlichen Bezugs­systemen heraus argumentieren.

In der ART dageben kommt unterschiedliches Altern der Zwillinge durchaus zustande, da hier auch die für beide Personen unterschiedlichen Beschleunigungen mit be­rücksichtigt werden: eine Art von Kraftwirkung also, die die SRT gar nicht erst zu betrachten versucht.

Gruß,
grtgrt
 

PS: Eine erweiterte, über Einstein hinausgehende Form der SRT löst das Zwillingsparadoxon dennoch. Genauer:

Obgleich Einstein selbst im Rahmen der SRT niemals auch beschleunigte Bewegung diskutiert hat, hat man das — so etwa um das Jahr 2000 herum — dennoch versucht und hierbei schnell festgestellt, dass die Lorentztransformation der SRT auch Aussagen darüber machen kann, wie sich Beschleunigung auf das beschleu­nigte System auswirkt (siehe etwa ein durch Joachim Schulz beschriebenes Gedankenexperiment).

Dass solche Ergebnisse tatsächlich mit denen der ART übereinstimmen, wird — wenigstens für die dem Zwillingsparadoxon zugrundeliegende Situation — explizit nachgerechnet von Bernd Sonne und Reinhard Weiß in ihrem Buch Einsteins Theorien: Spezielle und Allgemeine Relativitätstheorie für interessierte Einsteiger und zur Wiederholung (Springer, 2013). Ihre Rechnung auf Seite 111 bis 129 des Buches zeigt klar, dass auch die SRT den für die Zwillinge entstandenen Altersunterschied

ausschließlich auf jene Phasen der Reise zurückführt, in denen die beiden Zwillinge unterschiedlich beschleunigt waren.


FAZIT also:
  • Wer von der SRT (in Einsteins Fassung) ausgeht, geht von einer Theorie aus, die zu beschleunigten Bewegungen nichts aussagen will und demnach auf die Situation des Zwillingsparadoxon gar nicht anwendbar ist.
  • Seit etwa 2000 aber geht man nicht mehr davon aus, dass die SRT — wenn man versucht, sie auch auf beschleunigte Bewegung anzuwenden — falsche Aussagen macht. Soweit man nämlich Beispiele in SRT  u n d  ART durchgerechnet hat, kam man zum gleichen Ergebnis (was aber nicht heißt, dass wirklich alles, was die ART sagt, auch mit Mitteln der SRT nachrechenbar wäre).
    Es kommt hier wohl die Tatsache zum Tragen, dass in jeder hinreichend kleinen Umgebung eines nicht singulären Punktes P der Raumzeit der ART die SRT sehr gute Approximation der ART ist.



 

  Beitrag 1985-356
Beschleunigung (jeder Art) entspricht einer Krümmung der Raumzeit

 
 
Henry in 1985-354:
 
Also nur als Ergänzung: Man kann Gravitation und Beschleunigung nicht mit denselben Gleichungen behandeln, weil Beschleunigung durch eine Kraft erzeugt wird, während die Gravitation laut ART aber keine Kraft ist, sondern die Krümmung der Raumzeit, also eine geometrische Beschreibung darstellt. Gravitation und Beschleunigung sind äquivalent, was ihre Wirkung angeht, aber nicht identisch.


Hallo Henry,

ich habe Zweifel, ob wirklich richtig ist, was Du hier sagst.

Schließlich hat Einstein selbst — z.B. mit seinem Fahrstuhl-Gedankenexperiment — immer wieder darauf hingewiesen, dass Kräfte, die beschleunigen, sich ihrer Ursache nach in keiner Weise unterscheiden lassen: Sie alle gehorchen ein und demselben mathematischen Gesetz, und so müssen sie sich doch wohl auch  a l l e  als Raumkrümmung deuten lassen.


Gruß, grtgrt

PS: Wo auf ein Objekt Beschleunigungskräfte unterschiedlicher Herkunft wirken, wird das Objekt sie stets als in genau  e i n e  Richtung ziehende Kraft wahrnehmen ganz so als hätten sie nur eine einzige Quelle und Ursache (was wirkt, ist die vektorielle Summe aller Kräfte).

Die Krümmung des Raumes im Sinne der ART scheint mir durch die Forderung definiert, dass dieser Kraftvektor überall zum Nullvektor werden muss.

Im übrigen ist es genau (und nur) diese Forderung, welche garantiert, dass jeder Punkt der Raumzeit (auch der ART) eine Umgebung hat, in der — wenn man sie klein genug wählt — die Gesetze der SRT gelten.